PArt 1 of patho 2 Final

अब Quizwiz के साथ अपने होमवर्क और परीक्षाओं को एस करें!

18. A patient with severe, chronic COPD uses an inhaled LABA/glucocorticoid but continues to have frequent exacerbations of symptoms. The nurse will contact the provider to discuss: a. adding roflumilast [Daliresp] once daily. b. changing to oral theophylline twice daily. c. prescribing oral steroids once daily. d. Using an ipratropium/albuterol combination twice daily.

ANS: A For patients with chronic, severe COPD, the risk of exacerbations may be reduced with roflumilast. Theophylline is used only when other bronchodilators are not effective. Oral steroids are not indicated for this use. Ipratropium is used to treat bronchospasm in COPD.

1. A patient is brought to the emergency department with shortness of breath, a respiratory rate of 30 breaths per minute, intercostal retractions, and frothy, pink sputum. The nurse caring for this patient will expect to administer which drug? a. Furosemide [Lasix] b. Hydrochlorothiazide [HydroDIURIL] c. Mannitol [Osmitrol] d. Spironolactone [Aldactone]

ANS: A Furosemide, a potent diuretic, is used when rapid or massive mobilization of fluids is needed. This patient shows severe signs of congestive heart failure with respiratory distress and pulmonary edema and needs immediate mobilization of fluid. Hydrochlorothiazide and spironolactone are not indicated for pulmonary edema, because their diuretic effects are less rapid. Mannitol is indicated for patients with increased intracranial pressure and must be discontinued immediately if signs of pulmonary congestion or heart failure occur.

5. A parent asks a nurse about growth suppression resulting from the use of an inhaled glucocorticoid in children. What will the nurse tell the parent? a. Growth may be slowed, but eventual adult height will not be reduced. b. The growth rate is not impaired, but overall height will be reduced. c. The growth rate slows while the drug is used and only resumes when the drug is stopped. d. Long-term use of the drug results in a decrease in adult height.

ANS: A Glucocorticoids can slow growth in children and adolescents, but they do not reduce the eventual adult height. The growth rate will return to normal within a year, even when the drug is continued. Long-term use does not affect the eventual adult height.

8. An older adult patient with congestive heart failure develops crackles in both lungs and pitting edema of all extremities. The physician orders hydrochlorothiazide [HydroDIURIL]. Before administering this medication, the nurse reviews the patient's chart. Which laboratory value causes the nurse the most concern? a. Elevated creatinine clearance b. Elevated serum potassium level c. Normal blood glucose level d. Low levels of low-density lipoprotein (LDL) cholesterol

ANS: A Hydrochlorothiazide should not be given to patients with severe renal impairment; therefore, an elevated creatinine clearance would cause the most concern. Thiazide diuretics are potassium-wasting drugs and thus may actually improve the patient's potassium level. Thiazides may elevate the serum glucose level in diabetic patients. Thiazides increase LDL cholesterol; however, this patient's levels are low, so this is not a risk.

17. A patient with stable COPD is prescribed a bronchodilator medication. Which type of bronchodilator is preferred for this patient? a. A long-acting inhaled beta2 agonist b. An oral beta2 agonist c. A short-acting beta2 agonist d. An intravenous methylxanthine

ANS: A LABAs are preferred over SABAs for COPD. Oral beta2 agonists are not first-line therapy. Although theophylline, a methylxanthine, was once standard therapy in COPD, it is no longer recommended. It is used only if beta2 agonists are not available.

7. A 7-year-old child with asthma uses a daily inhaled glucocorticoid and an albuterol MDI as needed. The provider has added montelukast [Singulair] to the child's regimen. Which statement by the child's parent indicates understanding of this medication? a. "I may notice mood changes in my child." b. "I should give this medication twice daily." c. "I will give my child one 4-mg chewable tablet daily." d. "This drug can alleviate symptoms during an acute attack."

ANS: A Montelukast is given as an adjunct to inhaled glucocorticoids to help prevent inflammation. Some patients have reported mood changes when taking this drug, so parents should be warned of this effect. The medication is given once daily. The dose for a 7-year-old child is 5 mg daily. The drug does not treat symptoms of an acute attack.

26. A patient who has recently started taking pravastatin (Pravachol) and niacin (Nicobid) reports the following symptoms to the nurse. Which is most important to communicate to the health care provider? a. Generalized muscle aches and pains b. Dizziness when changing positions quickly c. Nausea when taking the drugs before eating d. Flushing and pruritus after taking the medications

ANS: A Muscle aches and pains may indicate myopathy and rhabdomyolysis, which have caused acute kidney injury and death in some patients who have taken the statin medications. These symptoms indicate that the pravastatin may need to be discontinued. The other symptoms are common side effects when taking niacin, and although the nurse should follow-up with the health care provider, they do not indicate that a change in medication is needed.

Lovastatin [Mevacor] is prescribed for a patient for the first time. The nurse should provide the patient with which instruction? a. "Take lovastatin with your evening meal." b. "Take this medicine before breakfast." c. "You may take lovastatin without regard to meals." d. "Take this medicine on an empty stomach."

ANS: A Patients should be instructed to take lovastatin with the evening meal. Statins should be taken with the evening meal, not before breakfast. Statins should not be administered without regard to meals and should not be taken on an empty stomach.

8. A patient with Stage C heart failure (HF) who has been taking an ACE inhibitor, a beta blocker, and a diuretic begins to have increased dyspnea, weight gain, and decreased urine output. The provider orders spironolactone [Aldactone]. The nurse will instruct the patient to: a. avoid potassium supplements. b. monitor for a decreased heart rate. c. take extra fluids. d. use a salt substitute instead of salt.

ANS: A Spironolactone is added to therapy for patients with worsening symptoms of HF. Because spironolactone is a potassium-sparing diuretic, patients should not take supplemental potassium. Patients taking digoxin need to monitor their heart rate. Extra fluids are not indicated. Salt substitutes contain potassium.

10. The potassium-sparing diuretic spironolactone [Aldactone] prolongs survival and improves heart failure symptoms by which mechanism? a. Blocking aldosterone receptors b. Increasing diuresis c. Reducing venous pressure d. Reducing afterload

ANS: A Spironolactone prolongs survival in patients with HF primarily by blocking receptors for aldosterone. Spironolactone cause only minimal diuresis. It does not reduce afterload, and it does not reduce venous pressure enough to prolong survival, because it causes only minimal diuresis.

A postmenopausal woman will begin taking atorvastatin [Lipitor] to treat hypercholesterolemia. The woman reports a history of osteopenia with a family risk of osteoporosis. What will the nurse include when teaching this patient? a. The need to discuss taking a bisphosphonate medication with her provider b. That statins are known to reduce the risk of osteoporosis c. That she should avoid foods high in calcium d. To discuss vitamin D supplements with her provider since statins deplete calcium

ANS: A Studies demonstrating a protective effect of statins in reducing the risk of osteoporosis have been inconclusive. Women at risk should discuss taking a bisphosphonate medication with their providers. Statins are not known to reduce the risk of osteoporosis. She should consume foods containing calcium. Vitamin D supplements are not indicated.

14. A patient has just received a prescription for fluticasone/salmeterol [Advair Diskus]. What will the nurse include as part of the teaching for this patient about the use of this device? a. "You do not need good hand-lung coordination to use this device." b. "You will begin to inhale before activating the device." c. "You will need to use a spacer to help control the medication." d. "You will take 2 inhalations twice daily."

ANS: A The Advair Diskus is a dry powder inhaler and is activated by inhalation; therefore, hand-lung coordination is not required. There is no need for a spacer. Patients who use the Diskus take 1 inhalation twice daily.

3. A patient who uses an inhaled glucocorticoid for chronic asthma calls the nurse to report hoarseness. What will the nurse do? a. Ask whether the patient is rinsing the mouth after each dose. b. Request an order for an antifungal medication. c. Suggest that the patient be tested for a bronchial infection. d. Tell the patient to discontinue use of the glucocorticoid.

ANS: A The most common side effects of inhaled glucocorticoids are oropharyngeal candidiasis and dysphonia. To minimize these, patients should be advised to gargle after each administration. Antifungal medications are used after a fungal infection has been diagnosed. Hoarseness is not a sign of a bronchial infection. There is no need to discontinue the glucocorticoid.

27. A patient who is being admitted to the emergency department with intermittent chest pain gives the following list of medications to the nurse. Which medication has the most immediate implications for the patient's care? a. Sildenafil (Viagra) b. Furosemide (Lasix) c. Captopril (Capoten) d. Warfarin (Coumadin)

ANS: A The nurse will need to avoid giving nitrates to the patient because nitrate administration is contraindicated in patients who are using sildenafil because of the risk of severe hypotension caused by vasodilation. The other home medications also should be documented and reported to the health care provider but do not have as immediate an impact on decisions about the patient's treatment.

38. The nurse in the emergency department receives arterial blood gas results for four recently admitted patients with obstructive pulmonary disease. Which patient will require the most rapid action by the nurse? a. 22-year-old with ABG results: pH 7.28, PaCO2 60 mm Hg, and PaO2 58 mm Hg b. 34-year-old with ABG results: pH 7.48, PaCO2 30 mm Hg, and PaO2 65 mm Hg c. 45-year-old with ABG results: pH 7.34, PaCO2 33 mm Hg, and PaO2 80 mm Hg d. 65-year-old with ABG results: pH 7.31, PaCO2 58 mm Hg, and PaO2 64 mm Hg

ANS: A The pH, PaCO2, and PaO2 indicate that the patient has severe uncompensated respiratory acidosis and hypoxemia. Rapid action will be required to prevent increasing hypoxemia and correct the acidosis. The other patients also should be assessed as quickly as possible but do not require interventions as quickly as the 22-year-old. DIF: Cognitive Level: Analyze (analysis) REF: 566 OBJ: Special Questions: Prioritization; Multiple Patients TOP: Nursing Process: Assessment MSC: NCLEX: Safe and Effective Care Environment

19. Three days after experiencing a myocardial infarction (MI), a patient who is scheduled for discharge asks for assistance with hygiene activities, saying, "I am too nervous to take care of myself." Based on this information, which nursing diagnosis is appropriate? a. Ineffective coping related to anxiety b. Activity intolerance related to weakness c. Denial related to lack of acceptance of the MI d. Disturbed personal identity related to understanding of illness

ANS: A The patient data indicate that ineffective coping after the MI caused by anxiety about the impact of the MI is a concern. The other nursing diagnoses may be appropriate for some patients after an MI, but the data for this patient do not support denial, activity intolerance, or disturbed personal identity.

35. A patient who is experiencing an asthma attack develops bradycardia and a decrease in wheezing. Which action should the nurse take first? a. Notify the health care provider. b. Document changes in respiratory status. c. Encourage the patient to cough and deep breathe. d. Administer IV methylprednisolone (Solu-Medrol).

ANS: A The patient's assessment indicates impending respiratory failure, and the nurse should prepare to assist with intubation and mechanical ventilation after notifying the health care provider. IV corticosteroids require several hours before having any effect on respiratory status. The patient will not be able to cough or deep breathe effectively. Documentation is not a priority at this time. DIF: Cognitive Level: Apply (application) REF: 565 OBJ: Special Questions: Prioritization TOP: Nursing Process: Implementation MSC: NCLEX: Physiological Integrity

15. A patient who has been newly diagnosed with asthma is referred to an asthma clinic. The patient reports daily symptoms requiring short-acting beta2-agonist treatments for relief. The patient has used oral glucocorticoids three times in the past 3 months and reports awakening at night with symptoms about once a week. The patient's forced expiratory volume in 1 second (FEV1) is 75% of predicted values. The nurse will expect this patient to be started on which regimen? a. Daily low-dose inhaled glucocorticoid/LABA with a SABA as needed b. Daily low-dose inhaled glucocorticoid and a SABA as needed c. Daily medium-dose inhaled glucocorticoid/LABA combination d. No daily medications; just a SABA as needed

ANS: A This patient has moderate persistent asthma, which requires step 3 management for initial treatment. Step 3 includes daily inhalation of a low-dose glucocorticoid/LABA combination supplemented with a SABA as needed. A daily low-dose glucocorticoid with an as-needed SABA is used for step 2 management. A daily medium-dose glucocorticoid/LABA is used for step 4 management. Patients requiring step 1 management do not need daily medications.

5. A patient is recovering from a myocardial infarction but does not have symptoms of heart failure. The nurse will expect to teach this patient about: a. ACE inhibitors and beta blockers. b. biventricular pacemakers. c. dietary supplements and exercise. d. diuretics and digoxin.

ANS: A This patient is classified as having Stage B heart failure with no current symptoms but with structural heart disease strongly associated with the development of heart failure. Treatment at this stage includes an ACE inhibitor and a beta blocker to help prevent the progression of symptoms. Biventricular pacemakers are used for patients in Stage C heart failure and have more advanced structural disease and symptoms. Dietary supplements and exercise have not been proven to prevent structural heart disease. Diuretics and digoxin are used for patients with Stage C heart failure.

A patient arrives in the emergency department complaining of chest pain that has lasted longer than 1 hour and is unrelieved by nitroglycerin. The patient's electrocardiogram reveals elevation of the ST segment. Initial cardiac troponin levels are negative. The patient is receiving oxygen via nasal cannula. Which drug should be given immediately? a. Aspirin 325 mg chewable b. Beta blocker given IV c. Ibuprofen 400 mg orally d. Morphine intravenously

ANS: A This patient shows signs of acute ST-elevation myocardial infarction (STEMI). Because cardiac troponin levels usually are not detectable until 2 to 4 hours after the onset of symptoms, treatment should begin as symptoms evolve. Chewable aspirin (ASA) should be given immediately to suppress platelet aggregation and produce an antithrombotic effect. Beta blockers are indicated but do not have to be given immediately. Ibuprofen is contraindicated. Morphine is indicated for pain management and should be administered after aspirin has been given.

8. A young adult patient who denies any history of smoking is seen in the clinic with a new diagnosis of chronic obstructive pulmonary disease (COPD). It is most appropriate for the nurse to teach the patient about a. α1-antitrypsin testing. b. use of the nicotine patch. c. continuous pulse oximetry. d. effects of leukotriene modifiers.

ANS: A When COPD occurs in young patients, especially without a smoking history, a genetic deficiency in α1-antitrypsin should be suspected. Because the patient does not smoke, a nicotine patch would not be ordered. There is no indication that the patient requires continuous pulse oximetry. Leukotriene modifiers would be used in patients with asthma, not with COPD. DIF: Cognitive Level: Apply (application) REF: 582-583 TOP: Nursing Process: Planning MSC: NCLEX: Physiological Integrity

21. A few days after experiencing a myocardial infarction (MI) and successful percutaneous coronary intervention, the patient states, "I just had a little chest pain. As soon as I get out of here, I'm going for my vacation as planned." Which reply would be most appropriate for the nurse to make? a. "What do you think caused your chest pain?" b. "Where are you planning to go for your vacation?" c. "Sometimes plans need to change after a heart attack." d. "Recovery from a heart attack takes at least a few weeks."

ANS: A When the patient is experiencing denial, the nurse should assist the patient in testing reality until the patient has progressed beyond this step of the emotional adjustment to MI. Asking the patient about vacation plans reinforces the patient's plan, which is not appropriate in the immediate post-MI period. Reminding the patient in denial about the MI is likely to make the patient angry and lead to distrust of the nursing staff.

18. A patient has been taking digoxin [Lanoxin] 0.25 mg, and furosemide [Lasix] 40 mg, daily. Upon routine assessment by the nurse, the patient states, "I see yellow halos around the lights." The nurse should perform which action based on this assessment? a. Check the patient for other symptoms of digitalis toxicity. b. Withhold the next dose of furosemide. c. Continue to monitor the patient for heart failure. d. Document the findings and reassess in 1 hour.

ANS: A Yellow halos around lights indicate digoxin toxicity. The use of furosemide increases the risk of hypokalemia, which in turn potentiates digoxin toxicity. The patient should also be assessed for headache, nausea, and vomiting, and blood should be drawn for measurement of the serum digoxin level. The nurse should not withhold the dose of furosemide until further assessment is done, including measurement of a serum digoxin level. No evidence indicates that the patient is in worsening heart failure. Documentation of findings is secondary to further assessment and prevention of digoxin toxicity.

The nurse notes that a patient who was admitted with diabetic ketoacidosis has rapid, deep respirations. Which action should the nurse take? a. Notify the patient's health care provider. b. Give the prescribed PRN lorazepam (Ativan). c. Start the prescribed PRN oxygen at 2 to 4 L/min. d. Encourage the patient to take deep, slow breaths.

ANS: A Notify the patient's health care provider. The rapid, deep (Kussmaul) respirations indicate a metabolic acidosis and the need for actions such as administration of sodium bicarbonate, which will require a prescription by the health care provider. Oxygen therapy is not indicated because there is no indication that the increased respiratory rate is related to hypoxemia. The respiratory pattern is compensatory, and the patient will not be able to slow the respiratory rate. Ativan administration will slow the respiratory rate and increase the level of acidosis.

Which assessment finding about a patient who has a serum calcium level of 7.0 mEq/L is most important for the nurse to report to the health care provider? a. The patient is experiencing laryngeal stridor. b. The patient complains of generalized fatigue. c. The patient's bowels have not moved for 4 days. d. The patient has numbness and tingling of the lips.

ANS: A The patient is experiencing laryngeal stridor. Laryngeal stridor may lead to respiratory arrest and requires rapid action to correct the patient's calcium level. The other data also are consistent with hypocalcemia, but do not indicate a need for immediate action.

The nurse in the outpatient clinic who notes that a patient has a decreased magnesium level should ask the patient about a. daily alcohol intake. b. intake of dietary protein. c. multivitamin/mineral use. d. use of over-the-counter (OTC) laxatives.

ANS: A daily alcohol intake. Hypomagnesemia is associated with alcoholism. Protein intake would not have a significant effect on magnesium level. OTC laxatives (such as milk of magnesia) and use of multivitamin/mineral supplements would tend to increase magnesium level.

A patient has the following arterial blood gas (ABG) results: pH 7.32, PaO2 88 mm Hg, PaCO2 37 mm Hg, and HCO3 16 mEq/L. The nurse interprets these results as a. metabolic acidosis. b. metabolic alkalosis. c. respiratory acidosis. d. respiratory alkalosis.

ANS: A metabolic acidosis. The pH and HCO3 indicate that the patient has a metabolic acidosis. The ABGs are inconsistent with the other responses.

A patient who has been receiving diuretic therapy is admitted to the emergency department with a serum potassium level of 3.1 mEq/L. Of the following medications that the patient has been taking at home, the nurse will be most concerned about a. oral digoxin (Lanoxin) 0.25 mg daily. b. ibuprofen (Motrin) 400 mg every 6 hours. c. metoprolol (Lopressor) 12.5 mg orally daily. d. lantus insulin 24 U subcutaneously every evening.

ANS: A oral digoxin (Lanoxin) 0.25 mg daily Hypokalemia increases the risk for digoxin toxicity, which can cause serious dysrhythmias. The nurse also will need to do more assessment regarding the other medications, but there is not as much concern with the potassium level.

Which drugs are used to treat COPD? (Select all that apply.) a. Anticholinergic medications b. Glucocorticoids c. Leukotriene modifiers d. Long-acting beta2 agonists e. Monoclonal antibodies

ANS: A, B, D Anticholinergic drugs, glucocorticoids, and LABAs are used to treat COPD. Leukotriene modifiers and monoclonal antibodies are used only to treat asthma.

What are the results of using glucocorticoid drugs to treat asthma? (Select all that apply.) a. Reduced bronchial hyperreactivity b. Reduced edema of the airway c. Reduced number of bronchial beta2 receptors d. Increased responsiveness to beta2-adrenergic agonists e. Increased synthesis of inflammatory mediators

ANS: A, B, D Glucocorticoids act by reducing bronchial hyperreactivity and airway edema and increasing airway responsiveness to beta2-adrenergic agonists. They do not reduce the number of beta2-adrenergic agonist receptors. They help reduce the synthesis of inflammatory mediators.

Besides having diuretic effects for patients with congestive heart failure, thiazides are also used to treat what? (Select all that apply.) a. Diabetes insipidus b. Hepatic failure c. Increased intracranial pressure d. Intraocular pressure e. Postmenopausal osteoporosis

ANS: A, B, E Thiazide diuretics have the paradoxical effect of reducing urine output in patients with diabetes insipidus. They can also be used to mobilize edema associated with liver disease. They promote tubular reabsorption of calcium, which may reduce the risk of osteoporosis in postmenopausal women. Mannitol is used to treat edema that causes increased intracranial pressure and intraocular pressure.

Patients with a history of myocardial infarction should take which medications indefinitely? (Select all that apply.) a. ACE inhibitors b. Alteplase c. Aspirin d. Beta blockers e. Clopidogrel

ANS: A, C, D Patients who have had an MI should take ACE inhibitors, ASA, and beta blockers indefinitely to prevent recurrence and to minimize continuing cardiac remodeling. Alteplase is given during acute management, and clopidogrel is used during acute management and as an adjunct to reperfusion therapy.

Which medications are included in first-line therapy for heart failure? (Select all that apply.) a. Agents that inhibit the renin-angiotensin-aldosterone system (RAAS) b. Aldosterone antagonists c. Beta blockers d. Cardiac glycosides e. Diuretics

ANS: A, C, E For routine therapy, heart failure is treated with agents that inhibit the RAAS, beta blockers, and diuretics. Aldosterone antagonists and cardiac glycosides are not first-line drugs for HF.

15. When caring for a patient who is hospitalized with active tuberculosis (TB), the nurse observes a student nurse who is assigned to take care of a patient. Which action, if performed by the student nurse, would require an intervention by the nurse? a. The patient is offered a tissue from the box at the bedside. b. A surgical face mask is applied before visiting the patient. c. A snack is brought to the patient from the unit refrigerator. d. Hand washing is performed before entering the patient's room.

ANS: B A high-efficiency particulate-absorbing (HEPA) mask, rather than a standard surgical mask, should be used when entering the patient's room because the HEPA mask can filter out 100% of small airborne particles. Hand washing before entering the patient's room is appropriate. Because anorexia and weight loss are frequent problems in patients with TB, bringing food to the patient is appropriate. The student nurse should perform hand washing after handling a tissue that the patient has used, but no precautions are necessary when giving the patient an unused tissue.

9. The nurse teaches a patient about the transmission of pulmonary tuberculosis (TB). Which statement, if made by the patient, indicates that teaching was effective? a. "I will avoid being outdoors whenever possible." b. "My husband will be sleeping in the guest bedroom." c. "I will take the bus instead of driving to visit my friends." d. "I will keep the windows closed at home to contain the germs."

ANS: B Teach the patient how to minimize exposure to close contacts and household members. Homes should be well ventilated, especially the areas where the infected person spends a lot of time. While still infectious, the patient should sleep alone, spend as much time as possible outdoors, and minimize time in congregate settings or on public transportation.

13. After 2 months of tuberculosis (TB) treatment with isoniazid (INH), rifampin (Rifadin), pyrazinamide (PZA), and ethambutol, a patient continues to have positive sputum smears for acid-fast bacilli (AFB). Which action should the nurse take next? a. Teach about treatment for drug-resistant TB treatment. b. Ask the patient whether medications have been taken as directed. c. Schedule the patient for directly observed therapy three times weekly. d. Discuss with the health care provider the need for the patient to use an injectable antibiotic.

ANS: B The first action should be to determine whether the patient has been compliant with drug therapy because negative sputum smears would be expected if the TB bacillus is susceptible to the medications and if the medications have been taken correctly. Assessment is the first step in the nursing process. Depending on whether the patient has been compliant or not, different medications or directly observed therapy may be indicated. The other options are interventions based on assumptions until an assessment has been completed.

A patient is admitted to the coronary care unit from the emergency department after initial management of STEMI. A primary percutaneous coronary intervention has been performed. The nurse notes an initial heart rate of 56 beats per minute and a blood pressure of 120/80 mm Hg. The patient has a history of stroke and a previous myocardial infarction. Which order will the nurse question? a. Aspirin b. Beta blocker c. Clopidogrel d. Heparin

ANS: B A beta blocker would be contraindicated in this patient, because it slows the heart, and this patient is already bradycardic. Aspirin, clopidogrel, and heparin are recommended in patients who have had a primary PCI.

42. The nurse receives a change-of-shift report on the following patients with chronic obstructive pulmonary disease (COPD). Which patient should the nurse assess first? a. A patient with loud expiratory wheezes b. A patient with a respiratory rate of 38/minute c. A patient who has a cough productive of thick, green mucus d. A patient with jugular venous distention and peripheral edema

ANS: B A respiratory rate of 38/minute indicates severe respiratory distress, and the patient needs immediate assessment and intervention to prevent possible respiratory arrest. The other patients also need assessment as soon as possible, but they do not need to be assessed as urgently as the tachypneic patient. DIF: Cognitive Level: Analyze (analysis) REF: 576 OBJ: Special Questions: Prioritization; Multiple Patients TOP: Nursing Process: Assessment MSC: NCLEX: Safe and Effective Care Environment

Intravenous potassium chloride (KCl) 60 mEq is prescribed for treatment of a patient with severe hypokalemia. Which action should the nurse take? a. Administer the KCl as a rapid IV bolus. b. Infuse the KCl at a rate of 20 mEq/hour. c. Give the KCl only through a central venous line. d. Add no more than 40 mEq/L to a liter of IV fluid.

ANS: B ANS: B Infuse the KCl at a rate of 20 mEq/hour. Intravenous KCl is administered at a maximal rate of 20 mEq/hr. Rapid IV infusion of KCl can cause cardiac arrest. Although the preferred concentration for KCl is no more than 40 mEq/L, concentrations up to 80 mEq/L may be used for some patients. KCl can cause inflammation of peripheral veins, but it can be administered by this route.

27. A patient with chronic obstructive pulmonary disease (COPD) has rhonchi throughout the lung fields and a chronic, nonproductive cough. Which nursing intervention will be most effective? a. Change the oxygen flow rate to the highest prescribed rate. b. Teach the patient to use the Flutter airway clearance device. c. Reinforce the ongoing use of pursed lip breathing techniques. d. Teach the patient about consistent use of inhaled corticosteroids.

ANS: B Airway clearance devices assist with moving mucus into larger airways where it can more easily be expectorated. The other actions may be appropriate for some patients with COPD, but they are not indicated for this patient's problem of thick mucus secretions. DIF: Cognitive Level: Apply (application) REF: 595 TOP: Nursing Process: Implementation MSC: NCLEX: Physiological Integrity

A patient has begun taking an HMG-COA reductase inhibitor. Which statement about this class of drugs made by the nurse during patient education would be inappropriate? a. "Statins reduce the risk of morbidity from influenza." b. "You should come into the clinic for liver enzymes in 1 month." c. "Statins reduce the risk of coronary events in people with normal LDL levels." d. "You should maintain a healthy lifestyle and avoid high-fat foods."

ANS: B Baseline liver enzyme tests should be done before a patient starts taking an HMG-COA reductase inhibitor. They should be measured again in 6 to 12 months unless the patient has poor liver function, in which case the tests are indicated every 3 months. A recent study demonstrated protection against influenza morbidity in patients because of a decrease in proinflammatory cytokine release. Statins do reduce the risk of stroke and coronary events in people with normal LDL levels. Maintaining a healthy lifestyle is important, as is avoiding high-fat foods.

33. When admitting a patient with a non-ST-segment-elevation myocardial infarction (NSTEMI) to the intensive care unit, which action should the nurse perform first? a. Obtain the blood pressure. b. Attach the cardiac monitor. c. Assess the peripheral pulses. d. Auscultate the breath sounds.

ANS: B Because dysrhythmias are the most common complication of myocardial infarction (MI), the first action should be to place the patient on a cardiac monitor. The other actions also are important and should be accomplished as quickly as possible.

39. To improve the physical activity level for a mildly obese 71-year-old patient, which action should the nurse plan to take? a. Stress that weight loss is a major benefit of increased exercise. b. Determine what kind of physical activities the patient usually enjoys. c. Tell the patient that older adults should exercise for no more than 20 minutes at a time. d. Teach the patient to include a short warm-up period at the beginning of physical activity.

ANS: B Because patients are more likely to continue physical activities that they already enjoy, the nurse will plan to ask the patient about preferred activities. The goal for older adults is 30 minutes of moderate activity on most days. Older adults should plan for a longer warm-up period. Benefits of exercises, such as improved activity tolerance, should be emphasized rather than aiming for significant weight loss in older mildly obese adults.

3. Which assessment data collected by the nurse who is admitting a patient with chest pain suggest that the pain is caused by an acute myocardial infarction (AMI)? a. The pain increases with deep breathing. b. The pain has lasted longer than 30 minutes. c. The pain is relieved after the patient takes nitroglycerin. d. The pain is reproducible when the patient raises the arms.

ANS: B Chest pain that lasts for 20 minutes or more is characteristic of AMI. Changes in pain that occur with raising the arms or with deep breathing are more typical of musculoskeletal pain or pericarditis. Stable angina is usually relieved when the patient takes nitroglycerin.

12. A patient asks a nurse why he cannot use digoxin [Lanoxin] for his heart failure, because both of his parents used it for HF. The nurse will explain that digoxin is not the first-line therapy for which reason? a. It causes tachycardia and increases the cardiac workload. b. It does not correct the underlying pathology of heart failure. c. It has a wide therapeutic range that makes dosing difficult. d. It may actually shorten the patient's life expectancy.

ANS: B Digoxin improves cardiac output, alters electrical effects, and helps to decrease sympathetic outflow from the central nervous system (CNS) through its neurohormonal effects; however, it does not alter the underlying pathology of heart failure or prevent cardiac remodeling. Digoxin causes bradycardia and increases the cardiac workload by increasing contractility. It has a narrow therapeutic range and many adverse effects. Digoxin does not improve life expectancy; in women it may actually shorten life expectancy.

11. A nurse and a nursing student are reviewing the care of a 30-kg patient who will receive intravenous aminophylline. Which statement by the student indicates an understanding of the administration of this medication? a. "After the loading dose has been given, the patient will receive 6 mg/kg/hr." b. "Dosing is titrated based on the serum theophylline levels." c. "If the patient's serum theophylline level is less than 15 mcg/mL, the rate should be reduced." d. "The patient will receive a loading dose of 180 mg over 5 minutes."

ANS: B Dosing for aminophylline is based on each patient's serum theophylline levels. The loading dose usually is 6 mg/kg; after that, the maintenance infusion is titrated according to the theophylline levels. A serum theophylline level of 15 mcg/mL is within the therapeutic range, so dosing would not need to change. The patient's total loading dose will be 180 mg, but infusions should never be given at a rate faster than 25 mg/min.

10. A patient who takes oral theophylline [Theochron] twice daily for chronic stable asthma develops an infection and will take ciprofloxacin. The nurse will contact the provider to discuss: a. changing to a different antibiotic. b. reducing the theophylline dose. c. giving theophylline once daily. d. switching from theophylline to a LABA.

ANS: B Fluoroquinolone antibiotics increase theophylline levels, so the dose of theophylline may need to be reduced to prevent theophylline toxicity. Changing antibiotics, giving the theophylline once daily, and changing to a LABA are not indicated.

14. Which finding by the nurse for a patient with a nursing diagnosis of impaired gas exchange will be most useful in evaluating the effectiveness of treatment? a. Even, unlabored respirations b. Pulse oximetry reading of 92% c. Respiratory rate of 18 breaths/minute d. Absence of wheezes, rhonchi, or crackles

ANS: B For the nursing diagnosis of impaired gas exchange, the best data for evaluation are arterial blood gases (ABGs) or pulse oximetry. The other data may indicate either improvement or impending respiratory failure caused by fatigue. DIF: Cognitive Level: Apply (application) REF: 598 TOP: Nursing Process: Evaluation MSC: NCLEX: Physiological Integrity

28. The nurse provides dietary teaching for a patient with chronic obstructive pulmonary disease (COPD) who has a low body mass index (BMI). Which patient statement indicates that the teaching has been effective? a. "I will drink lots of fluids with my meals." b. "I can have ice cream as a snack every day." c. "I will exercise for 15 minutes before meals." d. "I will decrease my intake of meat and poultry."

ANS: B High-calorie foods like ice cream are an appropriate snack for patients with COPD. Fluid intake of 3 L/day is recommended, but fluids should be taken between meals rather than with meals to improve oral intake of solid foods. The patient should avoid exercise for an hour before meals to prevent fatigue while eating. Meat and dairy products are high in protein and are good choices for the patient with COPD. DIF: Cognitive Level: Apply (application) REF: 595-596 TOP: Nursing Process: Evaluation MSC: NCLEX: Physiological Integrity

6. A young adult woman will begin using an inhaled glucocorticoid to treat asthma. The nurse will teach this patient about the importance of which action? a. Lowering her calcium intake and increasing her vitamin D intake b. Participating in weight-bearing exercises on a regular basis c. Taking oral glucocorticoids during times of acute stress d. Using two reliable forms of birth control to prevent pregnancy

ANS: B Like oral glucocorticoids, inhaled glucocorticoids can promote bone loss in premenopausal women. Patients should be encouraged to participate in weight-bearing exercises to help minimize this side effect. Patients should increase both their calcium and vitamin D intakes. Patients taking oral glucocorticoids need increased steroids in times of stress. It is not necessary to use two reliable forms of birth control.

10. The nurse will suspect that the patient with stable angina is experiencing a side effect of the prescribed metoprolol (Lopressor) if the a. patient is restless and agitated. b. blood pressure is 90/54 mm Hg. c. patient complains about feeling anxious. d. cardiac monitor shows a heart rate of 61 beats/minute.

ANS: B Patients taking β-adrenergic blockers should be monitored for hypotension and bradycardia. Because this class of medication inhibits the sympathetic nervous system, restlessness, agitation, hypertension, and anxiety will not be side effects.

7. After the nurse teaches the patient about the use of carvedilol (Coreg) in preventing anginal episodes, which statement by a patient indicates that the teaching has been effective? a. "Carvedilol will help my heart muscle work harder." b. "It is important not to suddenly stop taking the carvedilol." c. "I can expect to feel short of breath when taking carvedilol." d. "Carvedilol will increase the blood flow to my heart muscle."

ANS: B Patients who have been taking β-adrenergic blockers can develop intense and frequent angina if the medication is suddenly discontinued. Carvedilol (Coreg) decreases myocardial contractility. Shortness of breath that occurs when taking β-adrenergic blockers for angina may be due to bronchospasm and should be reported to the health care provider. Carvedilol works by decreasing myocardial oxygen demand, not by increasing blood flow to the coronary arteries.

A patient who recently started therapy with an HMG-COA reductase inhibitor asks the nurse, "How long will it take until I see an effect on my LDL cholesterol?" The nurse gives which correct answer? a. "It will take 6 months to see a change." b. "A reduction usually is seen within 2 weeks." c. "Blood levels normalize immediately after the drug is started." d. "Cholesterol will not be affected, but triglycerides will fall within the first week."

ANS: B Reductions in LDL cholesterol are significant within 2 weeks and maximal within 4 to 6 weeks. It does not take 6 months to see a change. The blood level of LDL cholesterol is not reduced immediately upon starting the drug; a reduction is seen within 2 weeks. Blood cholesterol is affected, specifically LDL cholesterol, not triglycerides.

23. A patient who is recovering from an acute myocardial infarction (AMI) asks the nurse about when sexual intercourse can be resumed. Which response by the nurse is best? a. "Most patients are able to enjoy intercourse without any complications." b. "Sexual activity uses about as much energy as climbing two flights of stairs." c. "The doctor will provide sexual guidelines when your heart is strong enough." d. "Holding and cuddling are good ways to maintain intimacy after a heart attack."

ANS: B Sexual activity places about as much physical stress on the cardiovascular system as most moderate-energy activities such as climbing two flights of stairs. The other responses do not directly address the patient's question or may not be accurate for this patient.

9. A patient with chronic congestive heart failure has repeated hospitalizations in spite of ongoing treatment with hydrochlorothiazide [HydroDIURIL] and digoxin. The prescriber has ordered spironolactone [Aldactone] to be added to this patient's drug regimen, and the nurse provides education about this medication. Which statement by the patient indicates understanding of the teaching? a. "I can expect improvement within a few hours after taking this drug." b. "I need to stop taking potassium supplements." c. "I should use salt substitutes to prevent toxic side effects." d. "I should watch closely for dehydration."

ANS: B Spironolactone is a potassium-sparing diuretic used to counter the potassium-wasting effects of hydrochlorothiazides. Patients taking potassium supplements are at risk for hyperkalemia when taking this medication, so they should be advised to stop the supplements. Spironolactone takes up to 48 hours to have effects. Salt substitutes contain high levels of potassium and are contraindicated. Spironolactone is a weak diuretic, so the risk of dehydration is not increased.

16. The nurse is admitting a patient diagnosed with an acute exacerbation of chronic obstructive pulmonary disease (COPD).What is the best way for the nurse to determine the appropriate oxygen flow rate? a. Minimize oxygen use to avoid oxygen dependency. b. Maintain the pulse oximetry level at 90% or greater. c. Administer oxygen according to the patient's level of dyspnea. d. Avoid administration of oxygen at a rate of more than 2 L/minute.

ANS: B The best way to determine the appropriate oxygen flow rate is by monitoring the patient's oxygenation either by arterial blood gases (ABGs) or pulse oximetry. An oxygen saturation of 90% indicates adequate blood oxygen level without the danger of suppressing the respiratory drive. For patients with an exacerbation of COPD, an oxygen flow rate of 2 L/min may not be adequate. Because oxygen use improves survival rate in patients with COPD, there is no concern about oxygen dependency. The patient's perceived dyspnea level may be affected by other factors (such as anxiety) besides blood oxygen level. DIF: Cognitive Level: Apply (application) REF: 589 TOP: Nursing Process: Implementation MSC: NCLEX: Physiological Integrity

5. The emergency department nurse is evaluating the effectiveness of therapy for a patient who has received treatment during an asthma attack. Which assessment finding is the best indicator that the therapy has been effective? a. No wheezes are audible. b. Oxygen saturation is >90%. c. Accessory muscle use has decreased. d. Respiratory rate is 16 breaths/minute.

ANS: B The goal for treatment of an asthma attack is to keep the oxygen saturation >90%. The other patient data may occur when the patient is too fatigued to continue with the increased work of breathing required in an asthma attack. DIF: Cognitive Level: Apply (application) REF: 569 TOP: Nursing Process: Evaluation MSC: NCLEX: Physiological Integrity

13. When titrating IV nitroglycerin (Tridil) for a patient with a myocardial infarction (MI), which action will the nurse take to evaluate the effectiveness of the medication? a. Monitor heart rate. b. Ask about chest pain. c. Check blood pressure. d. Observe for dysrhythmias.

ANS: B The goal of IV nitroglycerin administration in MI is relief of chest pain by improving the balance between myocardial oxygen supply and demand. The nurse also will monitor heart rate and blood pressure (BP) and observe for dysrhythmias, but these parameters will not indicate whether the medication is effective.

40. Which patient at the cardiovascular clinic requires the most immediate action by the nurse? a. Patient with type 2 diabetes whose current blood glucose level is 145 mg/dL b. Patient with stable angina whose chest pain has recently increased in frequency c. Patient with familial hypercholesterolemia and a total cholesterol of 465 mg/dL d. Patient with chronic hypertension whose blood pressure today is 172/98 mm Hg

ANS: B The history of more frequent chest pain suggests that the patient may have unstable angina, which is part of the acute coronary syndrome spectrum. This will require rapid implementation of actions such as cardiac catheterization and possible percutaneous coronary intervention. The data about the other patients suggest that their conditions are stable.

13. The nurse teaches a patient about pursed lip breathing. Which action by the patient would indicate to the nurse that further teaching is needed? a. The patient inhales slowly through the nose. b. The patient puffs up the cheeks while exhaling. c. The patient practices by blowing through a straw. d. The patient's ratio of inhalation to exhalation is 1:3.

ANS: B The patient should relax the facial muscles without puffing the cheeks while doing pursed lip breathing. The other actions by the patient indicate a good understanding of pursed lip breathing. DIF: Cognitive Level: Apply (application) REF: 579 TOP: Nursing Process: Evaluation MSC: NCLEX: Physiological Integrity

40. The clinic nurse makes a follow-up telephone call to a patient with asthma. The patient reports having a baseline peak flow reading of 600 L/minute and the current peak flow is 420 L/minute. Which action should the nurse take first? a. Tell the patient to go to the hospital emergency department. b. Instruct the patient to use the prescribed albuterol (Proventil). c. Ask about recent exposure to any new allergens or asthma triggers. d. Question the patient about use of the prescribed inhaled corticosteroids.

ANS: B The patient's peak flow is 70% of normal, indicating a need for immediate use of short-acting β2-adrenergic SABA medications. Assessing for correct use of medications or exposure to allergens also is appropriate, but would not address the current decrease in peak flow. Because the patient is currently in the yellow zone, hospitalization is not needed. DIF: Cognitive Level: Analyze (analysis) REF: 580 OBJ: Special Questions: Prioritization TOP: Nursing Process: Implementation MSC: NCLEX: Physiological Integrity

13. A nurse is preparing to administer digoxin [Lanoxin] to a patient. The patient's heart rate is 62 beats per minute, and the blood pressure is 120/60 mm Hg. The last serum electrolyte value showed a potassium level of 5.2 mEq/L. What will the nurse do? a. Contact the provider to request an increased dose of digoxin. b. Give the dose of digoxin and notify the provider of the potassium level. c. Request an order for a diuretic. d. Withhold the dose and notify the provider of the heart rate.

ANS: B The patient's serum potassium level is above normal limits, but only slightly. An elevated potassium level can reduce the effects of digoxin, so there is no risk of toxicity. There is no indication that an increased dose of digoxin is needed. There is no indication for a diuretic. The heart rate is acceptable; doses should be withheld if the heart rate is less than 60 beats per minute.

29. Which instruction should the nurse include in an exercise teaching plan for a patient with chronic obstructive pulmonary disease (COPD)? a. "Stop exercising if you start to feel short of breath." b. "Use the bronchodilator before you start to exercise." c. "Breathe in and out through the mouth while you exercise." d. "Upper body exercise should be avoided to prevent dyspnea."

ANS: B Use of a bronchodilator before exercise improves airflow for some patients and is recommended. Shortness of breath is normal with exercise and not a reason to stop. Patients should be taught to breathe in through the nose and out through the mouth (using a pursed lip technique). Upper-body exercise can improve the mechanics of breathing in patients with COPD. DIF: Cognitive Level: Apply (application) REF: 572 TOP: Nursing Process: Implementation MSC: NCLEX: Physiological Integrity

A client admitted to the facility for treatment for tuberculosis receives instructions about the disease. Which statement made by the client indicates the need for further instruction? a) "I'll stay in isolation for 6 weeks." b) "This disease may come back later if I am under stress." c) "I'll have to take the medication for up to a year." d) "I'll always have a positive test for tuberculosis."

a) "I'll stay in isolation for 6 weeks." Explanation: The client requires additional teaching if he states that he'll be in isolation for 6 weeks. The client needs to be in isolation for 2 weeks, not 6, while taking the tuberculosis drugs. After 2 weeks of antitubercular therapy, the client is no longer considered contagious. The client needs to receive the drugs for 9 months to a year. He'll be positive when tested and if he's sick or under some stress he could have a relapse of the disease. pg.587

Approximately what percentage of people who are initially infected with TB develop active disease? a) 10% b) 40% c) 20% d) 30%

a) 10% Explanation: Approximately 10% of people who are initially infected develop active disease. The other percentages are inaccurate. pg.587

A patient who has been hospitalized for 2 days has been receiving normal saline IV at 100 ml/hr, has a nasogastric tube to low suction, and is NPO. Which assessment finding by the nurse is the priority to report to the health care provider? a. Serum sodium level of 138 mEq/L (138 mmol/L) b. Gradually decreasing level of consciousness (LOC) c. Oral temperature of 100.1° F with bibasilar lung crackles d. Weight gain of 2 pounds (1 kg) above the admission weight

ANS: B Gradually decreasing level of consciousness (LOC) The patient's history and change in LOC could be indicative of several fluid and electrolyte disturbances: extracellular fluid (ECF) excess, ECF deficit, hyponatremia, hypernatremia, hypokalemia, or metabolic alkalosis. Further diagnostic information will be ordered by the health care provider to determine the cause of the change in LOC and the appropriate interventions. The weight gain, elevated temperature, crackles, and serum sodium level also will be reported, but do not indicate a need for rapid action to avoid complications.

A patient with advanced lung cancer is admitted to the emergency department with urinary retention caused by renal calculi. Which of these laboratory values will require the most immediate action by the nurse? a. Arterial blood pH is 7.32. b. Serum calcium is 18 mEq/L. c. Serum potassium is 5.1 mEq/L. d. Arterial oxygen saturation is 91%.

ANS: B Serum calcium is 18 mEq/L. The serum calcium is well above the normal level and puts the patient at risk for cardiac dysrhythmias. The nurse should initiate cardiac monitoring and notify the health care provider. The potassium, oxygen saturation, and pH also are abnormal, and the nurse should notify the health care provider about these values as well, but they are not immediately life-threatening.

A patient asks the nurse how long he will have to wait after taking nitroglycerin before experiencing pain relief. What is the best answer by the nurse? a) 3 minutes b) 15 minutes c) 60 minutes d) 30 minutes

a) 3 minutes Nitroglycerin may be given by several routes: sublingual tablet or spray, oral capsule, topical agent, and intravenous (IV) administration. Sublingual nitroglycerin is generally placed under the tongue or in the cheek (buccal pouch) and ideally alleviates the pain of ischemia within 3 minutes.

The following data are obtained by the nurse when assessing a pregnant patient with eclampsia who is receiving IV magnesium sulfate. Which finding is most important to report to the health care provider immediately? a. The bibasilar breath sounds are decreased. b. The patellar and triceps reflexes are absent. c. The patient has been sleeping most of the day. d. The patient reports feeling "sick to my stomach."

ANS: B The patellar and triceps reflexes are absent. The loss of the deep tendon reflexes indicates that the patient's magnesium level may be reaching toxic levels. Nausea and lethargy also are side effects associated with magnesium elevation and should be reported, but they are not as significant as the loss of deep tendon reflexes. The decreased breath sounds suggest that the patient needs to cough and deep breathe to prevent atelectasis.

Following a thyroidectomy, a patient complains of "a tingling feeling around my mouth." The nurse will immediately check for a. an elevated serum potassium level. b. the presence of Chvostek's sign. c. a decreased thyroid hormone level. d. bleeding on the patient's dressing.

ANS: B the presence of Chvostek's sign. The patient's symptoms indicate possible hypocalcemia, which can occur secondary to parathyroid injury/removal during thyroidectomy. There is no indication of a need to check the potassium level, the thyroid hormone level, or for bleeding.

43. A patient has acute bronchitis with a nonproductive cough and wheezes. Which topic should the nurse plan to include in the teaching plan? a. Purpose of antibiotic therapy b. Ways to limit oral fluid intake c. Appropriate use of cough suppressants d. Safety concerns with home oxygen therapy

ANS: C Cough suppressants are frequently prescribed for acute bronchitis. Because most acute bronchitis is viral in origin, antibiotics are not prescribed unless there are systemic symptoms. Fluid intake is encouraged. Home oxygen is not prescribed for acute bronchitis, although it may be used for chronic bronchitis.

16. An occupational health nurse works at a manufacturing plant where there is potential exposure to inhaled dust. Which action, if recommended by the nurse, will be most helpful in reducing the incidence of lung disease? a. Treat workers with pulmonary fibrosis. b. Teach about symptoms of lung disease. c. Require the use of protective equipment. d. Monitor workers for coughing and wheezing.

ANS: C Prevention of lung disease requires the use of appropriate protective equipment such as masks. The other actions will help in recognition or early treatment of lung disease but will not be effective in prevention of lung damage. Repeated exposure eventually results in diffuse pulmonary fibrosis. Fibrosis is the result of tissue repair after inflammation.

A nurse is teaching about risk factors that increase the probability of heart disease to a community group. Which of the following risk factors will the nurse include? Choose all that apply. a) African-American descent b) Family history of coronary heart disease c) Elevated C-reactive protein d) Body mass index (BMI) of 23 e) Age greater than 45 years for men

a) African-American descent b) Family history of coronary heart disease c) Elevated C-reactive protein e) Age greater than 45 years for men Risk factors for coronary heart disease (CHD) include family history of CHD, age older than 45 years for men and 65 years for women, African-American race, BMI of 25 or greater, and elevated C-reactive protein.

7. The health care provider writes an order for bacteriologic testing for a patient who has a positive tuberculosis skin test. Which action should the nurse take? a. Teach about the reason for the blood tests. b. Schedule an appointment for a chest x-ray. c. Teach about the need to get sputum specimens for 2 to 3 consecutive days. d. Instruct the patient to expectorate three specimens as soon as possible.

ANS: C Sputum specimens are obtained on 2 to 3 consecutive days for bacteriologic testing for M. tuberculosis. The patient should not provide all the specimens at once. Blood cultures are not used for tuberculosis testing. A chest x-ray is not bacteriologic testing. Although the findings on chest x-ray examination are important, it is not possible to make a diagnosis of TB solely based on chest x-ray findings because other diseases can mimic the appearance of TB.

14. Employee health test results reveal a tuberculosis (TB) skin test of 16-mm induration and a negative chest x-ray for a staff nurse working on the pulmonary unit. The nurse has no symptoms of TB. Which information should the occupational health nurse plan to teach the staff nurse? a. Standard four-drug therapy for TB b. Need for annual repeat TB skin testing c. Use and side effects of isoniazid (INH) d. Bacille Calmette-Guérin (BCG) vaccine

ANS: C The nurse is considered to have a latent TB infection and should be treated with INH daily for 6 to 9 months. The four-drug therapy would be appropriate if the nurse had active TB. TB skin testing is not done for individuals who have already had a positive skin test. BCG vaccine is not used in the United States for TB and would not be helpful for this individual, who already has a TB infection.

15. Following an acute myocardial infarction (AMI), a patient ambulates in the hospital hallway. When the nurse is evaluating the patient's response to the activity, which assessment data would indicate that the exercise level should be decreased? a. Blood pressure (BP) changes from 118/60 to 126/68 mm Hg. b. Oxygen saturation drops from 99% to 95%. c. Heart rate increases from 66 to 92 beats/minute. d. Respiratory rate goes from 14 to 20 breaths/minute.

ANS: C A change in heart rate of more than 20 beats over the resting heart rate indicates that the patient should stop and rest. The increases in BP and respiratory rate, and the slight decrease in oxygen saturation, are normal responses to exercise.

12. The nurse interviews a patient with a new diagnosis of chronic obstructive pulmonary disease (COPD). Which information is most helpful in confirming a diagnosis of chronic bronchitis? a. The patient tells the nurse about a family history of bronchitis. b. The patient's history indicates a 30 pack-year cigarette history. c. The patient complains about a productive cough every winter for 3 months. d. The patient denies having any respiratory problems until the last 12 months.

ANS: C A diagnosis of chronic bronchitis is based on a history of having a productive cough for 3 months for at least 2 consecutive years. There is no family tendency for chronic bronchitis. Although smoking is the major risk factor for chronic bronchitis, a smoking history does not confirm the diagnosis. DIF: Cognitive Level: Apply (application) REF: 579 TOP: Nursing Process: Assessment MSC: NCLEX: Physiological Integrity

32. A patient admitted to the coronary care unit (CCU) with an ST-segment-elevation myocardial infarction (STEMI) is restless and anxious. The blood pressure is 86/40 and heart rate is 123. Based on this information, which nursing diagnosis is a priority for the patient? a. Acute pain related to myocardial infarction b. Anxiety related to perceived threat of death c. Stress overload related to acute change in health d. Decreased cardiac output related to cardiogenic shock

ANS: C All the nursing diagnoses may be appropriate for this patient, but the hypotension and tachycardia indicate decreased cardiac output and shock from the damaged myocardium. This will result in decreased perfusion to all vital organs (e.g., brain, kidney, heart) and is a priority.

9. The nurse is caring for a patient with chronic obstructive pulmonary disease (COPD). Which information obtained from the patient would prompt the nurse to consult with the health care provider before administering the prescribed theophylline? a. The patient reports a recent 15-pound weight gain. b. The patient denies any shortness of breath at present. c. The patient takes cimetidine (Tagamet) 150 mg daily. d. The patient complains about coughing up green mucus.

ANS: C Cimetidine interferes with the metabolism of theophylline, and concomitant administration may lead rapidly to theophylline toxicity. The other patient information would not affect whether the theophylline should be administered or not. DIF: Cognitive Level: Apply (application) REF: 571 | 572 TOP: Nursing Process: Assessment MSC: NCLEX: Physiological Integrity

11. A patient with chronic obstructive pulmonary disease (COPD) has a nursing diagnosis of imbalanced nutrition: less than body requirements. Which intervention would be most appropriate for the nurse to include in the plan of care? a. Encourage increased intake of whole grains. b. Increase the patient's intake of fruits and fruit juices. c. Offer high-calorie snacks between meals and at bedtime. d. Assist the patient in choosing foods with high vegetable and mineral content.

ANS: C Eating small amounts more frequently (as occurs with snacking) will increase caloric intake by decreasing the fatigue and feelings of fullness associated with large meals. Patients with COPD should rest before meals. Foods that have a lot of texture like whole grains may take more energy to eat and get absorbed and lead to decreased intake. Although fruits, juices, and vegetables are not contraindicated, foods high in protein are a better choice. DIF: Cognitive Level: Apply (application) REF: 596 TOP: Nursing Process: Planning MSC: NCLEX: Physiological Integrity

32. A patient newly diagnosed with asthma is being discharged. The nurse anticipates including which topic in the discharge teaching? a. Use of long-acting β-adrenergic medications b. Side effects of sustained-release theophylline c. Self-administration of inhaled corticosteroids d. Complications associated with oxygen therapy

ANS: C Inhaled corticosteroids are more effective in improving asthma than any other drug and are indicated for all patients with persistent asthma. The other therapies would not typically be first-line treatments for newly diagnosed asthma. DIF: Cognitive Level: Apply (application) REF: 569 TOP: Nursing Process: Implementation MSC: NCLEX: Physiological Integrity

1. A patient is taking enalapril [Vasotec]. The nurse understands that patients taking this type of drug for heart failure need to be monitored carefully for: a. hypernatremia. b. hypertension. c. hyperkalemia. d. hypokalemia.

ANS: C One of the principal effects of angiotensin-converting enzyme (ACE) inhibitors is hyperkalemia, which is due to decreased aldosterone release arising from blockage of angiotensin II. There is no indication that careful monitoring of sodium for increased levels is indicated. Vasotec is indicated for heart failure, not hypertension. The drug therapy should be monitored to ascertain its effectiveness, but hyperkalemia is the main concern. Hyperkalemia, not hypokalemia, is a concern because of the decreased aldosterone release that occurs with blockage of angiotensin II.

A 65-year-old male client complains of pain and cramping in his thigh when climbing the stairs and numbness in his legs after exertion. The nurse anticipates the physician will perform which of the following diagnostic tests right in the office to determine PAD? a) Ankle-brachial index b) Exercise electrocardiography c) Photoplethysmography d) Electron beam computed tomography

a) Ankle-brachial index The client's symptoms indicate he may have peripheral artery disease (PAD). The ankle-brachial index is a simple, noninvasive test used for its diagnosis. An exercise electrocardiography may be ordered for a client with possible CAD. An EBCT is a radiologic test that produces x-rays of the coronary arteries using an electron beam. It is used to diagnose for CAD. Clients with suspected venous insufficiency will undergo photoplethysmography, a diagnostic test that measures light that is not absorbed by hemoglobin and consequently is reflected back to the machine.

Which of the following actions is most appropriate for the nurse to take when the patient demonstrates subcutaneous emphysema along the suture line or chest dressing 2 hours after chest surgery? a) Apply a compression dressing to the area. b) Record the observation. c) Measure the patient's pulse oximetry. d) Report the finding to the physician immediately.

a) Apply a compression dressing to the area. Explanation: Subcutaneous emphysema is a typical postoperative finding in the patient after chest surgery. During surgery the air within the pleural cavity is expelled through the tissue opening created by the surgical procedure. Subcutaneous emphysema is absorbed by the body spontaneously after the underlying leak is treated or halted. pg.616

A client with a history of chronic renal failure receives hemodialysis treatments three times per week through an arteriovenous (AV) fistula in the left arm. Which intervention should the nurse include in the care plan? a) Assess the AV fistula for a bruit and thrill. b) Keep the AV fistula site dry. c) Take the client's blood pressure in the left arm. d) Keep the AV fistula wrapped in gauze.

a) Assess the AV fistula for a bruit and thrill. The nurse needs to assess the AV fistula for a bruit and thrill because if these findings aren't present, the fistula isn't functioning. The AV fistula may get wet when the client isn't being dialyzed. Immediately after a dialysis treatment, the access site should be covered with adhesive bandages, not gauze. Blood pressure readings or venipunctures shouldn't be taken in the arm with the AV fistula.

A client is returning from the operating room after inguinal hernia repair. The nurse notes that he has fluid volume excess from the operation and is at risk for left-sided heart failure. Which sign or symptom indicates left-sided heart failure? a) Bibasilar crackles b) Right upper quadrant pain c) Dependent edema d) Jugular vein distention

a) Bibasilar crackles

4. A patient with asthma is admitted to an emergency department with a respiratory rate of 22 breaths per minute, a prolonged expiratory phase, tight wheezes, and an oxygen saturation of 90% on room air. The patient reports using fluticasone [Flovent HFA] 110 mcg twice daily and has used 2 puffs of albuterol [Proventil HFA], 90 mcg/puff, every 4 hours for 2 days. The nurse will expect to administer which drug? a. Four puffs of albuterol, oxygen, and intravenous theophylline b. Intramuscular glucocorticoids and salmeterol by metered-dose inhaler c. Intravenous glucocorticoids, nebulized albuterol and ipratropium, and oxygen d. Intravenous theophylline, oxygen, and fluticasone (Flovent HFA) 220 mcg

ANS: C Patients using inhaled glucocorticoids should be given IV or oral glucocorticoids for acute exacerbations. During asthma flares, nebulized albuterol with ipratropium may be better tolerated and more effective. Oxygen is indicated, because oxygen saturations are low despite the increased work of breathing. Increasing the dose of albuterol and giving theophylline are not indicated. Salmeterol is a long-term beta agonist and is not useful in an acute attack.

22. A patient with chronic obstructive pulmonary disease (COPD) has poor gas exchange. Which action by the nurse would be most appropriate? a. Have the patient rest in bed with the head elevated to 15 to 20 degrees. b. Ask the patient to rest in bed in a high-Fowler's position with the knees flexed. c. Encourage the patient to sit up at the bedside in a chair and lean slightly forward. d. Place the patient in the Trendelenburg position with several pillows behind the head.

ANS: C Patients with COPD improve the mechanics of breathing by sitting up in the "tripod" position. Resting in bed with the head elevated in a semi-Fowler's position would be an alternative position if the patient was confined to bed, but sitting in a chair allows better ventilation. The Trendelenburg position or sitting upright in bed with the knees flexed would decrease the patient's ability to ventilate well. DIF: Cognitive Level: Apply (application) REF: 599 TOP: Nursing Process: Implementation MSC: NCLEX: Physiological Integrity

A nurse is evaluating a patient admitted to the emergency department with an evolving STEMI for possible administration of thrombolytic therapy. Which information, identified during history taking, would contraindicate this type of therapy? a. The patient just completed her last menstrual cycle. b. The patient states that the chest pain started 1 hour ago. c. The patient has a history of a small cerebral aneurysm. d. The patient has hypertension that is well controlled by diuretic therapy.

ANS: C Patients with a history of CVA should not receive fibrinolytic therapy. This patient has had a known cerebral aneurysm. Active internal bleeding is a contraindication for thrombolysis except for menses, but the patient has indicated she has completed her last cycle. Fibrinolytic therapy should be administered for chest pain that has been present for no longer than 12 hours. Poorly controlled or severe hypertension is a relative contraindication. Thrombolytics can be administered with caution.

A nurse is discussing fibrinolytic therapy for the acute phase of STEMI management with a group of nursing students. Which statement by a student indicates understanding of this therapy? a. "Fibrinolytics are effective when the first dose is given up to 24 hours after symptom onset." b. "Fibrinolytics should be given once cardiac troponins reveal the presence of STEMI." c. "Fibrinolytics should be used with caution in patients with a history of cerebrovascular accident." d. "Patients should receive either an anticoagulant or an antiplatelet agent with a fibrinolytic drug."

ANS: C Patients with a history of cerebrovascular accident (CVA) should not receive fibrinolytic agents because of the increased risk of intracranial hemorrhage. Fibrinolytics are most effective when given within 30 minutes of arrival in the emergency department. Because cardiac troponins are not detectable until 2 to 4 hours after the onset of symptoms, fibrinolytics should be administered before these laboratory values are available. Patients receiving fibrinolytics should receive both an anticoagulant and an antiplatelet drug.

15. A patient is taking a thiazide diuretic for hypertension and quinidine to treat a dysrhythmia. The prescriber orders digoxin 0.125 mg to improve this patient's cardiac output. The nurse should contact the provider to request: a. adding spironolactone [Aldactone]. b. reducing the dose of digoxin. c. discontinuing the quinidine. d. giving potassium supplements.

ANS: C Quinidine can cause plasma levels of digoxin to rise; concurrent use of quinidine and digoxin is contraindicated. There is no indication for adding spironolactone unless this patient's potassium level is elevated. The dose of digoxin ordered is a low dose. Potassium supplements are contraindicated with digoxin.

3. A patient has 2+ pitting edema of the lower extremities bilaterally. Auscultation of the lungs reveals crackles bilaterally, and the serum potassium level is 6 mEq/L. Which diuretic agent ordered by the prescriber should the nurse question? a. Bumetanide [Bumex] b. Furosemide [Lasix] c. Spironolactone [Aldactone] d. Hydrochlorothiazide [HydroDIURIL]

ANS: C Spironolactone is a non-potassium-wasting diuretic; therefore, if the patient has a serum potassium level of 6 mEq/L, indicating hyperkalemia, an order for this drug should be questioned. Bumetanide, furosemide, and hydrochlorothiazide are potassium-wasting diuretics and would be appropriate to administer in a patient with hyperkalemia.

A patient who is taking simvastatin [Zocor] develops an infection and the provider orders azithromycin [Zithromax] to treat the infection. The nurse should be concerned if the patient complains of: a. nausea. b. tiredness. c. muscle pain. d. headache.

ANS: C Statins can injure muscle tissue, causing muscle aches and pain known as myopathy/rhabdomyolysis. Daptomycin also can cause myopathy and therefore should be used with caution in patients concurrently taking simvastatin. Nausea, tiredness, and headache would not cause the nurse as much concern as the likelihood of myopathy.

16. During the administration of the thrombolytic agent to a patient with an acute myocardial infarction (AMI), the nurse should stop the drug infusion if the patient experiences a. bleeding from the gums. b. increase in blood pressure. c. a decrease in level of consciousness. d. a nonsustained episode of ventricular tachycardia.

ANS: C The change in level of consciousness indicates that the patient may be experiencing intracranial bleeding, a possible complication of thrombolytic therapy. Some bleeding of the gums is an expected side effect of the therapy but not an indication to stop infusion of the thrombolytic medication. A decrease in blood pressure could indicate internal bleeding. A nonsustained episode of ventricular tachycardia is a common reperfusion dysrhythmia and may indicate that the therapy is effective.

35. The nurse obtains the following data when assessing a patient who experienced an ST-segment-elevation myocardial infarction (STEMI) 2 days previously. Which information is most important to report to the health care provider? a. The troponin level is elevated. b. The patient denies ever having a heart attack. c. Bilateral crackles are auscultated in the mid-lower lobes. d. The patient has occasional premature atrial contractions (PACs).

ANS: C The crackles indicate that the patient may be developing heart failure, a possible complication of myocardial infarction (MI). The health care provider may need to order medications such as diuretics or angiotensin-converting enzyme (ACE) inhibitors for the patient. Elevation in troponin level at this time is expected. PACs are not life-threatening dysrhythmias. Denial is a common response in the immediate period after the MI.

5. After the nurse has finished teaching a patient about the use of sublingual nitroglycerin (Nitrostat), which patient statement indicates that the teaching has been effective? a. "I can expect some nausea as a side effect of nitroglycerin." b. "I should only take the nitroglycerin if I start to have chest pain." c. "I will call an ambulance if I still have pain after taking 3 nitroglycerin 5 minutes apart." d. "Nitroglycerin helps prevent a clot from forming and blocking blood flow to my heart."

ANS: C The emergency medical services (EMS) system should be activated when chest pain or other symptoms are not completely relieved after 3 sublingual nitroglycerin tablets taken 5 minutes apart. Nitroglycerin can be taken to prevent chest pain or other symptoms from developing (e.g., before intercourse). Gastric upset (e.g., nausea) is not an expected side effect of nitroglycerin. Nitroglycerin does not impact the underlying pathophysiology of coronary artery atherosclerosis.

2. A nurse is discussing heart failure with a group of nursing students. Which statement by a student reflects an understanding of how compensatory mechanisms can compound existing problems in patients with heart failure? a. "An increase in arteriolar tone to improve tissue perfusion can decrease resistance." b. "An increase in contractility to increase cardiac output can cause pulmonary edema." c. "When the heart rate increases to increase cardiac output, it can prevent adequate filling of the ventricles." d. "When venous tone increases to increase ventricular filling, an increase in arterial pressure occurs."

ANS: C The heart rate increases to improve cardiac output, but it may prevent adequate ventricular filling. An increase in arteriole tone improves tissue perfusion but also increases both the resistance to the pumping of the heart and the cardiac workload. Increased contractility helps improve cardiac output but is detrimental because it increases the oxygen demand of the heart. An increase in venous tone improves ventricular filling, but as the ventricles fail, blood can back up and cause pulmonary edema.

42. After reviewing information shown in the accompanying figure from the medical records of a 43-year-old, which risk factor modification for coronary artery disease should the nurse include in patient teaching? a. Importance of daily physical activity b. Effect of weight loss on blood pressure c. Dietary changes to improve lipid levels d. Ongoing cardiac risk associated with history of tobacco use

ANS: C The patient has an elevated low-density lipoprotein (LDL) cholesterol and low high-density lipoprotein (HDL) cholesterol, which will increase the risk of coronary artery disease. Although the blood pressure is in the prehypertensive range, the patient's waist circumference and body mass index (BMI) indicate an appropriate body weight. The risk for coronary artery disease a year after quitting smoking is the same as a nonsmoker. The patient's occupation indicates that daily activity is at the levels suggested by national guidelines.

30. Which electrocardiographic (ECG) change is most important for the nurse to report to the health care provider when caring for a patient with chest pain? a. Inverted P wave b. Sinus tachycardia c. ST-segment elevation d. First-degree atrioventricular block

ANS: C The patient is likely to be experiencing an ST-segment-elevation myocardial infarction (STEMI). Immediate therapy with percutaneous coronary intervention (PCI) or thrombolytic medication is indicated to minimize myocardial damage. The other ECG changes may also suggest a need for therapy, but not as rapidly.

18. In preparation for discharge, the nurse teaches a patient with chronic stable angina how to use the prescribed short-acting and long-acting nitrates. Which patient statement indicates that the teaching has been effective? a. "I will check my pulse rate before I take any nitroglycerin tablets." b. "I will put the nitroglycerin patch on as soon as I get any chest pain." c. "I will stop what I am doing and sit down before I put the nitroglycerin under my tongue." d. "I will be sure to remove the nitroglycerin patch before taking any sublingual nitroglycerin."

ANS: C The patient should sit down before taking the nitroglycerin to decrease cardiac workload and prevent orthostatic hypotension. Transdermal nitrates are used prophylactically rather than to treat acute pain and can be used concurrently with sublingual nitroglycerin. Although the nurse should check blood pressure before giving nitroglycerin, patients do not need to check the pulse rate before taking nitrates.

6. A patient seen in the asthma clinic has recorded daily peak flows that are 75% of the baseline. Which action will the nurse plan to take next? a. Increase the dose of the leukotriene inhibitor. b. Teach the patient about the use of oral corticosteroids. c. Administer a bronchodilator and recheck the peak flow. d. Instruct the patient to keep the next scheduled follow-up appointment.

ANS: C The patient's peak flow reading indicates that the condition is worsening (yellow zone). The patient should take the bronchodilator and recheck the peak flow. Depending on whether the patient returns to the green zone, indicating well-controlled symptoms, the patient may be prescribed oral corticosteroids or a change in dosing of other medications. Keeping the next appointment is appropriate, but the patient also needs to be taught how to control symptoms now and use the bronchodilator. DIF: Cognitive Level: Apply (application) REF: 580 TOP: Nursing Process: Planning MSC: NCLEX: Physiological Integrity

17. A patient is recovering from a myocardial infarction (MI) and develops chest pain on day 3 that increases when taking a deep breath and is relieved by leaning forward. Which action should the nurse take next? a. Assess the feet for pedal edema. b. Palpate the radial pulses bilaterally. c. Auscultate for a pericardial friction rub. d. Check the heart monitor for dysrhythmias.

ANS: C The patient's symptoms are consistent with the development of pericarditis, a possible complication of MI. The other assessments listed are not consistent with the description of the patient's symptoms.

37. A patient who has chest pain is admitted to the emergency department (ED) and all of the following are ordered. Which one should the nurse arrange to be completed first? a. Chest x-ray b. Troponin level c. Electrocardiogram (ECG) d. Insertion of a peripheral IV

ANS: C The priority for the patient is to determine whether an acute myocardial infarction (AMI) is occurring so that reperfusion therapy can begin as quickly as possible. ECG changes occur very rapidly after coronary artery occlusion, and an ECG should be obtained as soon as possible. Troponin levels will increase after about 3 hours. Data from the chest x-ray may impact the patient's care but are not helpful in determining whether the patient is experiencing a myocardial infarction (MI). Peripheral access will be needed but not before the ECG.

41. A patient with diabetes mellitus and chronic stable angina has a new order for captopril (Capoten). The nurse should teach the patient that the primary purpose of captopril is to a. lower heart rate. b. control blood glucose levels. c. prevent changes in heart muscle. d. reduce the frequency of chest pain.

ANS: C The purpose for angiotensin-converting enzyme (ACE) inhibitors in patients with chronic stable angina who are at high risk for a cardiac event is to decrease ventricular remodeling. ACE inhibitors do not directly impact angina frequency, blood glucose, or heart rate.

16. A patient with heart failure who takes a thiazide diuretic and digoxin [Lanoxin] is admitted for shortness of breath. The patient's heart rate is 66 beats per minute, and the blood pressure is 130/88 mm Hg. The serum potassium level is 3.8 mEq/L, and the digoxin level is 0.8 ng/mL. The nurse admitting this patient understands that the patient: a. has digoxin toxicity. b. is showing signs of renal failure. c. is experiencing worsening of the disease. d. needs a potassium-sparing diuretic.

ANS: C This patient has a normal serum potassium level, and the digoxin level is normal. The patient is showing signs of pulmonary edema, which indicates progression of heart failure. The digoxin level is within normal limits, and the heart rate is above 60 beats per minute, so digoxin toxicity is not likely. There is no sign of renal failure. A potassium-sparing diuretic is not indicated.

2. A patient who is taking digoxin is admitted to the hospital for treatment of congestive heart failure. The prescriber has ordered furosemide [Lasix]. The nurse notes an irregular heart rate of 86 beats per minute, a respiratory rate of 22 breaths per minute, and a blood pressure of 130/82 mm Hg. The nurse auscultates crackles in both lungs. Which laboratory value causes the nurse the most concern? a. Blood glucose level of 120 mg/dL b. Oxygen saturation of 90% c. Potassium level of 3.5 mEq/L d. Sodium level of 140 mEq/L

ANS: C This patient has an irregular, rapid heartbeat that might be caused by a dysrhythmia. This patient's serum potassium level is low, which can trigger fatal dysrhythmias, especially in patients taking digoxin. Furosemide contributes to loss of potassium through its effects on the distal nephron. Potassium-sparing diuretics often are used in conjunction with furosemide to prevent this complication. This patient's serum glucose and sodium levels are normal and of no concern at this point, although they can be affected by furosemide. The oxygen saturation is somewhat low and needs to be monitored, although it may improve with diuresis.

3. A patient with chronic hypertension is admitted to the hospital. During the admission assessment, the nurse notes a heart rate of 96 beats per minute, a blood pressure of 150/90 mm Hg, bibasilar crackles, 2+ pitting edema of the ankles, and distension of the jugular veins. The nurse will contact the provider to request an order for which medication? a. ACE inhibitor b. Digoxin [Lanoxin] c. Furosemide [Lasix] d. Spironolactone [Aldactone]

ANS: C This patient shows signs of fluid volume overload and needs a diuretic. Furosemide is a loop diuretic, which can produce profound diuresis very quickly even when the glomerular filtration rate (GFR) is low. An ACE inhibitor will not reduce fluid volume overload. Digoxin has a positive inotropic effect on the heart, which may improve renal perfusion, but this is not its primary effect. Spironolactone is a potassium-sparing diuretic with weak diuresis effects; it is used in conjunction with other diuretics to improve electrolyte balance.

14. A patient with ST-segment elevation in three contiguous electrocardiographic (ECG) leads is admitted to the emergency department (ED) and diagnosed as having an ST-segment-elevation myocardial infarction (STEMI). Which question should the nurse ask to determine whether the patient is a candidate for thrombolytic therapy? a. "Do you have any allergies?" b. "Do you take aspirin on a daily basis?" c. "What time did your chest pain begin?" d. "Can you rate your chest pain using a 0 to 10 scale?"

ANS: C Thrombolytic therapy should be started within 6 hours of the onset of the myocardial infarction (MI), so the time at which the chest pain started is a major determinant of the appropriateness of this treatment. The other information will also be needed, but it will not be a factor in the decision about thrombolytic therapy.

34. The nurse assesses a patient with a history of asthma. Which assessment finding indicates that the nurse should take immediate action? a. Pulse oximetry reading of 91% b. Respiratory rate of 26 breaths/minute c. Use of accessory muscles in breathing d. Peak expiratory flow rate of 240 L/minute

ANS: C Use of accessory muscle indicates that the patient is experiencing respiratory distress and rapid intervention is needed. The other data indicate the need for ongoing monitoring and assessment but do not suggest that immediate treatment is required. DIF: Cognitive Level: Apply (application) REF: 564-565 OBJ: Special Questions: Prioritization TOP: Nursing Process: Assessment MSC: NCLEX: Physiological Integrity

31. The nurse takes an admission history on a patient with possible asthma who has new-onset wheezing and shortness of breath. Which information may indicate a need for a change in therapy? a. The patient has chronic inflammatory bowel disease. b. The patient has a history of pneumonia 6 months ago. c. The patient takes propranolol (Inderal) for hypertension. d. The patient uses acetaminophen (Tylenol) for headaches.

ANS: C β-Blockers such as propranolol can cause bronchospasm in some patients with asthma. The other information will be documented in the health history but does not indicate a need for a change in therapy. DIF: Cognitive Level: Apply (application) REF: 576 TOP: Nursing Process: Assessment MSC: NCLEX: Physiological Integrity

A patient with renal failure who has been taking aluminum hydroxide/magnesium hydroxide suspension (Maalox) at home for indigestion is somnolent and has decreased deep tendon reflexes. Which action should the nurse take first? a. Notify the patient's health care provider. b. Withhold the next scheduled dose of Maalox. c. Review the magnesium level on the patient's chart. d. Check the chart for the most recent potassium level.

ANS: C Review the magnesium level on the patient's chart. The patient has a history and symptoms consistent with hypermagnesemia; the nurse should check the chart for a recent serum magnesium level. Notification of the health care provider will be done after the nurse knows the magnesium level. The Maalox should be held, but more immediate action is needed to correct the patient's decreased deep tendon reflexes (DTRs) and somnolence. Monitoring of potassium levels also is important for patients with renal failure, but the patient's current symptoms are not consistent with hyperkalemia.

A recently admitted patient has a small cell carcinoma of the lung, which is causing the syndrome of inappropriate antidiuretic hormone (SIADH). The nurse will monitor carefully for a. increased total urinary output. b. elevation of serum hematocrit. c. decreased serum sodium level. d. rapid and unexpected weight loss.

ANS: C decreased serum sodium level. SIADH causes water retention and a decrease in serum sodium level. Weight loss, increased urine output, and elevated serum hematocrit may be associated with excessive loss of water, but not with SIADH and water retention.

34. A patient with pneumonia has a fever of 101.4° F (38.6° C), a nonproductive cough, and an oxygen saturation of 88%. The patient complains of weakness, fatigue, and needs assistance to get out of bed. Which nursing diagnosis should the nurse assign as the highest priority? a. Hyperthermia related to infectious illness b. Impaired transfer ability related to weakness c. Ineffective airway clearance related to thick secretions d. Impaired gas exchange related to respiratory congestion

ANS: D All these nursing diagnoses are appropriate for the patient, but the patient's oxygen saturation indicates that all body tissues are at risk for hypoxia unless the gas exchange is improved.

12. An alcoholic and homeless patient is diagnosed with active tuberculosis (TB). Which intervention by the nurse will be most effective in ensuring adherence with the treatment regimen? a. Arrange for a friend to administer the medication on schedule. b. Give the patient written instructions about how to take the medications. c. Teach the patient about the high risk for infecting others unless treatment is followed. d. Arrange for a daily noon meal at a community center where the drug will be administered.

ANS: D Directly observed therapy is the most effective means for ensuring compliance with the treatment regimen, and arranging a daily meal will help ensure that the patient is available to receive the medication. The other nursing interventions may be appropriate for some patients but are not likely to be as helpful for this patient.

8. A patient is admitted with active tuberculosis (TB). The nurse should question a health care provider's order to discontinue airborne precautions unless which assessment finding is documented? a. Chest x-ray shows no upper lobe infiltrates. b. TB medications have been taken for 6 months. c. Mantoux testing shows an induration of 10 mm. d. Three sputum smears for acid-fast bacilli are negative.

ANS: D Negative sputum smears indicate that Mycobacterium tuberculosis is not present in the sputum, and the patient cannot transmit the bacteria by the airborne route. Chest x-rays are not used to determine whether treatment has been successful. Taking medications for 6 months is necessary, but the multidrug-resistant forms of the disease might not be eradicated after 6 months of therapy. Repeat Mantoux testing would not be done because the result will not change even with effective treatment.

38. The nurse is performing tuberculosis (TB) skin tests in a clinic that has many patients who have immigrated to the United States. Which question is most important for the nurse to ask before the skin test? a. "Is there any family history of TB?" b. "How long have you lived in the United States?" c. "Do you take any over-the-counter (OTC) medications?" d. "Have you received the bacille Calmette-Guérin (BCG) vaccine for TB?"

ANS: D Patients who have received the BCG vaccine will have a positive Mantoux test. Another method for screening (such as a chest x-ray) will need to be used in determining whether the patient has a TB infection. The other information also may be valuable but is not as pertinent to the decision about doing TB skin testing.

The nurse is caring for a patient who was admitted to the telemetry unit with a diagnosis of rule/out acute MI. The patient's chest pain began 3 hours ago. Which of the following laboratory tests would be most helpful in confirming the diagnosis of a current MI? a) Creatinine kinase-myoglobin (CK-MB) level b) Troponin C level c) Myoglobin level d) CK-MM

a) Creatinine kinase-myoglobin (CK-MB) level Elevated CK-MB assessment by mass assay is an indicator of acute MI; the levels begin to increase within a few hours and peak within 24 hours of an MI. If the area is reperfused (due to thrombotic therapy or PCI), it peaks earlier. CK-MM (skeletal muscle) is not an indicator of cardiac muscle damage. There are three isomers of troponin: C, I, and T. Troponin I and T are specific for cardiac muscle, and these biomarkers are currently recognized as reliable and critical markers of myocardial injury. An increase in myoglobin is not very specific in indicating an acute cardiac event; however, negative results are an excellent parameter for ruling out an acute MI.

Which of the following body system responses correlates with systolic heart failure (HF)? a) Decrease in renal perfusion b) Increased blood volume ejected from ventricle c) Vasodilation of skin d) Dehydration

a) Decrease in renal perfusion

11. Nadolol (Corgard) is prescribed for a patient with chronic stable angina and left ventricular dysfunction. To determine whether the drug is effective, the nurse will monitor for a. decreased blood pressure and heart rate. b. fewer complaints of having cold hands and feet. c. improvement in the strength of the distal pulses. d. the ability to do daily activities without chest pain.

ANS: D Because the medication is ordered to improve the patient's angina, effectiveness is indicated if the patient is able to accomplish daily activities without chest pain. Blood pressure and heart rate may decrease, but these data do not indicate that the goal of decreased angina has been met. The noncardioselective β-adrenergic blockers can cause peripheral vasoconstriction, so the nurse would not expect an improvement in distal pulse quality or skin temperature.

4. Which information given by a patient admitted with chronic stable angina will help the nurse confirm this diagnosis? a. The patient states that the pain "wakes me up at night." b. The patient rates the pain at a level 3 to 5 (0 to 10 scale). c. The patient states that the pain has increased in frequency over the last week. d. The patient states that the pain "goes away" with one sublingual nitroglycerin tablet.

ANS: D Chronic stable angina is typically relieved by rest or nitroglycerin administration. The level of pain is not a consistent indicator of the type of angina. Pain occurring at rest or with increased frequency is typical of unstable angina.

A patient taking gemfibrozil [Lopid] and rosuvastatin [Crestor] concurrently begins to complain of muscle aches, fatigue, and weakness. What should the nurse monitor? a. For tendon tenderness b. For a lupus-like syndrome c. The patient's liver function test results d. The patient's creatinine kinase levels

ANS: D Creatinine kinase levels are the best laboratory indicator of myopathy and/or rhabdomyolysis, which may lead to renal failure. As with the statins, gemfibrozil and other fibrates can cause myopathy. Fibrates must be used with caution in patients taking statins. Concurrent use of gemfibrozil and rosuvastatin does not cause tendon tenderness or a lupus-like syndrome. Liver function levels should be determined at the start of statin therapy and every 6 months thereafter in patients who do not have liver disease.

20. The nurse develops a teaching plan to help increase activity tolerance at home for an older adult with severe chronic obstructive pulmonary disease (COPD). Which instructions would be most appropriate for the nurse to include in the plan of care? a. Stop exercising when short of breath. b. Walk until pulse rate exceeds 130 beats/minute. c. Limit exercise to activities of daily living (ADLs). d. Walk 15 to 20 minutes daily at least 3 times/week.

ANS: D Encourage the patient to walk 15 to 20 minutes a day at least three times a week with gradual increases. Shortness of breath is normal with exercise and not an indication that the patient should stop. Limiting exercise to ADLs will not improve the patient's exercise tolerance. A 70-year-old patient should have a pulse rate of 120 or less with exercise (80% of the maximal heart rate of 150). DIF: Cognitive Level: Apply (application) REF: 599 TOP: Nursing Process: Planning MSC: NCLEX: Physiological Integrity

A patient in the emergency department has severe chest pain. The nurse administers morphine intravenously. The patient asks the nurse why morphine is given. Which response by the nurse is correct? a. "Morphine helps by reducing anxiety and relieving pain." b. "Morphine helps by reducing pain and dissolving clots." c. "Morphine helps by relieving pain and lowering blood pressure." d. "Morphine helps by relieving pain and reducing the cardiac oxygen demand."

ANS: D IV morphine is the treatment of choice for STEMI-associated pain. Besides relieving pain, it promotes vasodilation and reduces cardiac preload, which lowers the cardiac oxygen demand. It does not reduce anxiety, dissolve clots, or lower blood pressure.

2. A patient with stable COPD receives prescriptions for an inhaled glucocorticoid and an inhaled beta2-adrenergic agonist. Which statement by the patient indicates understanding of this medication regimen? a. "I should use the glucocorticoid as needed when symptoms flare." b. "I will need to use the beta2-adrenergic agonist drug daily." c. "The beta2-adrenergic agonist suppresses the synthesis of inflammatory mediators." d. "The glucocorticoid is used as prophylaxis to prevent exacerbations."

ANS: D Inhaled glucocorticoids are used daily to prevent acute attacks. They are not used PRN. The beta2-adrenergic agonist drugs should not be used daily; they are used to treat symptoms as needed. They do not suppress mediators of inflammation.

A patient with a history of elevated triglycerides and LDL cholesterol begins taking nicotinic acid [Niacin]. The patient reports uncomfortable flushing of the face, neck, and ears when taking the drug. What will the nurse advise the patient? a. "Ask your provider about taking an immediate-release form of the medication." b. "Ask your provider about assessing your serum uric acid levels which may be elevated." c. "You should stop taking the Niacin immediately since this is a serious adverse effect." d. "You should take 325 mg of aspirin a half hour before each dose of Niacin to prevent this effect."

ANS: D Intense flushing of the face, neck, and ears occurs in practically all patients taking nicotinic acid in pharmacologic doses. Patients should be advised to take 325 mg of ASA 30 minutes prior to each dose to minimize this effect or to use an extended-release form of the drug. Serum uric acid levels may increase with Niacin use, but flushing does not indicate elevated levels. This side effect is not serious and does not warrant discontinuation of the drug.

13. A patient with persistent, frequent asthma exacerbations asks a nurse about a long-acting beta2-agonist medication. What will the nurse tell this patient? a. LABAs are safer than short-acting beta2 agonists. b. LABAs can be used on an as-needed basis to treat symptoms. c. LABAs reduce the risk of asthma-related deaths. d. LABAs should be combined with an inhaled glucocorticoid.

ANS: D LABAs can increase the risk of asthma-related deaths when used improperly; this risk is minimized when LABAs are combined with an inhaled glucocorticoid. LABAs are not safer than SABAs, and they are not used PRN. LABAs increase the risk of asthma-related deaths.

36. A patient had a non-ST-segment-elevation myocardial infarction (NSTEMI) 3 days ago. Which nursing intervention included in the plan of care is most appropriate for the registered nurse (RN) to delegate to an experienced licensed practical/vocational nurse (LPN/LVN)? a. Evaluation of the patient's response to walking in the hallway b. Completion of the referral form for a home health nurse follow-up c. Education of the patient about the pathophysiology of heart disease d. Reinforcement of teaching about the purpose of prescribed medications

ANS: D LPN/LVN education and scope of practice include reinforcing education that has previously been done by the RN. Evaluating the patient response to exercise after a NSTEMI requires more education and should be done by the RN. Teaching and discharge planning/ documentation are higher level skills that require RN education and scope of practice.

30. The nurse completes an admission assessment on a patient with asthma. Which information given by patient is most indicative of a need for a change in therapy? a. The patient uses albuterol (Proventil) before any aerobic exercise. b. The patient says that the asthma symptoms are worse every spring. c. The patient's heart rate increases after using the albuterol (Proventil) inhaler. d. The patient's only medications are albuterol (Proventil) and salmeterol (Serevent).

ANS: D Long-acting β2-agonists should be used only in patients who also are using an inhaled corticosteroid for long-term control. Salmeterol should not be used as the first-line therapy for long-term control. Using a bronchodilator before exercise is appropriate. The other information given by the patient requires further assessment by the nurse, but is not unusual for a patient with asthma. DIF: Cognitive Level: Apply (application) REF: 572 TOP: Nursing Process: Assessment MSC: NCLEX: Physiological Integrity

8. A nurse is teaching a group of nursing students about the different formulations of beta2-adrenergic agonist medications. Which statement by a student indicates understanding of the teaching? a. "Beta2-adrenergic agonists provide quick relief via any formulation." b. "Long-acting beta2 agonists may be used alone to prevent attacks." c. "Short-acting beta2 agonists are usually given by nebulizer." d. "Oral beta2 agonists are not useful for short-term treatment."

ANS: D Oral beta2 agonists are used only for long-term control. All formulations vary; long-acting beta2 agonists (LABAs) and oral preparations are used for long-term control, whereas short-acting beta2 agonists (SABAs) are useful for acute episodes. LABAs are used in conjunction with inhaled glucocorticoids to prevent attacks. SABAs may be given by MDI or nebulizer and usually are given by MDI.

16. Which medication should be used for asthma patients as part of step 1 management? a. Combination inhaled glucocorticoids/long-acting beta2 agonists b. Inhaled low-dose glucocorticoids c. Long-acting beta2 agonists d. Short-acting beta2 agonists

ANS: D Patients needing step 1 management have intermittent, mild symptoms and can be managed with a SABA as needed. Combination inhaled glucocorticoids/LABAs are used for step 3 management. Inhaled low-dose glucocorticoids are used for step 2 management. LABAs, along with inhaled glucocorticoids, are used for step 3 management.

7. A patient with heart failure who has been taking an ACE inhibitor, a thiazide diuretic, and a beta blocker for several months comes to the clinic for evaluation. As part of the ongoing assessment of this patient, the nurse will expect the provider to evaluate: a. complete blood count. b. ejection fraction. c. maximal exercise capacity. d. serum electrolyte levels.

ANS: D Patients taking thiazide diuretics can develop hypokalemia, which can increase the risk for dysrhythmias; therefore, the serum electrolyte levels should be monitored closely. A complete blood count is not recommended. This patient is taking the drugs recommended for patients with Stage C heart failure; although the patient's quality of life and ability to participate in activities should be monitored, routine measurement of the ejection fraction and maximal exercise capacity is not recommended.

43. After reviewing a patient's history, vital signs, physical assessment, and laboratory data, which information shown in the accompanying figure is most important for the nurse to communicate to the health care provider? a. Q waves on ECG b. Elevated troponin levels c. Fever and hyperglycemia d. Tachypnea and crackles in lungs

ANS: D Pulmonary congestion and tachypnea suggest that the patient may be developing heart failure, a complication of myocardial infarction (MI). Mild fever and hyperglycemia are common after MI because of the inflammatory process that occurs with tissue necrosis. Troponin levels will be elevated for several days after MI. Q waves often develop with ST-segment-elevation MI.

23. A 55-year-old patient with increasing dyspnea is being evaluated for a possible diagnosis of chronic obstructive pulmonary disease (COPD). When teaching a patient about pulmonary function testing (PFT) for this condition, what is the most important question the nurse should ask? a. "Are you claustrophobic?" b. "Are you allergic to shellfish?" c. "Do you have any metal implants or prostheses?" d. "Have you taken any bronchodilators in the past 6 hours?"

ANS: D Pulmonary function testing will help establish the COPD diagnosis. Bronchodilators should be avoided at least 6 hours before the test. PFTs do not involve being placed in an enclosed area such as for magnetic resonance imaging (MRI). Contrast dye is not used for PFTs. The patient may still have PFTs done if metal implants or prostheses are present, as these are contraindications for an MRI. DIF: Cognitive Level: Apply (application) REF: 566 TOP: Nursing Process: Planning MSC: NCLEX: Physiological Integrity

10. A patient with chronic obstructive pulmonary disease (COPD) has a nursing diagnosis of impaired breathing pattern related to anxiety. Which nursing action is most appropriate to include in the plan of care? a. Titrate oxygen to keep saturation at least 90%. b. Discuss a high-protein, high-calorie diet with the patient. c. Suggest the use of over-the-counter sedative medications. d. Teach the patient how to effectively use pursed lip breathing.

ANS: D Pursed lip breathing techniques assist in prolonging the expiratory phase of respiration and decrease air trapping. There is no indication that the patient requires oxygen therapy or an improved diet. Sedative medications should be avoided because they decrease respiratory drive. DIF: Cognitive Level: Apply (application) REF: 578 TOP: Nursing Process: Planning MSC: NCLEX: Physiological Integrity

7. The nurse teaches a patient who has asthma about peak flow meter use. Which action by the patient indicates that teaching was successful? a. The patient inhales rapidly through the peak flow meter mouthpiece. b. The patient takes montelukast (Singulair) for peak flows in the red zone. c. The patient calls the health care provider when the peak flow is in the green zone. d. The patient uses albuterol (Proventil) metered dose inhaler (MDI) for peak flows in the yellow zone.

ANS: D Readings in the yellow zone indicate a decrease in peak flow. The patient should use short-acting β2-adrenergic (SABA) medications. Readings in the green zone indicate good asthma control. The patient should exhale quickly and forcefully through the peak flow meter mouthpiece to obtain the readings. Readings in the red zone do not indicate good peak flow, and the patient should take a fast-acting bronchodilator and call the health care provider for further instructions. Singulair is not indicated for acute attacks but rather is used for maintenance therapy. DIF: Cognitive Level: Apply (application) REF: 568 | 580 TOP: Nursing Process: Evaluation MSC: NCLEX: Physiological Integrity

A nurse is instructing a patient receiving a cholesterol-lowering agent. Which information should the nurse include in the patient education? a. "This medication will replace other interventions you have been trying." b. "It is important for you to double your dose if you miss one to maintain therapeutic blood levels." c. "Stop taking the medication if you experience constipation." d. "You should continue your exercise program to increase your HDL serum levels."

ANS: D Regular exercise can reduce LDL cholesterol and elevate HDL cholesterol, thereby reducing the risk of coronary heart disease (CHD). The patient should consider the cholesterol-lowering drug an adjunct to a proper diet and exercise. Drug therapy cannot replace other important interventions, such as diet and exercise. The patient should never be instructed to double the dose. Constipation is a side effect of most cholesterol-lowering agents. The patient should be encouraged to eat a high-fiber diet and increase fluids if not contraindicated.

5. A patient with hypertension is taking furosemide [Lasix] for congestive heart failure. The prescriber orders digoxin to help increase cardiac output. What other medication will the nurse expect to be ordered for this patient? a. Bumetanide [Bumex] b. Chlorothiazide [Diuril] c. Hydrochlorothiazide [HydroDIURIL] d. Spironolactone [Aldactone]

ANS: D Spironolactone is used in conjunction with furosemide because of its potassium-sparing effects. Furosemide can contribute to hypokalemia, which can increase the risk of fatal dysrhythmias, especially with digoxin administration. The other diuretics listed are all potassium-wasting diuretics.

4. A nurse preparing to administer morning medications notes that a patient with a history of hypertension has been prescribed spironolactone [Aldactone]. The nurse assesses the patient and notes dyspnea, bilateral crackles, and pitting edema in both feet. Which intervention is appropriate? a. Administer the medications as ordered. b. Ask the patient about the use of salt substitutes. c. Contact the provider to request an order for serum electrolytes. d. Request an order for furosemide [Lasix].

ANS: D Spironolactone takes up to 48 hours for its effects to develop, so it should not be used when the patient needs immediate diuresis. This patient has shortness of breath, crackles, and edema and needs a short-acting diuretic, such as furosemide. Asking the patient about the use of salt substitutes is not indicated. The patient does not need assessment of serum electrolytes.

11. A nurse prepares to administer a scheduled dose of digoxin. The nurse finds a new laboratory report showing a plasma digoxin level of 0.7 ng/mL. What action should the nurse take? a. Withhold the drug for an hour and reassess the level. b. Withhold the drug and notify the prescriber immediately. c. Administer Digibind to counteract the toxicity. d. Check the patient's apical pulse, and if it is within a safe range, administer the digoxin.

ANS: D The optimum plasma digoxin range is 0.5 to 0.8 ng/mL. The patient's pulse should be checked before administration, as always, and the digoxin should be administered. The digoxin does not have to be withheld, nor does the prescriber need to be notified. If the digoxin level is demonstrating a trend of increasing, the issue should be discussed in rounds. Digibind is not indicated, because the digoxin level is therapeutic.

21. A patient with severe chronic obstructive pulmonary disease (COPD) tells the nurse, "I wish I were dead! I'm just a burden on everybody." Based on this information, which nursing diagnosis is most appropriate? a. Complicated grieving related to expectation of death b. Ineffective coping related to unknown outcome of illness c. Deficient knowledge related to lack of education about COPD d. Chronic low self-esteem related to increased physical dependence

ANS: D The patient's statement about not being able to do anything for himself or herself supports this diagnosis. Although deficient knowledge, complicated grieving, and ineffective coping may also be appropriate diagnoses for patients with COPD, the data for this patient do not support these diagnoses. DIF: Cognitive Level: Apply (application) REF: 599-600 TOP: Nursing Process: Diagnosis MSC: NCLEX: Psychosocial Integrity

17. A patient hospitalized with chronic obstructive pulmonary disease (COPD) is being discharged home on oxygen therapy. Which instruction should the nurse include in the discharge teaching? a. Storage of oxygen tanks will require adequate space in the home. b. Travel opportunities will be limited because of the use of oxygen. c. Oxygen flow should be increased if the patient has more dyspnea. d. Oxygen use can improve the patient's prognosis and quality of life.

ANS: D The use of home oxygen improves quality of life and prognosis. Because increased dyspnea may be a symptom of an acute process such as pneumonia, the patient should notify the physician rather than increasing the oxygen flow rate if dyspnea becomes worse. Oxygen can be supplied using liquid, storage tanks, or concentrators, depending on individual patient circumstances. Travel is possible using portable oxygen concentrators. DIF: Cognitive Level: Apply (application) REF: 592 TOP: Nursing Process: Implementation MSC: NCLEX: Physiological Integrity

19. A patient with COPD is prescribed tiotropium [Spiriva]. After the initial dose, the patient reports only mild relief within 30 minutes. What will the nurse tell the patient? a. "You may have another dose in 4 hours." b. "You may need to take two inhalations instead of one." c. "You should have peak effects in about 6 hours." d. "You should see improved effects within the next week."

ANS: D Tiotropium shows therapeutic effects in about 30 minutes, with improved bronchodilation occurring with subsequent doses, up to 8 days. The medication is given once daily, with inhalation of one capsule. Peak effects occur in 3 hours.

9. A patient with asthma comes to a clinic for treatment of an asthma exacerbation. The patient's medication history lists an inhaled glucocorticoid, montelukast [Singulair], and a SABA as needed via MDI. The nurse assesses the patient and notes a respiratory rate of 18 breaths per minute, a heart rate of 96 beats per minute, and an oxygen saturation of 95%. The nurse auscultates mild expiratory wheezes and equal breath sounds bilaterally. What will the nurse do? a. Contact the provider to request a systemic glucocorticoid. b. Contact the provider to suggest using a long-acting beta2 agonist. c. Evaluate the need for teaching about MDI use. d. Question the patient about how much albuterol has been used.

ANS: D To determine the next course of action, it is important to assess the drugs given before these symptoms were observed. Patients who continue to wheeze after using a SABA need systemic glucocorticoids and nebulized albuterol. If a SABA has not been used, that will be the first intervention. LABAs are not used for exacerbations. If a patient reports using a SABA without good results, evaluating the MDI technique may be warranted.

A patient who is recovering from a STEMI 3 months prior is in the clinic for a follow-up evaluation. The patient is taking 81 mg of aspirin, a beta blocker, and an ACE inhibitor daily and uses nitroglycerine as needed for angina. The patient's BMI is 24.5 kg/m2, and serum LDL is 150 mg/dL. The patient has a blood pressure of 135/80 mm Hg. What will the nurse expect the provider to order for this patient? a. An antihypertensive medication b. Counseling about a weight loss diet c. Discontinuing the ACE inhibitor d. High-dose statin therapy

ANS: D To help prevent recurrence of MI in patients post-STEMI, a high-dose statin should be given to patients with elevated cholesterol. This patient's blood pressure and BMI are normal, so antihypertensives and a weight loss diet are not recommended. The three drugs should be continued indefinitely.

4. Which information will the nurse include in the asthma teaching plan for a patient being discharged? a. Use the inhaled corticosteroid when shortness of breath occurs. b. Inhale slowly and deeply when using the dry powder inhaler (DPI). c. Hold your breath for 5 seconds after using the bronchodilator inhaler. d. Tremors are an expected side effect of rapidly acting bronchodilators.

ANS: D Tremors are a common side effect of short-acting β2-adrenergic (SABA) medications and not a reason to avoid using the SABA inhaler. Inhaled corticosteroids do not act rapidly to reduce dyspnea. Rapid inhalation is needed when using a DPI. The patient should hold the breath for 10 seconds after using inhalers. DIF: Cognitive Level: Apply (application) REF: 572 TOP: Nursing Process: Implementation MSC: NCLEX: Physiological Integrity

8. A patient who has had chest pain for several hours is admitted with a diagnosis of rule out acute myocardial infarction (AMI). Which laboratory test should the nurse monitor to help determine whether the patient has had an AMI? a. Myoglobin b. Homocysteine c. C-reactive protein d. Cardiac-specific troponin

ANS: D Troponin levels increase about 4 to 6 hours after the onset of myocardial infarction (MI) and are highly specific indicators for MI. Myoglobin is released within 2 hours of MI, but it lacks specificity and its use is limited. The other laboratory data are useful in determining the patient's risk for developing coronary artery disease (CAD) but are not helpful in determining whether an acute MI is in progress.

1. A patient with asthma will be using a metered-dose inhaler (MDI) for delivery of an inhaled medication. The provider has ordered 2 puffs to be given twice daily. It is important for the nurse to teach this patient that: a. the patient should inhale suddenly to receive the maximum dose. b. the patient should activate the device and then inhale. c. the patient should store the MDI in the refrigerator between doses. d. the patient should wait 1 minute between puffs.

ANS: D When two puffs are needed, an interval of at least 1 minute should separate the first puff from the second. Sudden inhalation can cause bronchospasm. The patient should begin inhaling and then activate the device. There is no need to store the drug in the refrigerator.

Spironolactone (Aldactone), an aldosterone antagonist, is prescribed for a patient as a diuretic. Which statement by the patient indicates that the teaching about this medication has been effective? a. "I will try to drink at least 8 glasses of water every day." b. "I will use a salt substitute to decrease my sodium intake." c. "I will increase my intake of potassium-containing foods." d. "I will drink apple juice instead of orange juice for breakfast."

ANS: D "I will drink apple juice instead of orange juice for breakfast." Since spironolactone is a potassium-sparing diuretic, patients should be taught to choose low potassium foods such as apple juice rather than foods that have higher levels of potassium, such as citrus fruits. Because the patient is using spironolactone as a diuretic, the nurse would not encourage the patient to increase fluid intake. Teach patients to avoid salt substitutes, which are high in potassium.

A postoperative patient who is receiving nasogastric suction is complaining of anxiety and incisional pain. The patient's respiratory rate is 32 breaths/minute and the arterial blood gases (ABGs) indicate respiratory alkalosis. Which action should the nurse take first? a. Discontinue the nasogastric suctions for a few hours. b. Notify the health care provider about the ABG results. c. Teach the patient about the need to take slow, deep breaths. d. Give the patient the PRN morphine sulfate 4 mg intravenously.

ANS: D Give the patient the PRN morphine sulfate 4 mg intravenously. The patient's respiratory alkalosis is caused by the increased respiratory rate associated with pain and anxiety. The nurse's first action should be to medicate the patient for pain. Although the nasogastric suction may contribute to the alkalosis, it is not appropriate to discontinue the tube when the patient needs gastric suction. The health care provider may be notified about the ABGs but is likely to instruct the nurse to medicate for pain. The patient will not be able to take slow, deep breaths when experiencing pain.

A patient with hypercalcemia is being cared for on the medical unit. Nursing actions included on the care plan will include a. maintaining the patient on bed rest. b. auscultating lung sounds every 4 hours. c. monitoring for Trousseau's and Chvostek's signs. d. encouraging fluid intake up to 4000 ml every day.

ANS: D encouraging fluid intake up to 4000 ml every day. To decrease the risk for renal calculi, the patient should have an intake of 3000 to 4000 ml daily. Ambulation helps decrease the loss of calcium from bone and is encouraged in patients with hypercalcemia. Trousseau's and Chvostek's signs are monitored when there is a possibility of hypocalcemia. There is no indication that the patient needs frequent assessment of lung sounds, although these would be assessed every shift.

A patient is taking a potassium-wasting diuretic for treatment of hypertension. The nurse will teach the patient to report symptoms of adverse effects such as a. personality changes. b. frequent loose stools. c. facial muscle spasms. d.generalized weakness.

ANS: D generalized weakness. Generalized weakness progressing to flaccidity is a manifestation of hypokalemia. Facial muscle spasms might occur with hypocalcemia. Loose stools are associated with hyperkalemia. Personality changes are not associated with electrolyte disturbances, although changes in mental status are common manifestations with sodium excess or deficit.

A patient who has required prolonged mechanical ventilation has the following arterial blood gas results: pH 7.48, PaO2 85 mm Hg, PaCO2 32 mm Hg, and HCO3 25 mEq/L. The nurse interprets these results as a. metabolic acidosis. b. metabolic alkalosis. c. respiratory acidosis. d. respiratory alkalosis.

ANS: D respiratory alkalosis. The pH indicates that the patient has alkalosis and the low PaCO2 indicates a respiratory cause. The other responses are incorrect based on the pH and the normal HCO3.

Which of the following is the most common cause of symptomatic hypomagnesemia? a) IV drug use b) Alcoholism c) Sedentary lifestyle d) Burns

Alcoholism

A (You selected: IgE-mediated Correct Explanation: Atopy, the genetic predisposition for the development of an IgE-mediated response to allergens, is the most common identifiable predisposing factor for asthma. Chronic exposure to airway allergens may sensitize IgE antibodies and the cells of the airway.)

Asthma is cause by which type of response? a) IgE-mediated b) IgM-mediated c) IgA-mediated d) IgD-mediated

23. Which intervention will be included in the plan of care for a male patient with acute kidney injury (AKI) who has a temporary vascular access catheter in the left femoral vein? a. Start continuous pulse oximetry. b. Restrict physical activity to bed rest. c. Restrict the patient's oral protein intake. d. Discontinue the urethral retention catheter.

B

24. A 25-year-old male patient has been admitted with a severe crushing injury after an industrial accident. Which laboratory result will be most important to report to the health care provider? a. Serum creatinine level 2.1 mg/dL b. Serum potassium level 6.5 mEq/L c. White blood cell count 11,500/µL d. Blood urea nitrogen (BUN) 56 mg/dL

B

26. A 62-year-old female patient has been hospitalized for 8 days with acute kidney injury (AKI) caused by dehydration. Which information will be most important for the nurse to report to the health care provider? a. The creatinine level is 3.0 mg/dL. b. Urine output over an 8-hour period is 2500 mL. c. The blood urea nitrogen (BUN) level is 67 mg/dL. d. The glomerular filtration rate is <30 mL/min/1.73m2.

B

28. A 42-year-old patient admitted with acute kidney injury due to dehydration has oliguria, anemia, and hyperkalemia. Which prescribed actions should the nurse take first? a. Insert a urinary retention catheter. b. Place the patient on a cardiac monitor. c. Administer epoetin alfa (Epogen, Procrit). d. Give sodium polystyrene sulfonate (Kayexalate).

B

5. A 48-year-old patient with stage 2 chronic kidney disease (CKD) is scheduled for an intravenous pyelogram (IVP). Which order for the patient will the nurse question? a. NPO for 6 hours before procedure b. Ibuprofen (Advil) 400 mg PO PRN for pain c. Dulcolax suppository 4 hours before procedure d. Normal saline 500 mL IV infused before procedure

B

7. Which information will the nurse monitor in order to determine the effectiveness of prescribed calcium carbonate (Caltrate) for a patient with chronic kidney disease (CKD)? a. Blood pressure b. Phosphate level c. Neurologic status d. Creatinine clearance

B

After change-of-shift report, which patient should the nurse assess first? a. 19-year-old with type 1 diabetes who has a hemoglobin A1C of 12% b. 23-year-old with type 1 diabetes who has a blood glucose of 40 mg/dL c. 40-year-old who is pregnant and whose oral glucose tolerance test is 202 mg/dL d. 50-year-old who uses exenatide (Byetta) and is complaining of acute abdominal pain

B

An active 28-year-old male with type 1 diabetes is being seen in the endocrine clinic. Which finding may indicate the need for a change in therapy? a. Hemoglobin A1C level 6.2% b. Blood pressure 146/88 mmHg c. Heart rate at rest 58 beats/minute d. High density lipoprotein (HDL) level 65 mg/dL

B

The nurse identifies a need for additional teaching when the patient who is self-monitoring blood glucose a. washes the puncture site using warm water and soap. b. chooses a puncture site in the center of the finger pad. c. hangs the arm down for a minute before puncturing the site. d. says the result of 120 mg indicates good blood sugar control.

B

The nurse is preparing to teach a 43-year-old man who is newly diagnosed with type 2 diabetes about home management of the disease. Which action should the nurse take first? a. Ask the patient's family to participate in the diabetes education program. b. Assess the patient's perception of what it means to have diabetes mellitus. c. Demonstrate how to check glucose using capillary blood glucose monitoring. d. Discuss the need for the patient to actively participate in diabetes management.

B

The nurse is taking a health history from a 29-year-old pregnant patient at the first prenatal visit. The patient reports no personal history of diabetes but has a parent who is diabetic. Which action will the nurse plan to take first? a. Teach the patient about administering regular insulin. b. Schedule the patient for a fasting blood glucose level. c. Discuss an oral glucose tolerance test for the twenty-fourth week of pregnancy. d. Provide teaching about an increased risk for fetal problems with gestational diabetes.

B

Which action should the nurse take after a 36-year-old patient treated with intramuscular glucagon for hypoglycemia regains consciousness? a. Assess the patient for symptoms of hyperglycemia. b. Give the patient a snack of peanut butter and crackers. c. Have the patient drink a glass of orange juice or nonfat milk. d. Administer a continuous infusion of 5% dextrose for 24 hours.

B

Which information will the nurse include when teaching a 50-year-old patient who has type 2 diabetes about glyburide (Micronase, DiaBeta, Glynase)? a. Glyburide decreases glucagon secretion from the pancreas. b. Glyburide stimulates insulin production and release from the pancreas. c. Glyburide should be taken even if the morning blood glucose level is low. d. Glyburide should not be used for 48 hours after receiving IV contrast media.

B

Which laboratory value reported to the nurse by the unlicensed assistive personnel (UAP) indicates the most urgent need for the nurse's assessment of the patient? a. Bedtime glucose of 140 mg/dL b. Noon blood glucose of 52 mg/dL c. Fasting blood glucose of 130 mg/dL d. 2-hr postprandial glucose of 220 mg/dL

B

Which patient action indicates good understanding of the nurse's teaching about administration of aspart (NovoLog) insulin? a. The patient avoids injecting the insulin into the upper abdominal area. b. The patient cleans the skin with soap and water before insulin administration. c. The patient stores the insulin in the freezer after administering the prescribed dose. d. The patient pushes the plunger down while removing the syringe from the injection site.

B

Which phase of acute renal failure signals that glomerular filtration has started to recover? a) Diuretic b) Recovery c) Initiation d) Oliguric

a) Diuretic The oliguric period is accompanied by an increase in the serum concentration of wastes such as urea, creatinine, organic acids, and the electrolytes potassium, phosphorous, and magnesium. The initiation period begins with the initial insult and ends when cellular injury and oliguria develops. The diuretic phase is marked by a gradual increase in urine output, which signals that glomerular filtration has started to recover. The recovery period signals the improvement of renal function and energy level and may take 6 to 12 months.

To monitor for complications in a patient with type 2 diabetes, which tests will the nurse in the diabetic clinic schedule at least annually (select all that apply)? a. Chest x-ray b. Blood pressure c. Serum creatinine d. Urine for microalbuminuria e. Complete blood count (CBC) f. Monofilament testing of the foot

B C D F

A patient with a diagnosis of heart failure has been started on a nitroglycerin patch by his primary care provider. What should this patient be taught to avoid?

B Drugs to treat erectile dysfunction The use of erectile drugs concurrent with nitrates creates a risk of severe hypotension and possibly death.

What should the nurse recognize as an indication for the use of dopamine (Intropin) in the care of a patient with heart failure?

B Hypotension and tachycardia Dopamine is a β-adrenergic agonist whose inotropic action is used for treatment of severe heart failure accompanied by hemodynamic instability.

The home care nurse visits a 73-year-old Hispanic woman with chronic heart failure. Which clinical manifestations, if assessed by the nurse, would indicate acute decompensated heart failure (pulmonary edema)?

B Severe dyspnea and blood-streaked, frothy sputum Clinical manifestations of pulmonary edema include anxiety, pallor, cyanosis, clammy and cold skin, severe dyspnea, use of accessory muscles of respiration, a respiratory rate > 30 breaths per minute, orthopnea, wheezing, and coughing with the production of frothy, blood-tinged sputum.

A female patient with critical limb ischemia has had peripheral artery bypass surgery to improve her circulation. What care should the nurse provide on postoperative day 1? A. Keep the patient on bed rest. B. Assist the patient with walking several times. C. Have the patient sit in the chair several times. D. Place the patient on her side with knees flexed.

B. Assist the patient with walking several times. To avoid blockage of the graft or stent, the patient should walk several times on postoperative day 1 and subsequent days. Having the patient's knees flexed for sitting in a chair or in bed increase the risk of venous thrombosis and may place stress on the suture lines.

67-year-old man with peripheral artery disease is seen in the primary care clinic. Which symptom reported by the patient would indicate to the nurse that the patient is experiencing intermittent claudication? A. Patient complains of chest pain with strenuous activity. B. Patient says muscle leg pain occurs with continued exercise. C. Patient has numbness and tingling of all his toes and both feet. D. Patient states the feet become red if he puts them in a dependent position.

B. Patient says muscle leg pain occurs with continued exercise. Intermittent claudication is an ischemic muscle ache or pain that is precipitated by a consistent level of exercise, resolves within 10 minutes or less with rest, and is reproducible. Angina is the term used to describe chest pain with exertion. Paresthesia is the term used to describe numbness or tingling in the toes or feet. Reactive hyperemia is the term used to describe redness of the foot; if the limb is in a dependent position the term is dependent rubor.

A 62-year-old Hispanic male patient with diabetes mellitus has been diagnosed with peripheral artery disease (PAD). The patient is a smoker and has a history of gout. What should the nurse focus her teaching on to prevent complications for this patient? A. Gender B. Smoking C. Ethnicity D. Co-morbidities

B. Smoking Smoking is the most significant factor for this patient. PAD is a marker of advanced systemic atherosclerosis. Therefore tobacco cessation is essential to reduce PAD progression, CVD events, and mortality. Diabetes mellitus and hyperuricemia are also risk factors. Being male or Hispanic are not risk factors for PAD.

1. After the insertion of an arteriovenous graft (AVG) in the right forearm, a 54-year-old patient complains of pain and coldness of the right fingers. Which action should the nurse take? a. Teach the patient about normal AVG function. b. Remind the patient to take a daily low-dose aspirin tablet. c. Report the patient's symptoms to the health care provider. d. Elevate the patient's arm on pillows to above the heart level.

C

11. A 37-year-old female patient is hospitalized with acute kidney injury (AKI). Which information will be most useful to the nurse in evaluating improvement in kidney function? a. Urine volume b. Creatinine level c. Glomerular filtration rate (GFR) d. Blood urea nitrogen (BUN) level

C

27. A patient with acute kidney injury (AKI) has longer QRS intervals on the electrocardiogram (ECG) than were noted on the previous shift. Which action should the nurse take first? a. Notify the patient's health care provider. b. Document the QRS interval measurement. c. Check the medical record for most recent potassium level. d. Check the chart for the patient's current creatinine level.

C

4. A patient who has acute glomerulonephritis is hospitalized with hyperkalemia. Which information will the nurse monitor to evaluate the effectiveness of the prescribed calcium gluconate IV? a. Urine volume b. Calcium level c. Cardiac rhythm d. Neurologic status

C

6. Which statement by a 62-year-old patient with stage 5 chronic kidney disease (CKD) indicates that the nurse's teaching about management of CKD has been effective? a. "I need to get most of my protein from low-fat dairy products." b. "I will increase my intake of fruits and vegetables to 5 per day." c. "I will measure my urinary output each day to help calculate the amount I can drink." d. "I need to take erythropoietin to boost my immune system and help prevent infection."

C

9. Which menu choice by the patient who is receiving hemodialysis indicates that the nurse's teaching has been successful? a. Split-pea soup, English muffin, and nonfat milk b. Oatmeal with cream, half a banana, and herbal tea c. Poached eggs, whole-wheat toast, and apple juice d. Cheese sandwich, tomato soup, and cranberry juice

C

A 26-year-old female with type 1 diabetes develops a sore throat and runny nose after caring for her sick toddler. The patient calls the clinic for advice about her symptoms and a blood glucose level of 210 mg/dL despite taking her usual glargine (Lantus) and lispro (Humalog) insulin. The nurse advises the patient to: a. use only the lispro insulin until the symptoms are resolved. b. limit intake of calories until the glucose is less than 120 mg/dL. c. monitor blood glucose every 4 hours and notify the clinic if it continues to rise. d. decrease intake of carbohydrates until glycosylated hemoglobin is less than 7%.

C

A 26-year-old patient with diabetes rides a bicycle to and from work every day. Which site should the nurse teach the patient to administer the morning insulin? a. thigh. b. buttock. c. abdomen. d. upper arm.

C

A 48-year-old male patient screened for diabetes at a clinic has a fasting plasma glucose level of 120 mg/dL (6.7 mmol/L). The nurse will plan to teach the patient about a. self-monitoring of blood glucose. b. using low doses of regular insulin. c. lifestyle changes to lower blood glucose. d. effects of oral hypoglycemic medications.

C

A few weeks after an 82-year-old with a new diagnosis of type 2 diabetes has been placed on metformin (Glucophage) therapy and taught about appropriate diet and exercise, the home health nurse makes a visit. Which finding by the nurse is most important to discuss with the health care provider? a. Hemoglobin A1C level is 7.9%. b. Last eye exam was 18 months ago. c. Glomerular filtration rate is decreased. d. Patient has questions about the prescribed diet.

C

A patient who was admitted with diabetic ketoacidosis secondary to a urinary tract infection has been weaned off an insulin drip 30 minutes ago. The patient reports feeling lightheaded and sweaty. Which action should the nurse take first? a. Infuse dextrose 50% by slow IV push. b. Administer 1 mg glucagon subcutaneously. c. Obtain a glucose reading using a finger stick. d. Have the patient drink 4 ounces of orange juice.

C

After change-of-shift report, which patient will the nurse assess first? a. 19-year-old with type 1 diabetes who was admitted with possible dawn phenomenon b. 35-year-old with type 1 diabetes whose most recent blood glucose reading was 230 mg/dL c. 60-year-old with hyperosmolar hyperglycemic syndrome who has poor skin turgor and dry oral mucosa d. 68-year-old with type 2 diabetes who has severe peripheral neuropathy and complains of burning foot pain

C

The health care provider suspects the Somogyi effect in a 50-year-old patient whose 6:00 AM blood glucose is 230 mg/dL. Which action will the nurse teach the patient to take? a. Avoid snacking at bedtime. b. Increase the rapid-acting insulin dose. c. Check the blood glucose during the night d. Administer a larger dose of long-acting insulin.

C

The nurse determines a need for additional instruction when the patient with newly diagnosed type 1 diabetes says which of the following? a. "I can have an occasional alcoholic drink if I include it in my meal plan." b. "I will need a bedtime snack because I take an evening dose of NPH insulin." c. "I can choose any foods, as long as I use enough insulin to cover the calories." d. "I will eat something at meal times to prevent hypoglycemia, even if I am not hungry."

C

The nurse is assessing a 22-year-old patient experiencing the onset of symptoms of type 1 diabetes. Which question is most appropriate for the nurse to ask? a. "Are you anorexic?" b. "Is your urine dark colored?" c. "Have you lost weight lately?" d. "Do you crave sugary drinks?"

C

When a patient who takes metformin (Glucophage) to manage type 2 diabetes develops an allergic rash from an unknown cause, the health care provider prescribes prednisone (Deltasone). The nurse will anticipate that the patient may: a. need a diet higher in calories while receiving prednisone. b. develop acute hypoglycemia while taking the prednisone. c. require administration of insulin while taking prednisone. d. have rashes caused by metformin-prednisone interactions.

C

Which information is most important for the nurse to report to the health care provider before a patient with type 2 diabetes is prepared for a coronary angiogram? a. The patient's most recent HbA1C was 6.5%. b. The patient's admission blood glucose is 128 mg/dL. c. The patient took the prescribed metformin (Glucophage) today. d. The patient took the prescribed captopril (Capoten) this morning.

C

Which statement by a nurse to a patient newly diagnosed with type 2 diabetes is correct? a. Insulin is not used to control blood glucose in patients with type 2 diabetes. b. Complications of type 2 diabetes are less serious than those of type 1 diabetes. c. Changes in diet and exercise may control blood glucose levels in type 2 diabetes. d. Type 2 diabetes is usually diagnosed when the patient is admitted with a hyperglycemic coma.

C

A 54-year-old male patient who had bladder surgery 2 days ago develops acute decompensated heart failure (ADHF) with severe dyspnea. Which action by the nurse would be indicated first?

C Assist the patient to a sitting position with arms on the overbed table. The nurse should place the patient with ADHF in a high Fowler's position with the feet horizontal in the bed or dangling at the bedside.

A patient with hypomagnesemia is undergoing treatment with a continuous IV infusion of magnesium. Which supply does the nurse ensure is at the bedside of this patient? 10% glucose Sodium chloride Calcium gluconate Potassium chloride

Calcium gluconate Calcium gluconate is indicated in the treatment of hypermagnesemia, which can occur with continuous IV infusion of magnesium. Because hypermagnesemia causes neuromuscular blockade, these patients are at risk for paralysis of respiratory muscles. Calcium gluconate can counteract the neuromuscular blocking actions of magnesium.

A 28-year-old male patient with type 1 diabetes reports how he manages his exercise and glucose control. Which behavior indicates that the nurse should implement additional teaching? a. The patient always carries hard candies when engaging in exercise. b. The patient goes for a vigorous walk when his glucose is 200 mg/dL. c. The patient has a peanut butter sandwich before going for a bicycle ride. d. The patient increases daily exercise when ketones are present in the urine.

D

A hospitalized diabetic patient received 38 U of NPH insulin at 7:00 AM. At 1:00 PM, the patient has been away from the nursing unit for 2 hours, missing the lunch delivery while awaiting a chest x-ray. To prevent hypoglycemia, the best action by the nurse is to: a. save the lunch tray for the patient's later return to the unit. b. ask that diagnostic testing area staff to start a 5% dextrose IV. c. send a glass of milk or orange juice to the patient in the diagnostic testing area. d. request that if testing is further delayed, the patient be returned to the unit to eat.

D

A patient with type 2 diabetes is scheduled for a follow-up visit in the clinic several months from now. Which test will the nurse schedule to evaluate the effectiveness of treatment for the patient? a. Urine dipstick for glucose b. Oral glucose tolerance test c. Fasting blood glucose level d. Glycosylated hemoglobin level

D

The nurse has been teaching a patient with type 2 diabetes about managing blood glucose levels and taking glipizide (Glucotrol). Which patient statement indicates a need for additional teaching? a. "If I overeat at a meal, I will still take the usual dose of medication." b. "Other medications besides the Glucotrol may affect my blood sugar." c. "When I am ill, I may have to take insulin to control my blood sugar." d. "My diabetes won't cause complications because I don't need insulin."

D

Which action by a patient indicates that the home health nurse's teaching about glargine and regular insulin has been successful? a. The patient administers the glargine 30 minutes before each meal. b. The patient's family prefills the syringes with the mix of insulins weekly. c. The patient draws up the regular insulin and then the glargine in the same syringe. d. The patient disposes of the open vials of glargine and regular insulin after 4 weeks.

D

Which finding indicates a need to contact the health care provider before the nurse administers metformin (Glucophage)? a. The patient's blood glucose level is 174 mg/dL. b. The patient has gained 2 lb (0.9 kg) since yesterday. c. The patient is scheduled for a chest x-ray in an hour. d. The patient's blood urea nitrogen (BUN) level is 52 mg/dL.

D

The nurse is administering a dose of digoxin (Lanoxin) to a patient with heart failure (HF). The nurse would become concerned with the possibility of digitalis toxicity if the patient reported which symptom(s)? D Anorexia and nausea

D Anorexia and nausea Anorexia, nausea, vomiting, blurred or yellow vision, and cardiac dysrhythmias are all signs of digitalis toxicity.

After having an MI, the nurse notes the patient has jugular venous distention, gained weight, developed peripheral edema, and has a heart rate of 108/minute. What should the nurse suspect is happening?

D Right-sided HF An MI is a primary cause of heart failure. The jugular venous distention, weight gain, peripheral edema, and increased heart rate are manifestations of right-sided heart failure.

A (A disease of the airways characterized by destruction of the walls of overdistended alveoli Explanation: Emphysema is a category of chronic obstructive pulmonary disease (COPD). In emphysema, impaired oxygen and carbon dioxide exchange results from destruction of the walls of over-distended alveoli. Emphysema is a pathologic term that describes an abnormal distention of the airspaces beyond the terminal bronchioles and destruction of the walls of the alveoli. Also, a chronic inflammatory response may induce disruption of the parenchymal tissues. Asthma has a clinical outcome of airflow obstruction. Bronchitis includes the presence of cough and sputum production for at least a combined total of 2 to 3 months in each of 2 consecutive years. Bronchiectasis is a condition of chronic dilatation of a bronchus or bronchi.)

Emphysema is described by which of the following statements? a) A disease of the airways characterized by destruction of the walls of overdistended alveoli b) Presence of cough and sputum production for at least a combined total of 2 to 3 months in each of 2 consecutive years c) Chronic dilatation of a bronchus or bronchi d) A disease that results in a common clinical outcome of reversible airflow obstruction

B (A disease of the airways characterized by destruction of the walls of overdistended alveoli Explanation: Emphysema is a category of COPD. Asthma has a clinical outcome of airflow obstruction. Bronchitis includes the presence of cough and sputum production for at least a combined total of 2 to 3 months in each of two consecutive years. Bronchiectasis is a condition of chronic dilatation of a bronchus or bronchi.)

Emphysema is described by which of the following statements? a) A disease that results in a common clinical outcome of reversible airflow obstruction b) A disease of the airways characterized by destruction of the walls of overdistended alveoli c) Presence of cough and sputum production for at least a combined total of 2 to 3 months in each of two consecutive years d) Chronic dilatation of a bronchus or bronchi

A client presents with anorexia, nausea and vomiting, deep bone pain, and constipation. The following are the client's laboratory values. Na + 130 mEq/L K + 4.6 mEq/L Cl - 94 mEq/L Mg ++ 2.8 mg/dL Ca ++ 13 mg/dL Which of the following alterations is consistent with the client's findings? a) Hyperkalemia b) Hyponatremia c) Hypermagnesemia d) Hypercalcemia

Hypercalcemia

The nurse monitors the patient with type 1 diabetes who is taking growth hormone for which side effect? Weight gain Hypertension Hyperglycemia Bone demineralization

Hyperglycemia Growth hormone increases blood glucose. In nondiabetic patients, this increased blood glucose stimulates release of insulin to maintain the blood glucose level within the normal range. Because of the lack of insulin in patients with type 1 diabetes, the growth hormone causes hyperglycemia.

Which laboratory value places the patient at greatest risk of digoxin toxicity? Hypokalemia Hyponatremia Hypocalcemia Hypomagnesemia

Hypokalemia Hypokalemia increases the risk of digoxin toxicity. Potassium has a major role in nerve impulse conduction and maintaining the electrical excitability of muscles.

A (Stage III patients demonstrate an FEV less than 30% with respiratory failure or clinical signs of right heart failure. Stage II patients demonstrate an FEV of 30% to 80%. Stage I is mild COPD with an FEV less than 70%. Stage 0 is characterized by normal spirometry)

In which stage of COPD is the forced expiratory volume (FEV) less than 30%? a) III b) II c) I d) 0

The nurse understands that the serum level of potassium is regulated by which organ? Liver Lungs Kidneys Small intestine

Kidneys The kidneys, largely under the control of aldosterone from the adrenal cortex, retain potassium while losing sodium and water.

A client hospitalized for treatment of a pulmonary embolism develops respiratory alkalosis. Which clinical findings commonly accompany respiratory alkalosis? a) Nausea or vomiting b) Abdominal pain or diarrhea c) Hallucinations or tinnitus d) Light-headedness or paresthesia

Light-headedness or paresthesia

A client has the following arterial blood gas (ABG) values: pH, 7.12; partial pressure of arterial carbon dioxide (PaCO2), 40 mm Hg; and bicarbonate (HCO3-), 15 mEq/L. These ABG values suggest which disorder? a) Metabolic acidosis b) Respiratory alkalosis c) Metabolic alkalosis d) Respiratory acidosis

Metabolic acidosis

A client in the emergency department reports that he has been vomiting excessively for the past 2 days. His arterial blood gas analysis shows a pH of 7.50, partial pressure of arterial carbon dioxide (PaCO2) of 43 mm Hg, partial pressure of arterial oxygen (PaO2) of 75 mm Hg, and bicarbonate (HCO3-) of 42 mEq/L. Based on these findings, the nurse documents that the client is experiencing which type of acid-base imbalance? a) Respiratory acidosis b) Respiratory alkalosis c) Metabolic alkalosis d) Metabolic acidosis

Metabolic alkalosis

A patient admitted with heart failure appears very anxious and complains of shortness of breath. Which nursing actions would be appropriate to alleviate this patient's anxiety (select all that apply)? A Administer ordered morphine sulfate. B Position patient in a semi-Fowler's position. D Instruct patient on the use of relaxation techniques. E Use a calm, reassuring approach while talking to patient.

Morphine sulfate reduces anxiety and may assist in reducing dyspnea. The patient should be positioned in semi-Fowler's position to improve ventilation that will reduce anxiety. Relaxation techniques and a calm reassuring approach will also serve to reduce anxiety.

The nurse monitors the patient with hypermagnesemia for what clinical manifestation? Seizures Disorientation Muscle weakness Hyperactive reflexes

Muscle weakness Clinical manifestations of hypermagnesemia include increasing muscle weakness related to neuromuscular blockade. Depression of respiration can occur. Additionally, hypotension, sedation, and ECG changes are associated with elevated magnesium levels.

The calcium level of the blood is regulated by which mechanism? a) Parathyroid hormone (PTH) b) Androgens c) Adrenal gland d) Thyroid hormone (TH)

Parathyroid hormone (PTH)

A client with a suspected overdose of an unknown drug is admitted to the emergency department. Arterial blood gas values indicate respiratory acidosis. What should the nurse do first? a) Monitor the client's heart rhythm. b) Prepare to assist with ventilation. c) Obtain a urine specimen for drug screening. d) Prepare for gastric lavage.

Prepare to assist with ventilation.

Which ECG changes would the nurse expect to observe in a patient with hyperkalemia? Widened QRS Inverted P wave Inverted T waves Prolonged PR interval

Prolonged PR interval An increased serum potassium level is associated with cardiac changes, including a prolonged PR interval. These changes are attributed to disruption of the electrical activity in the heart.

The nurse notes that a patient is hyperventilating and assesses him for which acid-base imbalance? Respiratory acidosis Respiratory alkalosis Metabolic acidosis Metabolic alkalosis

Respiratory alkalosis Hyperventilation leads to the blowing off of carbon dioxide, resulting in respiratory alkalosis.

Which of the following arterial blood gas results would be consistent with metabolic alkalosis? a) pH 7.26 b) Serum bicarbonate of 21 mEq/L c) pH 7.30 d) Serum bicarbonate of 28 mEq/L

Serum bicarbonate of 28 mEq/L

The nurse prepares to administer digoxin (Lanoxin) 0.125 mg to an 82-year-old man admitted with influenza and a history of chronic heart failure. What should the nurse assess before giving the medication? C Serum potassium level

Serum potassium should be monitored because hypokalemia increases the risk for digoxin toxicity.

The nurse notes that a patient's serum potassium level is 2.8 mEq/L. The nurse monitors her for which clinical manifestation as consistent with this laboratory value? Seizures Paresthesias Loss of consciousness Skeletal muscle weakness

Skeletal muscle weakness Hypokalemia is manifested by muscle weakness that may lead to paralysis with continued decreases in potassium. This electrolyte disorder can also lead to fatal dysthythmias.

The nurse working on the heart failure unit knows that teaching an older female patient with newly diagnosed heart failure is effective when the patient states that

Tc. all the clinic if her weight goes from 124 to 128 pounds in a week. eaching for a patient with heart failure includes information about the need to weigh daily and notify the health care provider about an increase of 3 pounds in 2 days or 3 to 5 pounds in a week.

The nurse is instructing a patient with recurrent hyperkalemia about following a potassium-restricted diet. Which of the following patient statements indicates the need for additional instruction? a) "Bananas have a lot of potassium in them, I'll stop buying them." b) "I need to check if my cola beverage has potassium in it." c) "I will not salt my food, instead I'll use salt substitute." d) "I'll drink cranberry juice with my breakfast instead of coffee."

"I will not salt my food, instead I'll use salt substitute."

E (severe COPD. Explanation: Stage III is severe COPD. Stage 0 is at risk for COPD. Stage I is mild COPD. Stage II is moderate COPD. Stage IV is very severe COPD.)

The classification of Stage III of COPD is defined as a) moderate COPD. b) at risk for COPD. c) very severe COPD. d) mild COPD. e) severe COPD.

A nurse is conducting an initial assessment on a client with possible tuberculosis. Which assessment finding indicates a risk factor for tuberculosis? a) The client has never traveled outside of the country. b) The client works in a health care insurance office. c) The client sees his physician for a check-up yearly. d) The client had a liver transplant 2 years ago.

The client had a liver transplant 2 years ago.

The patient with chronic heart failure is being discharged from the hospital. What information should the nurse emphasize in the patient's discharge teaching to prevent progression of the disease to ADHF?

The goal for the patient with chronic HF is to avoid exacerbations and hospitalization.

The nurse is preparing to administer digoxin to a patient with heart failure. In preparation, laboratory results are reviewed with the following findings: sodium 139 mEq/L, potassium 5.6 mEq/L, chloride 103 mEq/L, and glucose 106 mg/dL. What should the nurse do next? B Withhold the dose and report the potassium level.

The normal potassium level is 3.5 to 5.0 mEq/L.

) are a first-line therapy for this type of angina. Lipid-lowering drugs help reduce atherosclerosis (i.e., plaque formation), and β-adrenergic blockers decrease sympathetic stimulation of the heart (i.e., palpitations). Medications or activities that increase myocardial contractility will increase the incidence of angina by increasing oxygen demand.

...

D (Exhales hard and fast with a single blow Explanation: To use a peak flow meter, the client stands. Then the client takes a deep breath and exhales hard and fast with a single blow. The client repeats this twice and records a "personal best" in an asthma diary.)

The nurse has instructed the client to use a peak flow meter. The nurse evaluates client learning as satisfactory when the client a) Records in a diary the number achieved after one breath b) Inhales deeply and holds the breath c) Sits in a straight-back chair and leans forward d) Exhales hard and fast with a single blow

A, B, D (Explanation: The nurse also assesses the patient's skin turgor for signs of dehydration. Fluid intake is essential to combat dehydration, to loosen secretions, and to facilitate expectoration.)

The nurse is assigned to care for a patient in the ICU who is diagnosed with status asthmaticus. Why does the nurse include fluid intake as being an important aspect of the plan of care? (Select all that apply.) a) To combat dehydration b) To facilitate expectoration c) To assist with the effectiveness of the corticosteroids d) To loosen secretions e) To relieve bronchospasm

C (Providing sufficient oxygen to improve oxygenation Explanation: The main objective in treating patients with hypoxemia and hypercapnia is to give sufficient oxygen to improve oxygenation.)

The nurse is assigned to care for a patient with COPD with hypoxemia and hypercapnia. When planning care for this patient, what does the nurse understand is the main goal of treatment? a) Avoiding the use of oxygen to decrease the hypoxic drive b) Increasing pH c) Providing sufficient oxygen to improve oxygenation d) Monitoring the pulse oximetry to assess need for early intervention when PCO2 levels rise

C (90% Explanation: The goal of supplemental oxygen therapy is to increase the baseline resting partial arterial pressure of oxygen (PaO2) to at least 60 mm Hg at sea level and arterial oxygen saturation (SaO2) to at least 90%.)

The nurse is caring for a patient with COPD. The patient is receiving oxygen therapy via nasal cannula. The nurse understands that the goal of oxygen therapy is to maintain the patient's SaO2 level at or above what percent? a) 30% b) 70% c) 90% d) 50%

C (The patient should take the medication an hour before meals or 2 hours after a meal. Explanation: The nurse should instruct the patient to take the leukotriene receptor antagonist at least 1 hour before meals or 2 hours after meals.)

The nurse is instructing the patient with asthma in the use of a newly prescribed leukotriene receptor antagonist. What should the nurse be sure to include in the education? a) The patient should take the medication with a small amount of liquid. b) The patient should take the medication with meals since it may cause nausea. c) The patient should take the medication an hour before meals or 2 hours after a meal. d) The patient should take the medication separately without other medications.

A (Respiratory acidosis. Explanation: Decreased carbon dioxide elimination results in increased carbon dioxide tension (hypercapnia), which leads to respiratory acidosis and chronic respiratory failure.)

The nurse, caring for a patient with emphysema, understands that airflow limitations are not reversible. The end result of deterioration is: a) Respiratory acidosis. b) Diminished alveolar surface area. c) Hypercapnia resulting from decreased carbon dioxide elimination. d) Hypoxemia secondary to impaired oxygen diffusion.

A, C, D (Correct Because of the chronic inflammation and the body's attempts to repair it, changes and narrowing occur in the airways. In the peripheral airways, inflammation causes thickening of the airway wall, peribronchial fibrosis, exudate in the airway, and overall airway narrowing (obstructive bronchiolitis). The airways are actually moist, not dry. In the proximal airways, changes include increased goblet cells and enlarged submucosal glands, both of which lead to hypersecretion of mucus.)

The nursing student recalls that the underlying pathophysiology of chronic obstructive pulmonary disease (COPD) includes the following components: (Select all that apply.) a) Mucus secretions block airways. b) Dry airways obstruct airflow. c) Inflamed airways obstruct airflow. d) Overinflated alveoli impair gas exchange.

10. A 15-year-old child is brought to the emergency department with symptoms of hyperglycemia and is subsequently diagnosed with diabetes. Based on the fact that the child's pancreatic beta cells are being destroyed, the patient would be diagnosed with what type of diabetes? A) Type 1 diabetes B) Type 2 diabetes C) Non-insulin-dependent diabetes D) Prediabetes

: A Feedback: Beta cell destruction is the hallmark of type 1 diabetes. Non-insulin-dependent diabetes is synonymous with type 2 diabetes, which involves insulin resistance and impaired insulin secretion, but not beta cell destruction. Prediabetes is characterized by normal glucose metabolism, but a previous history of hyperglycemia, often during illness or pregnancy.

2. A patient presents to the clinic complaining of symptoms that suggest diabetes. What criteria would support checking blood levels for the diagnosis of diabetes? A) Fasting plasma glucose greater than or equal to 126 mg/dL B) Random plasma glucose greater than 150 mg/dL C) Fasting plasma glucose greater than 116 mg/dL on 2 separate occasions D) Random plasma glucose greater than 126 mg/dL

: A Feedback: Criteria for the diagnosis of diabetes include symptoms of diabetes plus random plasma glucose greater than or equal to 200 mg/dL, or a fasting plasma glucose greater than or equal to 126 mg/dL.

26. A patient with a longstanding diagnosis of type 1 diabetes has a history of poor glycemic control. The nurse recognizes the need to assess the patient for signs and symptoms of peripheral neuropathy. Peripheral neuropathy constitutes a risk for what nursing diagnosis? A) Infection B) Acute pain C) Acute confusion D) Impaired urinary elimination

: A Feedback: Decreased sensations of pain and temperature place patients with neuropathy at increased risk for injury and undetected foot infections. The neurologic changes associated with peripheral neuropathy do not normally result in pain, confusion, or impairments in urinary function.

30. A nurse is conducting a class on how to self-manage insulin regimens. A patient asks how long a vial of insulin can be stored at room temperature before it "goes bad." What would be the nurse's best answer? A) "If you are going to use up the vial within 1 month it can be kept at room temperature." B) "If a vial of insulin will be used up within 21 days, it may be kept at room temperature." C) "If a vial of insulin will be used up within 2 weeks, it may be kept at room temperature." D) "If a vial of insulin will be used up within 1 week, it may be kept at room temperature."

: A Feedback: If a vial of insulin will be used up within 1 month, it may be kept at room temperature.

27. A patient has been brought to the emergency department by paramedics after being found unconscious. The patient's Medic Alert bracelet indicates that the patient has type 1 diabetes and the patient's blood glucose is 22 mg/dL (1.2 mmol/L). The nurse should anticipate what intervention? A) IV administration of 50% dextrose in water B) Subcutaneous administration of 10 units of Humalog C) Subcutaneous administration of 12 to 15 units of regular insulin D) IV bolus of 5% dextrose in 0.45% NaCl

: A Feedback: In hospitals and emergency departments, for patients who are unconscious or cannot swallow, 25 to 50 mL of 50% dextrose in water (D50W) may be administered IV for the treatment of hypoglycemia. Five percent dextrose would be inadequate and insulin would exacerbate the patient's condition.

34. A patient has been living with type 2 diabetes for several years, and the nurse realizes that the patient is likely to have minimal contact with the health care system. In order to ensure that the patient maintains adequate blood sugar control over the long term, the nurse should recommend which of the following? A) Participation in a support group for persons with diabetes B) Regular consultation of websites that address diabetes management C) Weekly telephone "check-ins" with an endocrinologist D) Participation in clinical trials relating to antihyperglycemics

: A Feedback: Participation in support groups is encouraged for patients who have had diabetes for many years as well as for those who are newly diagnosed. This is more interactive and instructive than simply consulting websites. Weekly telephone contact with an endocrinologist is not realistic in most cases. Participation in research trials may or may not be beneficial and appropriate, depending on patients' circumstances.

28. A diabetic nurse is working for the summer at a camp for adolescents with diabetes. When providing information on the prevention and management of hypoglycemia, what action should the nurse promote? A) Always carry a form of fast-acting sugar. B) Perform exercise prior to eating whenever possible. C) Eat a meal or snack every 8 hours. D) Check blood sugar at least every 24 hours.

: A Feedback: The following teaching points should be included in information provided to the patient on how to prevent hypoglycemia: Always carry a form of fast-acting sugar, increase food prior to exercise, eat a meal or snack every 4 to 5 hours, and check blood sugar regularly.

7. A diabetes nurse educator is teaching a group of patients with type 1 diabetes about "sick day rules." What guideline applies to periods of illness in a diabetic patient? A) Do not eliminate insulin when nauseated and vomiting. B) Report elevated glucose levels greater than 150 mg/dL. C) Eat three substantial meals a day, if possible. D) Reduce food intake and insulin doses in times of illness.

: A Feedback: The most important issue to teach patients with diabetes who become ill is not to eliminate insulin doses when nausea and vomiting occur. Rather, they should take their usual insulin or oral hypoglycemic agent dose, then attempt to consume frequent, small portions of carbohydrates. In general, blood sugar levels will rise but should be reported if they are greater than 300 mg/dL.

18. A diabetic educator is discussing "sick day rules" with a newly diagnosed type 1 diabetic. The educator is aware that the patient will require further teaching when the patient states what? A) "I will not take my insulin on the days when I am sick, but I will certainly check my blood sugar every 2 hours." B) "If I cannot eat a meal, I will eat a soft food such as soup, gelatin, or pudding six to eight times a day." C) "I will call the doctor if I am not able to keep liquids in my body due to vomiting or diarrhea." D) "I will call the doctor if my blood sugar is over 300 mg/dL or if I have ketones in my urine."

: A Feedback: The nurse must explanation the "sick day rules" again to the patient who plans to stop taking insulin when sick. The nurse should emphasize that the patient should take insulin agents as usual and test one's blood sugar and urine ketones every 3 to 4 hours. In fact, insulin-requiring patients may need supplemental doses of regular insulin every 3 to 4 hours. The patient should report elevated glucose levels (greater than 300 mg/dL or as otherwise instructed) or urine ketones to the physician. If the patient is not able to eat normally, the patient should be instructed to substitute soft foods such a gelatin, soup, and pudding. If vomiting, diarrhea, or fever persists, the patient should have an intake of liquids every 30 to 60 minutes to prevent dehydration.

22. A nurse is caring for a patient newly diagnosed with type 1 diabetes. The nurse is educating the patient about self-administration of insulin in the home setting. The nurse should teach the patient to do which of the following? A) Avoid using the same injection site more than once in 2 to 3 weeks. B) Avoid mixing more than one type of insulin in a syringe. C) Cleanse the injection site thoroughly with alcohol prior to injecting. D) Inject at a 45º angle.

: A Feedback: To prevent lipodystrophy, the patient should try not to use the same site more than once in 2 to 3 weeks. Mixing different types of insulin in a syringe is acceptable, within specific guidelines, and the needle is usually inserted at a 90º angle. Cleansing the injection site with alcohol is optional.

12. An occupational health nurse is screening a group of workers for diabetes. What statement should the nurse interpret as suggestive of diabetes? A) "I've always been a fan of sweet foods, but lately I'm turned off by them." B) "Lately, I drink and drink and can't seem to quench my thirst." C) "No matter how much sleep I get, it seems to take me hours to wake up." D) "When I went to the washroom the last few days, my urine smelled odd."

: B Feedback: Classic clinical manifestations of diabetes include the "three Ps": polyuria, polydipsia, and polyphagia. Lack of interest in sweet foods, fatigue, and foul-smelling urine are not suggestive of diabetes.

23. A patient with type 2 diabetes achieves adequate glycemic control through diet and exercise. Upon being admitted to the hospital for a cholecystectomy, however, the patient has required insulin injections on two occasions. The nurse would identify what likely cause for this short-term change in treatment? A) Alterations in bile metabolism and release have likely caused hyperglycemia. B) Stress has likely caused an increase in the patient's blood sugar levels. C) The patient has likely overestimated her ability to control her diabetes using nonpharmacologic measures. D) The patient's volatile fluid balance surrounding surgery has likely caused unstable blood sugars.

: B Feedback: During periods of physiologic stress, such as surgery, blood glucose levels tend to increase, because levels of stress hormones (epinephrine, norepinephrine, glucagon, cortisol, and growth hormone) increase. The patient's need for insulin is unrelated to the action of bile, the patient's overestimation of previous blood sugar control, or fluid imbalance.

39. A diabetic patient calls the clinic complaining of having a "flu bug." The nurse tells him to take his regular dose of insulin. What else should the nurse tell the patient? A) "Make sure to stick to your normal diet." B) "Try to eat small amounts of carbs, if possible." C) "Ensure that you check your blood glucose every hour." D) "For now, check your urine for ketones every 8 hours."

: B Feedback: For prevention of DKA related to illness, the patient should attempt to consume frequent small portions of carbohydrates (including foods usually avoided, such as juices, regular sodas, and gelatin). Drinking fluids every hour is important to prevent dehydration. Blood glucose and urine ketones must be assessed every 3 to 4 hours.

33. A patient with a history of type 1 diabetes has just been admitted to the critical care unit (CCU) for diabetic ketoacidosis. The CCU nurse should prioritize what assessment during the patient's initial phase of treatment? A) Monitoring the patient for dysrhythmias B) Maintaining and monitoring the patient's fluid balance C) Assessing the patient's level of consciousness D) Assessing the patient for signs and symptoms of venous thromboembolism

: B Feedback: In addition to treating hyperglycemia, management of DKA is aimed at correcting dehydration, electrolyte loss, and acidosis before correcting the hyperglycemia with insulin. The nurse should monitor the patient for dysrhythmias, decreased LOC and VTE, but restoration and maintenance of fluid balance is the highest priority.

8. The nurse is discussing macrovascular complications of diabetes with a patient. The nurse would address what topic during this dialogue? A) The need for frequent eye examinations for patients with diabetes B) The fact that patients with diabetes have an elevated risk of myocardial infarction C) The relationship between kidney function and blood glucose levels D) The need to monitor urine for the presence of albumin

: B Feedback: Myocardial infarction and stroke are considered macrovascular complications of diabetes, while the effects on vision and renal function are considered to be microvascular.

16. A nurse is caring for a patient with type 1 diabetes who is being discharged home tomorrow. What is the best way to assess the patient's ability to prepare and self-administer insulin? A) Ask the patient to describe the process in detail. B) Observe the patient drawing up and administering the insulin. C) Provide a health education session reviewing the main points of insulin delivery. D) Review the patient's first hemoglobin A1C result after discharge.

: B Feedback: Nurses should assess the patient's ability to perform diabetes related self-care as soon as possible during the hospitalization or office visit to determine whether the patient requires further diabetes teaching. While consulting a home care nurse is beneficial, an initial assessment should be performed during the hospitalization or office visit. Nurses should directly observe the patient performing the skills such as insulin preparation and infection, blood glucose monitoring, and foot care. Simply questioning the patient about these skills without actually observing performance of the skill is not sufficient. Further education does not guarantee learning.

5. A medical nurse is caring for a patient with type 1 diabetes. The patient's medication administration record includes the administration of regular insulin three times daily. Knowing that the patient's lunch tray will arrive at 11:45, when should the nurse administer the patient's insulin? A) 10:45 B) 11:15 C) 11:45 D) 11:50

: B Feedback: Regular insulin is usually administered 20-30 min before a meal. Earlier administration creates a risk for hypoglycemia; later administration creates a risk for hyperglycemia.

36. A diabetes nurse is assessing a patient's knowledge of self-care skills. What would be the most appropriate way for the educator to assess the patient's knowledge of nutritional therapy in diabetes? A) Ask the patient to describe an optimally healthy meal. B) Ask the patient to keep a food diary and review it with the nurse. C) Ask the patient's family what he typically eats. D) Ask the patient to describe a typical day's food intake.

: B Feedback: Reviewing the patient's actual food intake is the most accurate method of gauging the patient's diet.

6. A patient has just been diagnosed with type 2 diabetes. The physician has prescribed an oral antidiabetic agent that will inhibit the production of glucose by the liver and thereby aid in the control of blood glucose. What type of oral antidiabetic agent did the physician prescribe for this patient? A) A sulfonylurea B) A biguanide C) A thiazolidinedione D) An alpha glucosidase inhibitor

: B Feedback: Sulfonylureas exert their primary action by directly stimulating the pancreas to secrete insulin and therefore require a functioning pancreas to be effective. Biguanides inhibit the production of glucose by the liver and are in used in type 2 diabetes to control blood glucose levels. Thiazolidinediones enhance insulin action at the receptor site without increasing insulin secretion from the beta cells of the pancreas. Alpha glucosidase inhibitors work by delaying the absorption of glucose in the intestinal system, resulting in a lower postprandial blood glucose level.

17. An elderly patient comes to the clinic with her daughter. The patient is a diabetic and is concerned about foot care. The nurse goes over foot care with the patient and her daughter as the nurse realizes that foot care is extremely important. Why would the nurse feel that foot care is so important to this patient? A) An elderly patient with foot ulcers experiences severe foot pain due to the diabetic polyneuropathy. B) Avoiding foot ulcers may mean the difference between institutionalization and continued independent living. C) Hypoglycemia is linked with a risk for falls; this risk is elevated in older adults with diabetes. D) Oral antihyperglycemics have the possible adverse effect of decreased circulation to the lower extremities.

: B Feedback: The nurse recognizes that providing information on the long-term complications—especially foot and eye problems—associated with diabetes is important. Avoiding amputation through early detection of foot ulcers may mean the difference between institutionalization and continued independent living for the elderly person with diabetes. While the nurse recognizes that hypoglycemia is a dangerous situation and may lead to falls, hypoglycemia is not directly connected to the importance of foot care. Decrease in circulation is related to vascular changes and is not associated with drugs administered for diabetes.

40. A patient is brought to the emergency department by the paramedics. The patient is a type 2 diabetic and is experiencing HHS. The nurse should identify what components of HHS? Select all that apply. A) Leukocytosis B) Glycosuria C) Dehydration D) Hypernatremia E) Hyperglycemia

: B, C, D, E Feedback: In HHS, persistent hyperglycemia causes osmotic diuresis, which results in losses of water and electrolytes. To maintain osmotic equilibrium, water shifts from the intracellular fluid space to the extracellular fluid space. With glycosuria and dehydration, hypernatremia and increased osmolarity occur. Leukocytosis does not take place.

3. A patient newly diagnosed with type 2 diabetes is attending a nutrition class. What general guideline would be important to teach the patients at this class? A) Low fat generally indicates low sugar. B) Protein should constitute 30% to 40% of caloric intake. C) Most calories should be derived from carbohydrates. D) Animal fats should be eliminated from the diet.

: C Feedback: Currently, the ADA and the Academy of Nutrition and Dietetics (formerly the American Dietetic Association) recommend that for all levels of caloric intake, 50% to 60% of calories should be derived from carbohydrates, 20% to 30% from fat, and the remaining 10% to 20% from protein.Low fat does not automatically mean low sugar. Dietary animal fat does not need to be eliminated from the diet.

4. A nurse is providing health education to an adolescent newly diagnosed with type 1 diabetes mellitus and her family. The nurse teaches the patient and family that which of the following nonpharmacologic measures will decrease the body's need for insulin? A) Adequate sleep B) Low stimulation C) Exercise D) Low-fat diet

: C Feedback: Exercise lowers blood glucose, increases levels of HDLs, and decreases total cholesterol and triglyceride levels. Low fat intake and low levels of stimulation do not reduce a patient's need for insulin. Adequate sleep is beneficial in reducing stress, but does not have an effect that is pronounced as that of exercise.

35. A patient with type 1 diabetes mellitus is seeing the nurse to review foot care. What would be a priority instruction for the nurse to give the patient? A) Examine feet weekly for redness, blisters, and abrasions. B) Avoid the use of moisturizing lotions. C) Avoid hot-water bottles and heating pads. D) Dry feet vigorously after each bath.

: C Feedback: High-risk behaviors, such as walking barefoot, using heating pads on the feet, wearing open-toed shoes, soaking the feet, and shaving calluses, should be avoided. Socks should be worn for warmth. Feet should be examined each day for cuts, blisters, swelling, redness, tenderness, and abrasions. Lotion should be applied to dry feet but never between the toes. After a bath, the patient should gently, not vigorously, pat feet dry to avoid injury.

20. A 28-year-old pregnant woman is spilling sugar in her urine. The physician orders a glucose tolerance test, which reveals gestational diabetes. The patient is shocked by the diagnosis, stating that she is conscientious about her health, and asks the nurse what causes gestational diabetes. The nurse should explain that gestational diabetes is a result of what etiologic factor? A) Increased caloric intake during the first trimester B) Changes in osmolality and fluid balance C) The effects of hormonal changes during pregnancy D) Overconsumption of carbohydrates during the first two trimesters

: C Feedback: Hyperglycemia and eventual gestational diabetes develops during pregnancy because of the secretion of placental hormones, which causes insulin resistance. The disease is not the result of food intake or changes in osmolality.

32. A student with diabetes tells the school nurse that he is feeling nervous and hungry. The nurse assesses the child and finds he has tachycardia and is diaphoretic with a blood glucose level of 50 mg/dL (2.8 mmol/L). What should the school nurse administer? A) A combination of protein and carbohydrates, such as a small cup of yogurt B) Two teaspoons of sugar dissolved in a cup of apple juice C) Half of a cup of juice, followed by cheese and crackers D) Half a sandwich with a protein-based filling

: C Feedback: Initial treatment for hypoglycemia is 15 g concentrated carbohydrate, such as two or three glucose tablets, 1 tube glucose gel, or 0.5 cup juice. After initial treatment, the nurse should follow with a snack including starch and protein, such as cheese and crackers, milk and crackers, or half of a sandwich. It is unnecessary to add sugar to juice, even it if is labeled as unsweetened juice, because the fruit sugar in juice contains enough simple carbohydrate to raise the blood glucose level and additional sugar may result in a sharp rise in blood sugar that will last for several hours.

13. A diabetes educator is teaching a patient about type 2 diabetes. The educator recognizes that the patient understands the primary treatment for type 2 diabetes when the patient states what? A) "I read that a pancreas transplant will provide a cure for my diabetes." B) "I will take my oral antidiabetic agents when my morning blood sugar is high." C) "I will make sure to follow the weight loss plan designed by the dietitian." D) "I will make sure I call the diabetes educator when I have questions about my insulin."

: C Feedback: Insulin resistance is associated with obesity; thus the primary treatment of type 2 diabetes is weight loss. Oral antidiabetic agents may be added if diet and exercise are not successful in controlling blood glucose levels. If maximum doses of a single category of oral agents fail to reduce glucose levels to satisfactory levels, additional oral agents may be used. Some patients may require insulin on an ongoing basis or on a temporary basis during times of acute psychological stress, but it is not the central component of type 2 treatment. Pancreas transplantation is associated with type 1 diabetes.

1. A patient with type 1 diabetes has told the nurse that his most recent urine test for ketones was positive. What is the nurse's most plausible conclusion based on this assessment finding? A) The patient should withhold his next scheduled dose of insulin. B) The patient should promptly eat some protein and carbohydrates. C) The patient's insulin levels are inadequate. D) The patient would benefit from a dose of metformin (Glucophage).

: C Feedback: Ketones in the urine signal that there is a deficiency of insulin and that control of type 1 diabetes is deteriorating. Withholding insulin or eating food would exacerbate the patient's ketonuria. Metformin will not cause short-term resolution of hyperglycemia.

24. A physician has explained to a patient that he has developed diabetic neuropathy in his right foot. Later that day, the patient asks the nurse what causes diabetic neuropathy. What would be the nurse's best response? A) "Research has shown that diabetic neuropathy is caused by fluctuations in blood sugar that have gone on for years." B) "The cause is not known for sure but it is thought to have something to do with ketoacidosis." C) "The cause is not known for sure but it is thought to involve elevated blood glucose levels over a period of years." D) "Research has shown that diabetic neuropathy is caused by a combination of elevated glucose levels and elevated ketone levels."

: C Feedback: The etiology of neuropathy may involve elevated blood glucose levels over a period of years. High blood sugars (rather than fluctuations or variations in blood sugars) are thought to be responsible. Ketones and ketoacidosis are not direct causes of neuropathies.

19. Which of the following patients with type 1 diabetes is most likely to experience adequate glucose control? A) A patient who skips breakfast when his glucose reading is greater than 220 mg/dL B) A patient who never deviates from her prescribed dose of insulin C) A patient who adheres closely to a meal plan and meal schedule D) A patient who eliminates carbohydrates from his daily intake

: C Feedback: The therapeutic goal for diabetes management is to achieve normal blood glucose levels without hypoglycemia. Therefore, diabetes management involves constant assessment and modification of the treatment plan by health professionals and daily adjustments in therapy (possibly including insulin) by patients. For patients who require insulin to help control blood glucose levels, maintaining consistency in the amount of calories and carbohydrates ingested at meals is essential. In addition, consistency in the approximate time intervals between meals, and the snacks, help maintain overall glucose control. Skipping meals is never advisable for person with type 1 diabetes.

38. A nurse is assessing a patient who has diabetes for the presence of peripheral neuropathy. The nurse should question the patient about what sign or symptom that would suggest the possible development of peripheral neuropathy? A) Persistently cold feet B) Pain that does not respond to analgesia C) Acute pain, unrelieved by rest D) The presence of a tingling sensation

: D Feedback: Although approximately half of patients with diabetic neuropathy do not have symptoms, initial symptoms may include paresthesias (prickling, tingling, or heightened sensation) and burning sensations (especially at night). Cold and intense pain are atypical early signs of this complication.

31. A patient has received a diagnosis of type 2 diabetes. The diabetes nurse has made contact with the patient and will implement a program of health education. What is the nurse's priority action? A) Ensure that the patient understands the basic pathophysiology of diabetes. B) Identify the patient's body mass index. C) Teach the patient "survival skills" for diabetes. D) Assess the patient's readiness to learn.

: D Feedback: Before initiating diabetes education, the nurse assesses the patient's (and family's) readiness to learn. This must precede other physiologic assessments (such as BMI) and providing health education.

14. A diabetes nurse educator is presenting the American Diabetes Association (ADA) recommendations for levels of caloric intake. What do the ADA's recommendations include? A) 10% of calories from carbohydrates, 50% from fat, and the remaining 40% from protein B) 10% to 20% of calories from carbohydrates, 20% to 30% from fat, and the remaining 50% to 60% from protein C) 20% to 30% of calories from carbohydrates, 50% to 60% from fat, and the remaining 10% to 20% from protein D) 50% to 60% of calories from carbohydrates, 20% to 30% from fat, and the remaining 10% to 20% from protein

: D Feedback: Currently, the ADA and the Academy of Nutrition and Dietetics (formerly the American Dietetic Association) recommend that for all levels of caloric intake, 50% to 60% of calories come from carbohydrates, 20% to 30% from fat, and the remaining 10% to 20% from protein.

21. A medical nurse is aware of the need to screen specific patients for their risk of hyperglycemic hyperosmolar syndrome (HHS). In what patient population does hyperosmolar nonketotic syndrome most often occur? A) Patients who are obese and who have no known history of diabetes B) Patients with type 1 diabetes and poor dietary control C) Adolescents with type 2 diabetes and sporadic use of antihyperglycemics D) Middle-aged or older people with either type 2 diabetes or no known history of diabetes

: D Feedback: HHS occurs most often in older people (50 to 70 years of age) who have no known history of diabetes or who have type 2 diabetes.

29. A nurse is teaching basic "survival skills" to a patient newly diagnosed with type 1 diabetes. What topic should the nurse address? A) Signs and symptoms of diabetic nephropathy B) Management of diabetic ketoacidosis C) Effects of surgery and pregnancy on blood sugar levels D) Recognition of hypoglycemia and hyperglycemia

: D Feedback: It is imperative that newly diagnosed patients know the signs and symptoms and management of hypo- and hyperglycemia. The other listed topics are valid points for education, but are not components of the patient's immediate "survival skills" following a new diagnosis.

25. A patient with type 2 diabetes has been managing his blood glucose levels using diet and metformin (Glucophage). Following an ordered increase in the patient's daily dose of metformin, the nurse should prioritize which of the following assessments? A) Monitoring the patient's neutrophil levels B) Assessing the patient for signs of impaired liver function C) Monitoring the patient's level of consciousness and behavior D) Reviewing the patient's creatinine and BUN levels

: D Feedback: Metformin has the potential to be nephrotoxic; consequently, the nurse should monitor the patient's renal function. This drug does not typically affect patients' neutrophils, liver function, or cognition.

9. A school nurse is teaching a group of high school students about risk factors for diabetes. Which of the following actions has the greatest potential to reduce an individual's risk for developing diabetes? A) Have blood glucose levels checked annually. B) Stop using tobacco in any form. C) Undergo eye examinations regularly. D) Lose weight, if obese.

: D Feedback: Obesity is a major modifiable risk factor for diabetes. Smoking is not a direct risk factor for the disease. Eye examinations are necessary for persons who have been diagnosed with diabetes, but they do not screen for the disease or prevent it. Similarly, blood glucose checks do not prevent the diabetes.

15. An older adult patient with type 2 diabetes is brought to the emergency department by his daughter. The patient is found to have a blood glucose level of 623 mg/dL. The patient's daughter reports that the patient recently had a gastrointestinal virus and has been confused for the last 3 hours. The diagnosis of hyperglycemic hyperosmolar syndrome (HHS) is made. What nursing action would be a priority? A) Administration of antihypertensive medications B) Administering sodium bicarbonate intravenously C) Reversing acidosis by administering insulin D) Fluid and electrolyte replacement

: D Feedback: The overall approach to HHS includes fluid replacement, correction of electrolyte imbalances, and insulin administration. Antihypertensive medications are not indicated, as hypotension generally accompanies HHS due to dehydration. Sodium bicarbonate is not administered to patients with HHS, as their plasma bicarbonate level is usually normal. Insulin administration plays a less important role in the treatment of HHS because it is not needed for reversal of acidosis, as in diabetic ketoacidosis (DKA).

11. A newly admitted patient with type 1 diabetes asks the nurse what caused her diabetes. When the nurse is explaining to the patient the etiology of type 1 diabetes, what process should the nurse describe? A) "The tissues in your body are resistant to the action of insulin, making the glucose levels in your blood increase." B) "Damage to your pancreas causes an increase in the amount of glucose that it releases, and there is not enough insulin to control it." C) "The amount of glucose that your body makes overwhelms your pancreas and decreases your production of insulin." D) "Destruction of special cells in the pancreas causes a decrease in insulin production. Glucose levels rise because insulin normally breaks it down."

: D Feedback: Type 1 diabetes is characterized by the destruction of pancreatic beta cells, resulting in decreased insulin production, unchecked glucose production by the liver, and fasting hyperglycemia. Also, glucose derived from food cannot be stored in the liver and remains circulating in the blood, which leads to postprandial hyperglycemia. Type 2 diabetes involves insulin resistance and impaired insulin secretion. The body does not "make" glucose.

25. A 72-year-old patient with a history of benign prostatic hyperplasia (BPH) is admitted with acute urinary retention and elevated blood urea nitrogen (BUN) and creatinine levels. Which prescribed therapy should the nurse implement first? a. Insert urethral catheter. b. Obtain renal ultrasound. c. Draw a complete blood count. d. Infuse normal saline at 50 mL/hour.

A

8. Sodium polystyrene sulfonate (Kayexalate) is ordered for a patient with hyperkalemia. Before administering the medication, the nurse should assess the a. bowel sounds. b. blood glucose. c. blood urea nitrogen (BUN). d. level of consciousness (LOC).

A

A 27-year-old patient admitted with diabetic ketoacidosis (DKA) has a serum glucose level of 732 mg/dL and serum potassium level of 3.1 mEq/L. Which action prescribed by the health care provider should the nurse take first? a. Place the patient on a cardiac monitor. b. Administer IV potassium supplements. c. Obtain urine glucose and ketone levels. d. Start an insulin infusion at 0.1 units/kg/hr.

A

A 32-year-old patient with diabetes is starting on intensive insulin therapy. Which type of insulin will the nurse discuss using for mealtime coverage? a. Lispro (Humalog) b. Glargine (Lantus) c. Detemir (Levemir) d. NPH (Humulin N)

A

A 38-year-old patient who has type 1 diabetes plans to swim laps daily at 1:00 PM. The clinic nurse will plan to teach the patient to: a. check glucose level before, during, and after swimming. b. delay eating the noon meal until after the swimming class. c. increase the morning dose of neutral protamine Hagedorn (NPH) insulin. d. time the morning insulin injection so that the peak occurs while swimming.

A

A 54-year-old patient is admitted with diabetic ketoacidosis. Which admission order should the nurse implement first? a. Infuse 1 liter of normal saline per hour. b. Give sodium bicarbonate 50 mEq IV push. c. Administer regular insulin 10 U by IV push. d. Start a regular insulin infusion at 0.1 units/kg/hr.

A

A 55-year-old female patient with type 2 diabetes has a nursing diagnosis of imbalanced nutrition: more than body requirements. Which goal is most important for this patient? a. The patient will reach a glycosylated hemoglobin level of less than 7%. b. The patient will follow a diet and exercise plan that results in weight loss. c. The patient will choose a diet that distributes calories throughout the day. d. The patient will state the reasons for eliminating simple sugars in the diet.

A

A patient receives aspart (NovoLog) insulin at 8:00 AM. Which time will it be most important for the nurse to monitor for symptoms of hypoglycemia? a. 10:00 AM b. 12:00 AM c. 2:00 PM d. 4:00 PM

A

The nurse has administered 4 oz of orange juice to an alert patient whose blood glucose was 62 mg/dL. Fifteen minutes later, the blood glucose is 67 mg/dL. Which action should the nurse take next? a. Give the patient 4 to 6 oz more orange juice. b. Administer the PRN glucagon (Glucagon) 1 mg IM. c. Have the patient eat some peanut butter with crackers. d. Notify the health care provider about the hypoglycemia.

A

Which information will the nurse include in teaching a female patient who has peripheral arterial disease, type 2 diabetes, and sensory neuropathy of the feet and legs? a. Choose flat-soled leather shoes. b. Set heating pads on a low temperature. c. Use callus remover for corns or calluses. d. Soak feet in warm water for an hour each day.

A

Which question during the assessment of a diabetic patient will help the nurse identify autonomic neuropathy? a. "Do you feel bloated after eating?" b. "Have you seen any skin changes?" c. "Do you need to increase your insulin dosage when you are stressed?" d. "Have you noticed any painful new ulcerations or sores on your feet?"

A

Which statement by the patient indicates a need for additional instruction in administering insulin? a. "I need to rotate injection sites among my arms, legs, and abdomen each day." b. "I can buy the 0.5 mL syringes because the line markings will be easier to see." c. "I should draw up the regular insulin first after injecting air into the NPH bottle." d. "I do not need to aspirate the plunger to check for blood before injecting insulin."

A

In which order will the nurse take these steps to prepare NPH 20 units and regular insulin 2 units using the same syringe? a. Rotate NPH vial. b. Withdraw regular insulin. c. Withdraw 20 units of NPH. d. Inject 20 units of air into NPH vial. e. Inject 2 units of air into regular insulin vial.

A D E B C

A patient with a recent diagnosis of heart failure has been prescribed furosemide (Lasix) in an effort to physiologically do what for the patient?

A Reduce preload. Diuretics such as furosemide are used in the treatment of HF to mobilize edematous fluid, reduce pulmonary venous pressure, and reduce preload.

D (Correct Explanation: Short-acting beta2-adrenergic agonists (albuterol [AccuNeb, Proventil, Ventolin], levalbuterol [Xopenex HFA], and pirbuterol [Maxair]) are the medications of choice for relief of acute symptoms and prevention of exercise-induced asthma. Cromolyn sodium (Crolom, NasalCrom) and nedocromil (Alocril, Tilade) are mild to moderate anti-inflammatory agents and are considered alternative medications for treatment. These medications stabilize mast cells. These medications are contraindicated in acute asthma exacerbations. Long-acting beta2-adrenergic agonists are not indicated for immediate relief of symptoms. These include theophylline (Slo-Bid, Theo- Dur) and salmeterol (Serevent Diskus).)

A child is having an asthma attack and the parent can't remember which inhaler to use for quick relief. The nurse accesses the child's medication information and tells the parent to use which inhalant? a) Serevent b) Theo-Dur c) Cromolyn sodium d) Proventil

A (Lung sounds Explanation: A client with COPD is at risk for developing pneumothorax. The description given is consistent with possible pneumothorax. Though the nurse will assess all the data, auscultating the lung sounds will provide the nurse with the information if the client has a pneumothorax.)

A client has a history of chronic obstructive pulmonary disease (COPD). Following a coughing episode, the client reports sudden and unrelieved shortness of breath. Which of the following is the most important for the nurse to assess? a) Lung sounds b) Skin color c) Heart rate d) Respiratory rate

A (salmeterol/fluticasone (Seretide) MDI daily at 0800 Explanation: When providing information about medications, the nurse needs to include right drug, right dose, right route, right frequency, and right time. Salmeterol/fluticasone does not include how many puffs the client is to take.)

A client is being admitted to an acute healthcare facility with an exacerbation of chronic obstructive pulmonary disease (COPD). The client had been taking an antibiotic at home with poor relief of symptoms and has recently decided to stop smoking. The nurse is reviewing at-home medications with the client. The nurse is placing this information on the Medication Reconciliation Record. Which of the following is incomplete information? a) salmeterol/fluticasone (Seretide) MDI daily at 0800 b) azithromycin (Zithromax) 600 mg oral daily for 10 days at 0800, on day 4 c) prednisone 5 mg oral daily at 0800 d) nicotine patch (Nicoderm) 21 mg 1 patch daily at 0800

B (Oxygen through nasal cannula at 2 L/minute Explanation: All options listed are treatments that may be used for a client with an exacerbation of COPD. The first line of treatment is oxygen therapy.)

A client is being seen in the emergency department for exacerbation of chronic obstructive pulmonary disease (COPD). The first action of the nurse is to administer which of the following prescribed treatments? a) Ipratropium bromide (Alupent) by metered-dose inhaler b) Oxygen through nasal cannula at 2 L/minute c) Vancomycin 1 gram intravenously over 1 hour d) Intravenous methylprednisolone (Solu-Medrol) 120 mg

A (An inspiratory-expiratory (I:E) ratio of 2:1 Explanation: The normal I:E ratio is 1:2, meaning that expiration takes twice as long as inspiration. A ratio of 2:1 is seen in clients with COPD because inspiration is shorter than expiration. A client with COPD typically has a barrel chest in which the anteroposterior diameter is larger than the transverse chest diameter. A client with COPD usually has a respiratory rate greater than 12 breaths/minute and an oxygen saturation rate below 93%.)

A client with chronic obstructive pulmonary disease (COPD) is admitted to an acute care facility because of an acute respiratory infection. When assessing the client's respiratory status, which finding should the nurse anticipate? a) An inspiratory-expiratory (I:E) ratio of 2:1 b) A transverse chest diameter twice that of the anteroposterior diameter c) A respiratory rate of 12 breaths/minute d) An oxygen saturation of 99%

C (Instruct the client to drink at least 2 L of fluid daily. Explanation: Mobilizing secretions is crucial to maintaining a patent airway and maximizing gas exchange in the client with COPD. Measures that help mobilize secretions include drinking 2 L of fluid daily, practicing controlled pursed-lip breathing, and engaging in moderate activity. Anxiolytics rarely are recommended for the client with COPD because they may cause sedation and subsequent infection from inadequate mobilization of secretions. Because COPD rarely causes pain, pain medication isn't indicated.)

A client with chronic obstructive pulmonary disease (COPD) is admitted to the medical-surgical unit. To help this client maintain a patent airway and achieve maximal gas exchange, the nurse should: a) administer anxiolytics, as ordered, to control anxiety. b) maintain the client on bed rest. c) instruct the client to drink at least 2 L of fluid daily. d) administer pain medication as ordered.

B (Dyspnea Explanation: Dyspnea is characteristic of emphysema. A chronic cough is considered the primary symptom of chronic bronchitis. Refer to Table 11-1 in the text.)

Which of the following is a symptom diagnostic of emphysema? a) Normal elastic recoil b) Dyspnea c) The occurrence of cor pulmonale d) Copious sputum production

A (Inflammation Explanation: Inflammation is the key underlying feature and leads to recurrent episodes of asthma symptoms: cough, chest tightness, wheeze, and dyspnea.)

Which of the following is the key underlying feature of asthma? a) Inflammation b) Shortness of breath c) Productive cough d) Chest tightness

A ("Delay self-care activities for 1 hour." Explanation: Some clients with COPD have shortness of breath and fatigue in the morning on arising as a result of bronchial secretions. Planning self-care activities around this time may be better tolerated by the client, such as delaying activities until the client is less short of breath or fatigued. The client raising the arms over the head may increase dyspnea and fatigue. Sitting in a chair when bathing or dressing will aid in dyspnea and fatigue but does not address the situation upon arising. Drinking fluids will assist in liquifying secretions which, thus, will aid in breathing, but again does not address the situation in the morning.)

A client with chronic obstructive pulmonary disease (COPD) reports increased shortness of breath and fatigue for 1 hour after awakening in the morning. Which of the following statements by the nurse would best help with the client's shortness of breath and fatigue? a) "Delay self-care activities for 1 hour." b) "Sit in a chair whenever doing an activity." c) "Drink fluids upon arising from bed." d) "Raise your arms over your head."

B (Theophylline Explanation: Theophylline is an example of a methylxanthine. All the others are examples of inhaled short-acting beta2 agonists.)

A commonly prescribed methylxanthine used as a bronchodilator is which of the following? a) Levalbuteral b) Theophylline c) Albuteral d) Terbutaline

C (High-protein Explanation: Breathing is more difficult for clients with COPD, and increased metabolic demand puts them at risk for nutritional deficiencies. These clients must have a high intake of protein for increased calorie consumption. Full liquids, 1,800-calorie ADA, and low-fat diets aren't appropriate for a client with COPD.)

A nurse consulting with a nutrition specialist knows it's important to consider a special diet for a client with chronic obstructive pulmonary disease (COPD). Which diet is appropriate for this client? a) 1,800-calorie ADA b) Low-fat c) High-protein d) Full-liquid

B, C, E (Explanation: Singulair, Accolate, and Zyflo are leukotriene modifiers. Atrovent is a short-acting anticholinergic. Spiriva is a long-acting anticholinergic.)

Which of the following medications are classified as leukotriene modifiers (inhibitors)? Select all that apply. a) Tiotropium (Spiriva) b) Zafirlukast (Accolate) c) Montelukast (Singulair) d) Ipratropium HFA (Atrovent) e) Zileuton (Zyflo)

A ("I do not need to rinse my mouth with this type of inhaler." Mouth-washing and spitting are effective in reducing the amount of drug swallowed and absorbed systemically. Actuation during a slow (30 L/min or 3 to 5 seconds) and deep inhalation should be followed by 10 seconds of holding the breath. The patient should actuate only once. Simple tubes do not obviate the spacer/VHC per inhalation.)

A nurse has just completed teaching with a patient who has been prescribed a meter-dosed inhaler for the first time. Which of the following statements would the nurse use to initiate further teaching and follow-up care? a) "I do not need to rinse my mouth with this type of inhaler." b) "I will make sure to take a slow, deep breath as I push on my inhaler." c) "If I use the spacer, I know I am only supposed to push on the inhaler once." d) "After I breathe in, I will hold my breath for 10 seconds."

C (Increase walking distance around a city block without shortness of breath. Explanation: If the client has mild COPD, goals are to increase exercise and prevent further loss of pulmonary function. The client who increases his walking distance without shortness of breath meets these criteria. If the client has severe COPD, goals are then to preserve current pulmonary function and relieve symptoms as much as possible. Examples of these goals are the other options, in which the activity level is at current and symptoms are relieved to tolerable or close to tolerable.)

A nurse is assisting a client with mild chronic obstructive pulmonary disease (COPD) to set a goal related to the condition. Which of the following is an appropriate goal for this client? a) Maintain activity level of walking to the mailbox. b) Relieve shortness of breath to a level as close as possible to tolerable. c) Increase walking distance around a city block without shortness of breath. d) Continue with current level of mobility at home.

D (uses the sternocleidomastoid muscles. Explanation: Use of accessory muscles indicates worsening breathing conditions. Assuming the tripod position, a 93% pulse oximetry reading, and a request for the nurse to raise the head of the bed don't indicate that the client's condition is worsening.)

A nurse is caring for a client admitted with an exacerbation of asthma. The nurse knows the client's condition is worsening when he: a) wants the head of the bed raised to a 90-degree level. b) has a pulse oximetry reading of 93%. c) sits in tripod position. d) uses the sternocleidomastoid muscles.

B (An inhaled beta2-adrenergic agonist Correct Explanation: An inhaled beta2-adrenergic agonist helps promote bronchodilation, which improves oxygenation. Although an I.V. beta2-adrenergic agonist can be used, the client needs be monitored because of the drug's greater systemic effects. The I.V. form is typically used when the inhaled beta2-adrenergic agonist doesn't work. A corticosteroid is slow acting, so its use won't reduce hypoxia in the acute phase.)

A nurse is caring for a client with status asthmaticus. Which medication should the nurse prepare to administer? a) An inhaled corticosteroid b) An inhaled beta2-adrenergic agonist c) An I.V. beta2-adrenergic agonist d) An oral corticosteroid

D (Ineffective coping related to anxiety Explanation: Any factor that interferes with normal breathing quite naturally induces anxiety, depression, and changes in behavior. Constant shortness of breath and fatigue may make the patient irritable and apprehensive to the point of panic. Although the other choices are correct, the most important psychosocial nursing diagnosis for a patient with COPD is ineffective coping related to a high level of anxiety.)

A nursing student understands the importance of the psychosocial aspects of disease processes. When working with a patient with COPD, the student would rank which of the following nursing diagnoses as the MOST important when analyzing the psychosocial effects? a) Activity intolerance related to fatigue b) Disturbed sleep pattern related to cough c) High risk for ineffective therapeutic regimen management related to lack of knowledge d) Ineffective coping related to anxiety

A (Sputum and a productive cough Explanation: Chronic bronchitis, a disease of the airways, is defined as the presence of cough and sputum production for at least 3 months in each of 2 consecutive years.)

A patient comes to the clinic for the third time in 2 months with chronic bronchitis. What clinical symptoms does the nurse anticipate assessing for this patient? a) Sputum and a productive cough b) Tachypnea and tachycardia c) Fever, chills, and diaphoresis d) Chest pain during respiration

A (Intal (Cromolyn Sodium) Intal is contraindicated in patients with acute asthma exacerbation. Indications for Intal are long-term prevention of symptoms in mild, persistent asthma; it may modify inflammation. Intal is also a preventive treatment prior to exposure to exercise or known allergen. Proventil (albuterol), Xopenex (levalbuterol HFA), and Atrovent (ipratropium) can be used to relieve acute symptoms.)

A patient is being admitted to the medical-surgical unit for the treatment of an exacerbation of acute asthma. Which of the following medications is contraindicated in the treatment of asthma exacerbations? a) Intal (Cromolyn Sodium) b) Atrovent (Ipratropium) c) Proventil (Albuterol) d) Xopenex (Levalbuterol HFA)

C (Respiratory acidosis Correct Explanation: In status asthmaticus, increasing PaCO2 (to normal levels or levels indicating respiratory acidosis) is a danger sign signifying impending respiratory failure. Understanding the sequence of the pathophysiologic processes in status asthmaticus is important for understanding assessment findings. Respiratory alkalosis occurs initially because the patient hyperventilates and PaCO2 decreases. As the condition continues, air becomes trapped in the narrowed airways and carbon dioxide is retained, leading to respiratory acidosis.)

A patient is being treated for status asthmaticus. What danger sign does the nurse observe that can indicate impending respiratory failure? a) Metabolic alkalosis b) Metabolic acidosis c) Respiratory acidosis d) Respiratory alkalosis

A, B, C, E (Explanation: The goal is to have a stable patient with COPD leading the most productive life possible. COPD cannot necessarily be cured, but it can be managed so that the patient can live a reasonably normal life. With adequate management, patients should not have to give up their usual activities.)

A patient presents to the ED experiencing symptoms of COPD exacerbation. The nurse understands there are goals of therapy that are achieved to improve the patient's condition. Which of the following are therapy goals? Select all that apply. a) Return the patient to his original functioning abilities. b) Treat the underlying cause of the event. c) Provide long-term support for medical management. d) Teach the patient to suspend activity. e) Provide medical support for the current exacerbation.

D (Correct Explanation: Proventil, a SABA, is given to asthmatic patients for quick relief of symptoms. Atrovent is an anticholinergic. Combivent is a combination SABA/anticholinergic, and Flovent is a corticosteroid.)

A patient with asthma is prescribed a short acting beta-adrenergic (SABA) for quick relief. Which of the following is the most likely drug to be prescribed? a) Atrovent b) Flovent c) Combivent d) Proventil

B (Administer the salmeterol and then administer the triamcinolone. Explanation: A client with asthma typically takes bronchodilators and uses corticosteroid inhalers to prevent acute episodes. Triamcinolone is a corticosteroid; Salmeterol is an adrenergic stimulant (bronchodilator). If the client is ordered a bronchodilator and another inhaled medication, the bronchodilator should be administered first to dilate the airways and to enhance the effectiveness of the second medication. The client may not choose the order in which these drugs are administered because they must be administered in a particular order. Monitoring the client's theophylline level isn't necessary before administering these drugs because neither drug contains theophylline.)

A physician orders triamcinolone (Azmacort) and salmeterol (Serevent) for a client with a history of asthma. What action should the nurse take when administering these drugs? a) Administer the triamcinolone and then administer the salmeterol. b) Administer the salmeterol and then administer the triamcinolone. c) Allow the client to choose the order in which the drugs are administered. d) Monitor the client's theophylline level before administering the medications.

A (Sharp, stabbing chest pain Explanation: The initial symptom is usually chest pain of sudden onset that leads to feelings of chest pressure, dyspnea, and tachycardia. A cough may be present.)

A pneumothorax is a possible complication of COPD. Symptoms will depend on the suddenness of the attack and the size of the air leak. The most common, immediate symptom that should be assessed is: a) Sharp, stabbing chest pain b) Dyspnea c) Tachycardia d) A dry, hacking cough

Which person should the nurse identify as having the highest risk for abdominal aortic aneurysm? A. A 70-year-old male, with high cholesterol and hypertension B. A 40-year-old female with obesity and metabolic syndrome C. A 60-year-old male with renal insufficiency who is physically inactive D. A 65-year-old female with hyperhomocysteinemia and substance abuse

A. A 70-year-old male, with high cholesterol and hypertension The most common etiology of descending abdominal aortic aneurysm (AAA) is atherosclerosis. Male gender, age 65 years or older, and tobacco use are the major risk factors for AAAs of atherosclerotic origin. Other risk factors include the presence of coronary or peripheral artery disease, high blood pressure, and high cholesterol.

What medications should the nurse expect to include in the teaching plan to decrease the risk of cardiovascular events and death for PAD patients (select all that apply)? A. Ramipril (Altace) B. Cilostazol (Pletal) C. Simvastatin (Zocor) D. Clopidogrel (Plavix) E. Warfarin (Coumadin) F. Aspirin (acetylsalicylic acid)

A. Ramipril (Altace) C. Simvastatin (Zocor) F. Aspirin (acetylsalicylic acid) Angiotensin-converting enzyme inhibitors (e.g., ramipril [Altace]) are used to control hypertension. Statins (e.g., simvastatin [Zocor]) are used for lipid management. Aspirin is used as an antiplatelet agent. Cilostazol (Pletal) is used for intermittent claudication, but it does not reduce CVD morbidity and mortality risks. Clopidogrel may be used if the patient cannot tolerate aspirin. Anticoagulants (e.g., warfarin [Coumadin]) are not recommended to prevent CVD events in PAD patients.

A patient with asthma has a personal best peak expiratory flow rate (PEFR) of 400 L/minute. When explaining the asthma action plan, the nurse will teach the patient that a change in therapy is needed when the PEFR is less than ___ L/minute

ANS: 320 A PEFR less than 80% of the personal best indicates that the patient is in the yellow zone where changes in therapy are needed to prevent progression of the airway narrowing. DIF: Cognitive Level: Apply (application) REF: 579 TOP: Nursing Process: Evaluation MSC: NCLEX: Physiological Integrity

17. The clinic nurse teaches a patient with a 42 pack-year history of cigarette smoking about lung disease. Which information will be most important for the nurse to include? a. Options for smoking cessation b. Reasons for annual sputum cytology testing c. Erlotinib (Tarceva) therapy to prevent tumor risk d. Computed tomography (CT) screening for lung cancer

ANS: A Because smoking is the major cause of lung cancer, the most important role for the nurse is teaching patients about the benefits of and means of smoking cessation. CT scanning is currently being investigated as a screening test for high-risk patients. However, if there is a positive finding, the person already has lung cancer. Erlotinib may be used in patients who have lung cancer, but it is not used to reduce the risk of developing cancer.

11. An older patient is receiving standard multidrug therapy for tuberculosis (TB). The nurse should notify the health care provider if the patient exhibits which finding? a. Yellow-tinged skin b. Orange-colored sputum c. Thickening of the fingernails d. Difficulty hearing high-pitched voices

ANS: A Noninfectious hepatitis is a toxic effect of isoniazid (INH), rifampin, and pyrazinamide, and patients who develop hepatotoxicity will need to use other medications. Changes in hearing and nail thickening are not expected with the four medications used for initial TB drug therapy. Presbycusis is an expected finding in the older adult patient. Orange discoloration of body fluids is an expected side effect of rifampin and not an indication to call the health care provider.

1. Following assessment of a patient with pneumonia, the nurse identifies a nursing diagnosis of ineffective airway clearance. Which assessment data best supports this diagnosis? a. Weak, nonproductive cough effort b. Large amounts of greenish sputum c. Respiratory rate of 28 breaths/minute d. Resting pulse oximetry (SpO2) of 85%

ANS: A The weak, nonproductive cough indicates that the patient is unable to clear the airway effectively. The other data would be used to support diagnoses such as impaired gas exchange and ineffective breathing pattern.

29. When caring for a patient who has just arrived on the medical-surgical unit after having cardiac catheterization, which nursing intervention should the nurse delegate to a licensed practical/vocational nurse (LPN/LVN)? a. Give the scheduled aspirin and lipid-lowering medication. b. Perform the initial assessment of the catheter insertion site. c. Teach the patient about the usual postprocedure plan of care. d. Titrate the heparin infusion according to the agency protocol.

ANS: A Administration of oral medications is within the scope of practice for LPNs/LVNs. The initial assessment of the patient, patient teaching, and titration of IV anticoagulant medications should be done by the registered nurse (RN).

41. The nurse reviews the medication administration record (MAR) for a patient having an acute asthma attack. Which medication should the nurse administer first? a. Albuterol (Ventolin) 2.5 mg per nebulizer b. Methylprednisolone (Solu-Medrol) 60 mg IV c. Salmeterol (Serevent) 50 mcg per dry-powder inhaler (DPI) d. Triamcinolone (Azmacort) 2 puffs per metered-dose inhaler (MDI)

ANS: A Albuterol is a rapidly acting bronchodilator and is the first-line medication to reverse airway narrowing in acute asthma attacks. The other medications work more slowly. DIF: Cognitive Level: Apply (application) REF: 570 | 576 OBJ: Special Questions: Prioritization TOP: Nursing Process: Implementation MSC: NCLEX: Physiological Integrity

9. A patient has heart failure and is taking an ACE inhibitor. The patient has developed fibrotic changes in the heart and vessels. The nurse expects the provider to order which medication to counter this development? a. Aldosterone antagonist b. Angiotensin II receptor blocker (ARB) c. Beta blocker d. Direct renin inhibitor (DRI)

ANS: A Aldosterone antagonists are added to therapy for patients with worsening symptoms of HF. Aldosterone promotes myocardial remodeling and myocardial fibrosis, so aldosterone antagonists can help with this symptom. ARBs are given for patients who do not tolerate ACE inhibitors. Beta blockers do not prevent fibrotic changes. DRIs are not widely used.

14. A patient with heart failure who has been given digoxin [Lanoxin] daily for a week complains of nausea. Before giving the next dose, the nurse will: a. assess the heart rate (HR) and give the dose if the HR is greater than 60 beats per minute. b. contact the provider to report digoxin toxicity. c. request an order for a decreased dose of digoxin. d. review the serum electrolyte values and withhold the dose if the potassium level is greater than 3.5 mEq/L.

ANS: A Anorexia, nausea, and vomiting are the most common adverse effects of digoxin and should cause nurses to evaluate for more serious signs of toxicity. If the HR is greater than 60 beats per minute, the dose may be given. Nausea by itself is not a sign of toxicity. A decreased dose is not indicated. A serum potassium level less than 3.5 mEq/L is an indication for withholding the dose.

4. A patient newly diagnosed with heart failure is admitted to the hospital. The nurse notes a pulse of 90 beats per minute. The nurse will observe this patient closely for: a. decreased urine output. b. increased blood pressure. c. jugular vein distension. d. shortness of breath.

ANS: A As the heart rate increases, ventricular filling decreases, and cardiac output and renal perfusion decrease. Tachycardia does not elevate blood pressure. Jugular vein distension and shortness of breath occur with fluid volume overload.

A nurse is giving aspirin to a patient during acute management of STEMI. The patient asks why a chewable tablet is given. Which response by the nurse is correct? a. "Aspirin is absorbed more quickly when it is chewed." b. "Chewing aspirin prevents it from being metabolized by the liver." c. "Chewing aspirin prevents stomach irritation." d. "More of the drug is absorbed when aspirin is chewed."

ANS: A Aspirin should be chewed to allow rapid absorption across the buccal mucosa. Chewing aspirin does not affect hepatic metabolism, stomach irritation, or the amount absorbed.

36. A patient who is experiencing an acute asthma attack is admitted to the emergency department. Which assessment should the nurse complete first? a. Listen to the patient's breath sounds. b. Ask about inhaled corticosteroid use. c. Determine when the dyspnea started. d. Obtain the forced expiratory volume (FEV) flow rate.

ANS: A Assessment of the patient's breath sounds will help determine how effectively the patient is ventilating and whether rapid intubation may be necessary. The length of time the attack has persisted is not as important as determining the patient's status at present. Most patients having an acute attack will be unable to cooperate with an FEV measurement. It is important to know about the medications the patient is using but not as important as assessing the breath sounds. DIF: Cognitive Level: Apply (application) REF: 564-565 OBJ: Special Questions: Prioritization TOP: Nursing Process: Assessment MSC: NCLEX: Physiological Integrity

34. Which information about a patient who has been receiving thrombolytic therapy for an acute myocardial infarction (AMI) is most important for the nurse to communicate to the health care provider? a. No change in the patient's chest pain b. An increase in troponin levels from baseline c. A large bruise at the patient's IV insertion site d. A decrease in ST-segment elevation on the electrocardiogram

ANS: A Continued chest pain suggests that the thrombolytic therapy is not effective and that other interventions such as percutaneous coronary intervention (PCI) may be needed. Bruising is a possible side effect of thrombolytic therapy, but it is not an indication that therapy should be discontinued. The decrease of the ST-segment elevation indicates that thrombolysis is occurring and perfusion is returning to the injured myocardium. An increase in troponin levels is expected with reperfusion and is related to the washout of cardiac markers into the circulation as the blocked vessel is opened.

6. A patient with volume overload begins taking a thiazide diuretic. The nurse will tell the patient to expect which outcome when taking this drug? a. Improved exercise tolerance b. Increased cardiac output c. Prevention of cardiac remodeling d. Prolonged survival

ANS: A Diuretics help reduce fluid volume overload, which, by reducing pulmonary edema, can improve exercise tolerance. Diuretics do not improve cardiac output. ACE inhibitors are used to prevent cardiac remodeling and to improve long-term survival.

25. The nurse is caring for a patient who was admitted to the coronary care unit following an acute myocardial infarction (AMI) and percutaneous coronary intervention the previous day. Teaching for this patient would include a. when cardiac rehabilitation will begin. b. the typical emotional responses to AMI. c. information regarding discharge medications. d. the pathophysiology of coronary artery disease.

ANS: A Early after an AMI, the patient will want to know when resumption of usual activities can be expected. At this time, the patient's anxiety level or denial will interfere with good understanding of complex information such as the pathophysiology of coronary artery disease (CAD). Teaching about discharge medications should be done closer to discharge. The nurse should support the patient by decreasing anxiety rather than discussing the typical emotional responses to myocardial infarction (MI).

What is the priority assessment by the nurse caring for a patient receiving IV nesiritide (Natrecor) to treat heart failure? C Blood pressure

the priority assessment would be monitoring for hypotension, the main adverse effect of nesiritide.

A 23-year-old male client who has recently started working in a coal mine confides that he is concerned about his long-term health. The nurse instructs the client which of the following ways to prevent occupational lung disease? Select all that apply. a) Do not smoke or quit smoking if currently smoking. b) Wear appropriate protective equipment when around airborne irritants and dusts. c) Schedule an annual lung x-ray to monitor his health. d) Try to find another occupation as soon as possible.

a) Do not smoke or quit smoking if currently smoking. b) Wear appropriate protective equipment when around airborne irritants and dusts. Explanation: The nurse may instruct clients that the following precautions may help prevent occupational lung disease: not smoking, wearing appropriate protective equipment when around airborne irritants and dusts, scheduling lung function evaluation with spirometry as recommended, becoming educated about lung diseases, and paying attention to risk evaluation of the workplace to identify risks for lung disease. pg.605

A client who is diagnosed with chronic respiratory failure will have which of the following symptoms? a) Dyspnea b) Hypercapnia c) Hypoxemia d) Ventilatory failure

a) Dyspnea Explanation: Chronic respiratory failure develops over a long time period as the result of another condition. The most common diseases leading to chronic respiratory failure are COPD and neuromuscular disorders. A fall in arterial oxygen levels is a sign of acute respiratory failure. A rise in arterial CO2 is a sign of acute respiratory failure. Ventilatory failure develops in acute respiratory failure when the alveoli cannot adequately expand. pg.595

A patient with acute pericarditis is exhibiting distended jugular veins, tachycardia, tachypnea, bradycardia, and muffled heart sounds. The senior nursing student recognizes these symptoms occur when a) Excess pericardial fluid compresses the heart and prevents adequate diastolic filling. b) Fibrin accumulation on the visceral pericardium infiltrates into the myocardium, creating generalized myocardial dysfunction. c) The pericardial space is eliminated with scar tissue and thickened pericardium. d) The parietal and visceral pericardial membranes adhere to each other, preventing normal myocardial contraction.

a) Excess pericardial fluid compresses the heart and prevents adequate diastolic filling.

A client in chronic renal failure becomes confused and complains of abdominal cramping, racing heart rate, and numbness of the extremities. The nurse relates these symptoms to which of the following lab values? a) Hyperkalemia b) Elevated white blood cells c) Hypocalcemia d) Elevated urea levels

a) Hyperkalemia Hyperkalemia is the life-threatening effect of renal failure. The client can become apathetic; confused; and have abdominal cramping, dysrhythmias, nausea, muscle weakness, and numbness of the extremities. Symptoms of hypocalcemia are muscle twitching, irritability, and tetany. Elevation in urea levels can result in azotemia, which can be exhibited in fluid and electrolyte and/or acid-base imbalance. Elevation of WBCs is not indicated.

The client with acute renal failure progresses through four phases. Which of the following describes the initiation phase? a) It is accompanied by reduced blood flow to the nephrons. b) The excretion of wastes and electrolytes continues to be impaired despite increased water content of the urine. c) Normal glomerular filtration and tubular function are restored. d) Fluid volume excess develops, which leads to edema, hypertension, and cardiopulmonary complications.

a) It is accompanied by reduced blood flow to the nephrons. The initiation phase is accompanied by reduced blood flow to the nephrons. In the oliguric phase, fluid volume excess develops, which leads to edema, hypertension, and cardiopulmonary complications. During the diuretic phase, excretion of wastes and electrolytes continues to be impaired despite increased water content of the urine. During the recovery phase, normal glomerular filtration and tubular function are restored.

A nurse completed a physical exam for an insurance company. The nurse noted a cluster of abnormalities that she knew was considered a major risk factor for coronary artery disease. Choose that condition. a) Metabolic syndrome b) Diabetes mellitus c) Hypolipidemia d) Congestive heart failure

a) Metabolic syndrome Metabolic syndrome includes three of six conditions that are recognized as a major risk factor for CAD. Insulin resistance is part of the syndrome but the patient may not yet have diabetes.

Upon discharge from the hospital, patients diagnosed with a myocardial infarction (MI) must be placed on all of the following medications except: a) Morphine IV b) Angiotensin-converting enzyme (ACE) inhibitor c) Aspirin d) Statin

a) Morphine IV Upon patient discharge, there needs to be documentation that the patient was discharged on a statin, an ACE or angiotensin receptor blocking agent (ARB), and aspirin. Morphine IV is used for these patients to reduce pain and anxiety. The patient would not be discharged with IV morphine.

A group of students are reviewing the phases of acute renal failure. The students demonstrate understanding of the material when they identify which of the following as occurring during the second phase? a) Oliguria b) Acute tubular necrosis c) Diuresis d) Restored glomerular function

a) Oliguria During the second phase, the oliguric phase, oliguria occurs. Diuresis occurs during the third or diuretic phase. Acute tubular necrosis (ATN) occurs during the first, or initiation, phase in which reduced blood flow to the nephrons leads to ATN. Restoration of glomerular function, if it occurs, occurs during the fourth, or recovery, phase.

The patient with cardiac failure is taught to report which of the following symptoms to the physician or clinic immediately? a) Persistent cough b) Ability to sleep through the night c) Increased appetite d) Weight loss

a) Persistent cough

A client is admitted to the hospital with systolic left-sided heart failure. The nurse knows to look for which of the following assessment findings for this client? a) Pulmonary congestion b) Jugular venous distention c) Nausea d) Pedal edema

a) Pulmonary congestion

You've been invited to speak to the Hospital Guild of the hospital where you practice nursing. You've been asked to address "Communicable Diseases of Winter" and are speaking to a large group of volunteer women, most of whom are older than 60 years. What practices should you encourage in these women, who are at the risk of pneumococcal and influenza infections? Select all that apply. a) Receiving vaccinations b) Techniques for incentive spirometry c) Hand antisepsis d) Using prescribed opioids

a) Receiving vaccinations c) Hand antisepsis b) Techniques for incentive spirometry Explanation: A powerful weapon against the spread of communicable disease is effective and frequent handwashing. Teaching the Guild members the proper method and times to wash their hands go a long way in disease prevention. The pneumococcal vaccine provides specific prevention against pneumococcal pneumonia and other infections caused by S. pneumoniae. pg.571

Common tests of renal function include which of the following? Select all that apply. a) Serum creatinine b) Arterial blood gas analysis c) Blood urea nitrogen (BUN) d) Creatinine clearance e) Renal concentration test

a) Serum creatinine c) Blood urea nitrogen (BUN) d) Creatinine clearance e) Renal concentration test Common tests of renal function include BUN, serum creatinine, creatinine clearance, and renal concentration tests. Arterial blood gas analysis is a test of respiratory function.

The laboratory results for a patient with renal failure, accompanied by decreased glomerular filtration, would be evaluated frequently. Which of the following is the most sensitive indicator of renal function? a) Serum creatinine of 1.5 mg/dL b) Creatinine clearance of 90 mL/min c) Urinary protein level of 150 mg/24h. d) BUN of 20 mg/dLb

a) Serum creatinine of 1.5 mg/dL As glomerular filtration decreases, the serum creatinine and BUN levels increase and the creatinine clearance decreases. Serum creatinine is the more sensitive indicator of renal function because of its constant production in the body. The BUN is affected not only by renal disease but also by protein intake in the diet, tissue catabolism, fluid intake, parenteral nutrition, and medications such as corticosteroids.

After 48 hours, a Mantoux test is evaluated. At the site, there is a 10 mm induration. This finding would be considered: a) Significant b) Negative c) Nonreactive d) Not significant

a) Significant Explanation: An induration of 10 mm or greater is usually considered significant and reactive in people who have normal or mildly impaired immunity. Erythema without induration is not considered significant. pg.588

Patients who are taking beta-adrenergic blocking agents should be cautioned not to stop taking their medications abruptly because which of the following may occur? a) Worsening angina b) Internal bleeding c) Thrombocytopenia d) Formation of blood clots

a) Worsening angina Patients taking beta blockers are cautioned not to stop taking them abruptly because angina may worsen and myocardial infarction may develop. Beta blockers do not cause the formation of blood clots, internal bleeding, or thrombocytopenia.

A positive Mantoux test indicates that a client: a) has produced an immune response. b) has an active case of tuberculosis. c) will develop full-blown tuberculosis. d) is actively immune to tuberculosis.

a) has produced an immune response. Explanation: The Mantoux test is based on the antigen/antibody response and will show a positive reaction after an individual has been exposed to tuberculosis and has formed antibodies to the tuberculosis bacteria. Thus, a positive Mantoux test indicates the production of an immune response. Exposure doesn't confer immunity. A positive test doesn't confirm that a person has (or will develop) tuberculosis. pg.588

A client with severe angina pectoris and electrocardiogram changes is seen by a physician in the emergency department. In terms of serum testing, it's most important for the physician to order cardiac: a) troponin. b) lactate dehydrogenase. c) myoglobin. d) creatine kinase.

a) troponin. This client exhibits signs of myocardial infarction (MI), and the most accurate serum determinant of an MI is troponin level. Creatine kinase, lactate dehydrogenase and myoglobin tests can show evidence of muscle injury, but they're less specific indicators of myocardial damage than troponin.

After receiving change-of-shift report on a heart failure unit, which patient should the nurse assess first?

a. A patient who is cool and clammy, with new-onset confusion and restlessness The patient who has "wet-cold" clinical manifestations of heart failure is perfusing inadequately and needs rapid assessment and changes in management.

A patient admitted with heart failure appears very anxious and complains of shortness of breath. Which nursing actions would be appropriate to alleviate this patient's anxiety (select all that apply)? .

a. Administer ordered morphine sulfate. b. Position patient in a semi-Fowler's position. d. Instruct patient on the use of relaxation techniques. e. Use a calm, reassuring approach while talking to patient Morphine sulfate reduces anxiety and may assist in reducing dyspnea. The patient should be positioned in semi-Fowler's position to improve ventilation that will reduce anxiety. Relaxation techniques and a calm reassuring approach will also serve to reduce anxiety.

Which complication of diabetes causes the most deaths? a. Cardiovascular effects b. Hypoglycemia c. Ketoacidosis d. Kidney disease

a. Cardiovascular effects

Based on the Joint Commission Core Measures for patients with heart failure, which topics should the nurse include in the discharge teaching plan for a patient who has been hospitalized with chronic heart failure (select all that apply)?

a. How to take and record daily weight c. Date and time of follow-up appointment d. Symptoms indicating worsening heart failure e. Actions and side effects of prescribed medications he Joint Commission Core Measures state that patients should be taught about prescribed medications, follow-up appointments, weight monitoring, and actions to take for worsening symptoms. Patients with heart failure are encouraged to begin or continue aerobic exercises such as walking, while self-monitoring to avoid excessive fatigue.

The patient has heart failure (HF) with an ejection fraction of less than 40%. What core measures should the nurse expect to include in the plan of care for this patient (select all that apply)?

a. Left ventricular function is documented. d. Prescription for angiotensin-converting enzyme (ACE) inhibitor at discharge e. Education materials about activity, medications, weight monitoring, and what to do if symptoms worsen The Joint Commission has identified these three core measures for heart failure patients. Although controlling dysrhythmias will improve CO and workload, it will not eliminate HF.

A patient with chronic HF and atrial fibrillation is treated with a digitalis glycoside and a loop diuretic. To prevent possible complications of this combination of drugs, what does the nurse need to do (select all that apply)?

a. Monitor serum potassium levels b. teach the patient how to take a pulse rate. Hypokalemia, which can be caused by the use of potassium-depleting diuretics (e.g., thiazides, loop diuretics), is one of the most common causes of digitalis toxicity. Low serum levels of potassium enhance the action of digitalis, causing a therapeutic dose to achieve toxic levels. Hypokalemia can also precipitate dysrhythmias. Monitoring the serum potassium levels of patients receiving digitalis preparations and potassium-depleting diuretics is essential. Patients taking digitalis preparations should be taught how to measure their pulse rate because bradycardia and atrioventricular blocks are late signs of digitalis toxicity. In addition, patients should know what pulse rate would necessitate a call to the health care provider.

During a visit to a 78-year-old with chronic heart failure, the home care nurse finds that the patient has ankle edema, a 2-kg weight gain over the past 2 days, and complains of "feeling too tired to get out of bed." Based on these data, the best nursing diagnosis for the patient is

a. activity intolerance related to fatigue. The patient's statement supports the diagnosis of activity intolerance.

when obtaining a new viral of NPH insulin from the refrigerator, the nurse notes that the suspension is partially frozen. What should the nurse do? a. discard the vial and obtain a new one from the pharmacy b. shake the vial vigorously to produce heat c. warm the suspension in warm water d. withdraw the unfrozen portion then discard the rest

a. discard the vial and obtain a new one from the pharmacy

The nurse assesses for which symptom of the electrolyte balance imbalance that is most likely when a patient is receiving large doses of insulin? a. muscle weakness and constipation b. restlessness and irritability c. spasms of the wrist and fingers when circulation of the upper arm is constricted for several minutes d. twitching of the facial nerve when the face is tapped over the nerve

a. muscle weakness and constipation

A nurse administered 30 units of glargine (Lantus) insulin in the same syringe as 8 units of aspart (Humalog) insulin coverage so that the patient would need only one injection. The nurse contacts the prescriber and monitors the patient for which symptom caused by the effects of this combination on the absorption of the insulin? a. profuse sweating b. itching c. thirst d. vomiting

a. profuse sweating

2. When a patient with acute kidney injury (AKI) has an arterial blood pH of 7.30, the nurse will expect an assessment finding of a. persistent skin tenting b. rapid, deep respirations. c. bounding peripheral pulses. d. hot, flushed face and neck.

b

3. The nurse is planning care for a patient with severe heart failure who has developed elevated blood urea nitrogen (BUN) and creatinine levels. The primary collaborative treatment goal in the plan will be a. augmenting fluid volume. b. maintaining cardiac output. c. diluting nephrotoxic substances. d. preventing systemic hypertension.

b

A client is ordered a nitroglycerine transdermal patch for treatment of CAD and asks the nurse why the patch is removed at bedtime. Which is the best response by the nurse? a) "Contact dermatitis and skin irritations are common when the patch remains on all day." b) "Removing the patch at night prevents drug tolerance while keeping the benefits." c) "Nitroglycerine causes headaches, but removing the patch decreases the incidence." d) "You do not need the effects of nitroglycerine while you sleep."

b) "Removing the patch at night prevents drug tolerance while keeping the benefits." Tolerance to antiangina effects of nitrates can occur when taking these drugs for long periods of time. Therefore, to prevent tolerance and maintain benefits, it is a common regime to remove transdermal patches at night. Common adverse effects of nitroglycerine are headaches and contact dermatitis but not the reason for removing the patch at night. It is true that while you rest, there is less demand on the heart but not the primary reason for removing the patch.

A nurse recognizes that a client with tuberculosis needs further teaching when the client states: a) "I'll have to take these medications for 9 to 12 months." b) "The people I have contact with at work should be checked regularly." c) "I'll need to have scheduled laboratory tests while I'm on the medication." d) "It won't be necessary for the people I work with to take medication."

b) "The people I have contact with at work should be checked regularly." Explanation: The client requires additional teaching if he states that coworkers need to be checked regularly. Such casual contacts needn't be tested for tuberculosis. However, a person in close contact with a person who's infectious is at risk and should be checked. The client demonstrates effective teaching if he states that he'll take his medications for 9 to 12 months, that coworkers don't need medication, and that he requires laboratory tests while on medication. Coworkers not needing medications, taking the medication for 9 to 12 months, and having scheduled laboratory tests are all appropriate statements. pg.587

A nurse is reviewing a client's X-ray. The X-ray shows an endotracheal (ET) tube placed 3/4? (2 cm) above the carina and reveals nodular lesions and patchy infiltrates in the upper lobe. Which interpretation of the X-ray is accurate? a) The X-ray is inconclusive. b) A disease process is present. c) The ET tube must be pulled back. d) The ET tube must be advanced.

b) A disease process is present. Explanation: This X-ray suggests tuberculosis. An ET tube that's 3/4? above the carina is at an adequate level in the trachea. There's no need to advance it or pull it back. pg.588

A nurse is reevaluating a client receiving IV fibrinolytic therapy. Which of the following patient findings requires immediate intervention by the nurse? a) Minimal oozing of blood from the IV site b) Altered level of consciousness c) Chest pain: 2 of 10 (1-to-10 pain scale) d) Presence of reperfusion dysrhythmias

b) Altered level of consciousness A patient receiving fibrinolytic therapy is at risk for complications associated with bleeding. Altered level of consciousness may indicate hypoxia and intracranial bleeding and the infusion should be discontinued immediately. Minimal bleeding requires manual pressure. Reperfusion dysrhythmias are an expected finding. A chest pain score of 2 is low, and indicates the patient's chest pain is subsiding, an expected outcome of this therapy.

A client is admitted to the emergency department with a stab wound and is now presenting dyspnea, tachypnea, and sucking noise heard on inspiration and expiration. The nurse should care for the wound in which manner? a) Apply vented dressing. b) Apply airtight dressing. c) Apply direct pressure to the wound. d) Clean the wound and leave open to the air.

b) Apply airtight dressing. Explanation: The client has developed a pneumothorax, and the best action is to prevent further deflation of the affected lung by placing an airtight dressing over the wound. A vented dressing would be used in a tension pneumothorax, but because air is heard moving in and out, a tension pneumothorax is not indicated. Applying direct pressure is required if active bleeding is noted. pg.613

A client is returning from the operating room after inguinal hernia repair. The nurse notes that he has fluid volume excess from the operation and is at risk for left-sided heart failure. Which sign or symptom indicates left-sided heart failure? a) Dependent edema b) Bibasilar crackles c) Right upper quadrant pain d) Jugular vein distention

b) Bibasilar crackles

An elderly client is diagnosed with pulmonary tuberculosis. Upset and tearful, he asks the nurse how long he must be separated from his family. Which nursing diagnosis is most appropriate for this client? a) Social isolation b) Deficient knowledge (disease process and treatment regimen) c) Impaired social interaction d) Anxiety

b) Deficient knowledge (disease process and treatment regimen) Explanation: This client is exhibiting Deficient knowledge about the disease process and treatment regimen; treatment of tuberculosis no longer requires isolation, provided the client complies with the ordered medication regimen. Although the client is upset, his question reflects sadness at the prospect of being separated from his family rather than anxiety about the disease. Because he has just been diagnosed and hasn't had a chance to demonstrate compliance, a nursing diagnosis of Social isolation isn't appropriate. A diagnosis of Impaired social interaction usually has a psychiatric or neurologic basis, not a respiratory one, such as pulmonary tuberculosis. pg.587

A recent immigrant is diagnosed with pulmonary tuberculosis (TB). Which intervention is the most important for the nurse to implement with this client? a) Client teaching about the cause of TB b) Developing a list of people with whom the client has had contact c) Client teaching about the importance of TB testing d) Reviewing the risk factors for TB

b) Developing a list of people with whom the client has had contact Explanation: To lessen the spread of TB, everyone who had contact with the client must undergo a chest X-ray and TB skin test. Testing will help determine if the client infected anyone else. Teaching about the cause of TB, reviewing the risk factors, and the importance of testing are important areas to address when educating high-risk populations about TB before its development. pg.590

The nurse is obtaining data on an older adult client. What finding may indicate to the nurse the early symptom of heart failure? a) Hypotension b) Dyspnea on exertion c) Tachycardia d) Decreased urinary output

b) Dyspnea on exertion

A patient with congestive heart failure is admitted to the hospital with complaints of shortness of breath. How should the nurse position the patient in order to decrease preload? a) Supine with arms elevated on pillows above the level of the heart b) Head of the bed elevated at 45 degrees and lower arms supported by pillows c) Prone with legs elevated on pillows d) Head of the bed elevated at 30 degrees and legs elevated on pillows

b) Head of the bed elevated at 45 degrees and lower arms supported by pillows

A patient diagnosed with a myocardial infarction (MI) has begun an active rehabilitation program. The nurse recognizes an overall goal of rehabilitation for a patient who has had an MI includes which of the following? a) Returning the patient to work and a preillness lifestyle b) Improvement of the quality of life c) Prevention of another cardiac event d) Limiting the effects and progression of atherosclerosis

b) Improvement of the quality of life Overall, cardiac rehabilitation is a complete program dedicated to extending and improving quality of life.

The client, with a lower respiratory airway infection, is presenting with the following symptoms: fever, chills, dry hacking cough, and wheezing. Which nursing diagnosis best supports the assessment by the nurse? a) Risk for Infection b) Ineffective Airway Clearance c) Ineffective Breathing Pattern d) Impaired Gas Exchange

b) Ineffective Airway Clearance Explanation: The symptom of wheezing indicates a narrowing or partial obstruction of the airway from inflammation or secretions. Risk for Infection is a real potential because the client is already exhibiting symptoms of infection (fever with chills). Impaired Gas Exchange may occur, but no symptom listed supports poor exchange of gases. No documentation of respiratory rate or abnormalities is listed to justify this nursing diagnosis. pg. 582

Which of the following is the hallmark of systolic heart failure? a) Basilar crackles b) Low ejection fraction (EF) c) Limitation of activities of daily living (ADLs) d) Pulmonary congestion

b) Low ejection fraction (EF)

A client develops cardiogenic pulmonary edema and is extremely apprehensive. What medication can the nurse administer with physician orders that will relieve anxiety and slow respiratory rate? a) Dopamine (Intropin) b) Morphine sulfate c) Nitroglycerin d) Furosemide (Lasix)

b) Morphine sulfate

A physician orders digoxin (Lanoxin) for a client with heart failure. During digoxin therapy, which laboratory value may predispose the client to digoxin toxicity? a) Magnesium level of 2.5 mg/dl b) Potassium level of 2.8 mEq/L c) Sodium level of 152 mEq/L d) Calcium level of 7.5 mg/dl

b) Potassium level of 2.8 mEq/L

Which of the following actions is most appropriate for the nurse to take when the patient demonstrates subcutaneous emphysema along the suture line or chest dressing 2 hours after chest surgery? a) Measure the patient's pulse oximetry b) Record the observation c) Report the finding to the physician immediately d) Apply a compression dressing to the area

b) Record the observation Explanation: Subcutaneous emphysema occurs after chest surgery as the air that is located within the pleural cavity is expelled through the tissue opening created by the surgical procedure. Subcutaneous emphysema is a typical postoperative finding in the patient after chest surgery. Subcutaneous emphysema is absorbed by the body spontaneously after the underlying leak is treated or halted. Subcutaneous emphysema results from air entering the tissue planes. pg.614

A nurse assesses a patient diagnosed in the prerenal stage of ARF. The nurse expects to find the following signs and symptoms. Select all that apply. a) BUN value of <10 mg/dL b) Urine specific gravity of 1.029 c) Creatinine level of 1.3 mg/dL d) Urine sodium <20 mEq/L e) Increase in urinary sediment f) Urine osmolality of 350 mOsm/Kg

b) Urine specific gravity of 1.029 c) Creatinine level of 1.3 mg/dL d) Urine sodium <20 mEq/L The BUN reading is within normal range, the urine osmolality would be greater than 500 mOsm/kg, and there would be few hyaline casts. Refer to Table 27-2 in the text.

A client with chest pain doesn't respond to nitroglycerin. When he's admitted to the emergency department, the health care team obtains an electrocardiogram and administers I.V. morphine. The physician also considers administering alteplase (Activase). This thrombolytic agent must be administered how soon after onset of myocardial infarction (MI) symptoms? a) Within 5 to 7 days b) Within 6 hours c) Within 12 hours d) Within 24 to 48 hours

b) Within 6 hours For the best chance of salvaging the client's myocardium, a thrombolytic agent must be administered within 6 hours after onset of chest pain or other signs or symptoms of MI. Sudden death is most likely to occur within the first 24 hours after an MI. Physicians initiate I.V. heparin therapy after administration of a thrombolytic agent; it usually continues for 5 to 7 days.

A client with acute renal failure is undergoing dialysis for the first time. The nurse monitors the client closely for dialysis equilibrium syndrome, a complication that's most common during the first few dialysis sessions. Typically, dialysis equilibrium syndrome causes: a) acute bone pain and confusion. b) confusion, headache, and seizures. c) weakness, tingling, and cardiac arrhythmias. d) hypotension, tachycardia, and tachypnea.

b) confusion, headache, and seizures. Dialysis equilibrium syndrome causes confusion, a decreasing level of consciousness, headache, and seizures. These findings, which may last several days, probably result from a relative excess of interstitial or intracellular solutes caused by rapid solute removal from the blood. The resultant organ swelling interferes with normal physiological functions. To prevent this syndrome, many dialysis centers keep first-time sessions short and use a reduced blood flow rate. Acute bone pain and confusion are associated with aluminum intoxication, another potential complication of dialysis. Weakness, tingling, and cardiac arrhythmias suggest hyperkalemia, which is associated with renal failure. Hypotension, tachycardia, and tachypnea signal hemorrhage, another dialysis complication.

A Class 1 with regards to TB indicates a) disease that is not clinically active. b) exposure and no evidence of infection. c) no exposure and no infection. d) latent infection with no disease.

b) exposure and no evidence of infection. Explanation: Class 1 is exposure, but no evidence of infection. Class 0 is no exposure and no infection. Class 2 is a latent infection, with no disease. Class 4 is disease, but not clinically active. pg.588

Arterial blood gas analysis would reveal which of the following related to acute respiratory failure? a) pH 7.35 b) pH 7.28 c) PaCO 32 mm Hg d) PaO 80 mm Hg

b) pH 7.28 Explanation: Acute respiratory failure (ARF) is defined as a decrease in the arterial oxygen tension (PaO) to less than 50 mm Hg (hypoxemia) and an increase in arterial carbon dioxide tension (PaCO) to greater than 50 mm Hg (hypercapnia), with an arterial pH of less than 7.35. pg.597

When interpreting the results of a Mantoux test, the nurse explains to the patient that a reaction occurs when the intradermal injection site shows a) bruising. b) redness and induration. c) drainage. d) tissue sloughing.

b) redness and induration. Explanation: The injection site is inspected for redness and palpated for hardening. Drainage at the injection site does not indicate a reaction to the tubercle bacillus. Sloughing of tissue at the injection site does not indicate a reaction to the tubercle bacillus. Bruising of tissue at the site may occur from the injection, but does not indicate a reaction to the tubercle bacillus. pg.588

A patient who has type 2 DM has been unable to follow the recommended diet and exercise regimen. He tries to alter his laboratory test results by eating less than usual before having blood testing performed. Which test would be most accurate for this patient because it evaluates his glucose control over the past 3 months? a. fasting glucose b. A l c c. Postprandial glucose d. two- hour glucose tolerance

b. A l c

After receiving change-of-shift report on a heart failure unit, which patient should the nurse assess first?

b. Patient who is taking digoxin and has a potassium level of 3.1 mEq/L The patient's low potassium level increases the risk for digoxin toxicity and potentially fatal dysrhythmias

The nurse is preparing to administer digoxin to a patient with heart failure. In preparation, laboratory results are reviewed with the following findings: sodium 139 mEq/L, potassium 5.6 mEq/L, chloride 103 mEq/L, and glucose 106 mg/dL. What should the nurse do next?

b. Withhold the dose and report the potassium level. The normal potassium level is 3.5 to 5.0 mEq/L.

What would be the most appropriate nursing intervention for a 23- year- old patient who, for the first time, has a fasting plasma glucose level of 145 mg/dL a. advise the patient to include 30 minutes of vigorous exercise in his daily activities b. discuss possible diet changes with the patient c. explain the need for oral anti diabetic medication to the patient d. teach the patient how to do a urine dip for glucose and ketones

b. discuss possible diet changes with the patient

A patient with diabetes who has been using traditional insulin therapy has been prescribed intensive insulin therapy to achieve tighter glucose control. Which information should be included in the teaching? a. an insulin pump is used to provide the best glucose control and requires about the same amount of attention as intensive insulin therapy b. intensive insulin therapy usually requires four injections of a rapid- acting insulin each day in addition to an injection of a basal insulin c. intensive insulin therapy is indicated only for newly diagnosed type 1 diabetics who have never experienced ketoacidosis d. studies have shown that intensive insulin therapy is most effective in preventing the microvascular complications characteristic of type 2 DM

b. intensive insulin therapy usually requires four injections of a rapid- acting insulin each day in addition to an injection of a basal insulin

The nurse is reviewing the results of a total cholesterol level for a patient who has been taking simvastatin (Zocor). What results display the effectiveness of the medication? a) 250-275 mg/dL b) 210-240 mg/dL c) 160-190 mg/dL d) 280-300 mg/dL

c) 160-190 mg/dL Simvastatin (Zocor) is a statin frequently given as initial therapy for significantly elevated cholesterol and low-density lipoprotein levels. Normal total cholesterol is less than 200 mg/dL.

Shortly after being admitted to the coronary care unit with an acute myocardial infarction (MI), a client reports midsternal chest pain radiating down his left arm. The nurse notes that the client is restless and slightly diaphoretic, and measures a temperature of 99.6° F (37.6° C); a heart rate of 102 beats/minute; regular, slightly labored respirations at 26 breaths/minute; and a blood pressure of 150/90 mm Hg. Which nursing diagnosis takes highest priority? a) Decreased cardiac output b) Risk for imbalanced body temperature c) Acute pain d) Anxiety

c) Acute pain The nursing diagnosis of Acute pain takes highest priority because it increases the client's pulse and blood pressure. During the acute phase of an MI, low-grade fever is an expected result of the body's response to myocardial tissue necrosis. This makes Risk for imbalanced body temperature an incorrect answer. The client's blood pressure and heart rate don't suggest a nursing diagnosis of Decreased cardiac output. Anxiety could be an appropriate nursing diagnosis, but addressing Acute pain (the priority concern) may alleviate the client's anxiety.

You are presenting a workshop at the senior citizens center about how the changes of aging predispose clients to vascular occlusive disorders. What would you name as the most common cause of peripheral arterial problems in the older adult? a) Raynaud's disease b) Coronary thrombosis c) Atherosclerosis d) Arteriosclerosis

c) Atherosclerosis Atherosclerosis is the most common cause of peripheral arterial problems in the older adult. The disease correlates with the aging process. The other choices may occur at any age.

A client with chest pain arrives in the emergency department and receives nitroglycerin, morphine (Duramorph), oxygen, and aspirin. The physician diagnoses acute coronary syndrome. When the client arrives on the unit, his vital signs are stable and he has no complaints of pain. The nurse reviews the physician's orders. In addition to the medications already given, which medication does the nurse expect the physician to order? a) Nitroprusside (Nipride) b) Furosemide (Lasix) c) Carvedilol (Coreg) d) Digoxin (Lanoxin)

c) Carvedilol (Coreg) A client with suspected myocardial infarction should receive aspirin, nitroglycerin, morphine, and a beta-adrenergic blocker such as carvedilol. Digoxin treats arrhythmias; there is no indication that the client is having arrhythmias. Furosemide is used to treat signs of heart failure, which isn't indicated at this point. Nitroprusside increases blood pressure. This client has stable vital signs and isn't hypotensive.

The nurse is educating a patient who is required to restrict potassium intake. What foods would the nurse suggest the patient eliminate that are rich in potassium? a) Cooked white rice b) Salad oils c) Citrus fruits d) Butter

c) Citrus fruits Foods and fluids containing potassium or phosphorus (e.g., bananas, citrus fruits and juices, coffee) are restricted.

A nurse taking care of a patient recently admitted to the ICU observes the patient coughing up large amounts of pink, frothy sputum. Auscultation of the lungs reveals course crackles to lower lobes bilaterally. Based on this assessment, the nurse recognizes this patient is developing which of the following problems? a) Bilateral pneumonia b) Acute exacerbation of chronic obstructive pulmonary disease c) Decompensated heart failure with pulmonary edema d) Tuberculosis

c) Decompensated heart failure with pulmonary edema

A nurse is caring for a patient diagnosed with empyema. Which of the following interventions does a nurse implement for patients with empyema? a) Institute droplet precautions. b) Do not allow visitors with respiratory infection. c) Encourage breathing exercises. d) Place suspected patients together.

c) Encourage breathing exercises. Explanation: The nurse teaches the patient with empyema to do breathing exercises as prescribed. The nurse should institute droplet precautions, isolate suspected and confirmed influenza patients in private rooms, or place suspected and confirmed patients together, and not allow visitors with symptoms of respiratory infection to visit the hospital to prevent outbreaks of influenza from occurring in health care settings. pg.595

Which of the following therapies are for patient who have advanced heart failure (HF) after all other therapies have failed? a) Cardiac resynchronization therapy b) Ventricular access device c) Heart transplant d) Implantable cardiac defibrillator (ICD)

c) Heart transplant

In a client with chronic bronchitis, which sign would lead the nurse to suspect right-sided heart failure? a) Productive cough b) Cyanosis of the lips c) Leg edema d) Bilateral crackles

c) Leg edema

A patient presents to the ED complaining of anxiety and chest pain after shoveling heavy snow that morning. The patient says that he has not taken nitroglycerin for months but did take three nitroglycerin tablets and although the pain is less, "They did not work all that well. " The patient shows the nurse the nitroglycerin bottle and the prescription was filled 12 months ago. The nurse anticipates which of the following physician orders? a) Serum electrolytes b) Ativan 1 mg orally c) Nitroglycerin SL d) Chest x-ray

c) Nitroglycerin SL Nitroglycerin is volatile and is inactivated by heat, moisture, air, light, and time. Nitroglycerin should be renewed every 6 months to ensure full potency. The client's tablets were expired and the nurse should anticipate administering nitroglycerin to assess if the chest pain subsides. The other choices may be ordered at a later time, but the priority is to relieve the patient's chest pain.

A client has hypoxemia of pulmonary origin. What portion of arterial blood gas results is most useful in distinguishing between acute respiratory distress syndrome and acute respiratory failure? a) Partial pressure of arterial carbon dioxide (PaCO2) b) pH c) Partial pressure of arterial oxygen (PaO2) d) Bicarbonate (HCO3-)

c) Partial pressure of arterial oxygen (PaO2) Explanation: In acute respiratory failure, administering supplemental oxygen elevates the PaO2. In acute respiratory distress syndrome, elevation of the PaO2 requires positive end-expiratory pressure. In both situations, the PaCO2 is elevated and the pH and HCO3- are depressed. pg.595

A nurse in the emergency department is caring for a client with acute heart failure. Which laboratory value is most important for the nurse to check before administering medications to treat heart failure? a) White blood cell (WBC) count b) Calcium c) Potassium d) Platelet count

c) Potassium

A client is admitted to the health care facility with active tuberculosis (TB). The nurse should include which intervention in the care plan? a) Keeping the door to the client's room open to observe the client b) Instructing the client to wear a mask at all times c) Putting on an individually fitted mask when entering the client's room d) Wearing a gown and gloves when providing direct care

c) Putting on an individually fitted mask when entering the client's room Explanation: Because TB is transmitted by droplet nuclei from the respiratory tract, the nurse should put on a mask when entering the client's room. Occupation Safety and Health Administration standards require an individually fitted mask. Having the client wear a mask at all times would hinder sputum expectoration and respirations would make the mask moist. A nurse who doesn't anticipate contact with the client's blood or body fluids need not wear a gown or gloves when providing direct care. A client with TB should be in a room with laminar airflow, and the room's door should be shut at all times. pg.591

Based on her knowledge of the primary cause of ESRD, the nurse knows to assess the most important indicator. What is that indicator? a) pH and HCO3 b) Blood pressure c) Serum glucose d) Urine protein

c) Serum glucose The nurse would evaluate serum and urine levels of glucose because diabetes is the primary cause of renal failure.

A patient has been diagnosed with systolic heart failure. The nurse would expect the patient's ejection fraction to be at which level? a) Normal b) High c) Slightly reduced d) Severely reduced

c) Slightly reduced

A client develops acute renal failure (ARF) after receiving I.V. therapy with a nephrotoxic antibiotic. Because the client's 24-hour urine output totals 240 ml, the nurse suspects that the client is at risk for: a) dehydration. b) paresthesia. c) cardiac arrhythmia. d) pruritus.

c) cardiac arrhythmia. As urine output decreases, the serum potassium level rises; if it rises sufficiently, hyperkalemia may occur, possibly triggering a cardiac arrhythmia. Hyperkalemia doesn't cause paresthesia (sensations of numbness and tingling). Dehydration doesn't occur during this oliguric phase of ARF, although typically it does arise during the diuretic phase. In the client with ARF, pruritus results from increased phosphates and isn't associated with hyperkalemia.

A client is receiving captopril (Capoten) for heart failure. The nurse should notify the physician that the medication therapy is ineffective if an assessment reveals: a) postural hypotension. b) dry cough. c) peripheral edema. d) skin rash.

c) peripheral edema.

A patient with heart failure has a new order for captopril (Capoten) 12.5 mg PO. After administering the first dose and teaching the patient about the drug, which statement by the patient indicates that teaching has been effective?

c. "I will call for help when I need to get up to use the bathroom." Captopril can cause hypotension, especially after the initial dose, so it is important that the patient not get up out of bed without assistance until the nurse has had a chance to evaluate the effect of the first dose.

The nurse has done an initial assessment on assigned patients and is preparing to administer prescribed insulin. All patients are alert and oriented. It is 8:07 am, and breakfast try are scheduled to arrive at 8:45 am. The nurse should administer the insulin first to the patient who has a. 8:00 am SMBG 100 mg/dL; prescribed insulin aspart (NovoLog) 5 units b. 8:00 am SMBG 110 mg/dL; prescribed insulin glulisine (Apidra) 5 units c. 8:00 am SMBG 80 mg/dL; prescribed NPH insulin (Novolin N) 34 units d. 8:00 am SMBG 90 mg/dL; prescribed regular insulin (Humulin R) 5 units

c. 8:00 am SMBG 80 mg/dL; prescribed NPH insulin (Novolin N) 34 units

A patient with a history of chronic heart failure is admitted to the emergency department (ED) with severe dyspnea and a dry, hacking cough. Which action should the nurse do first?

c. Auscultate the breath sounds. This patient's severe dyspnea and cough indicate that acute decompensated heart failure (ADHF) is occurring.

Which diagnostic test will be most useful to the nurse in determining whether a patient admitted with acute shortness of breath has heart failure?

c. B-type natriuretic peptide B-type natriuretic peptide (BNP) is secreted when ventricular pressures increase, as they do with heart failure.

Which topic will the nurse plan to include in discharge teaching for a patient with systolic heart failure and an ejection fraction of 33%?

c. Benefits and side effects of angiotensin-converting enzyme (ACE) inhibitors The core measures for the treatment of heart failure established by The Joint Commission indicate that patients with an ejection fraction (EF) <40% receive an ACE inhibitor to decrease the progression of heart failure.

What is the priority assessment by the nurse caring for a patient receiving IV nesiritide (Natrecor) to treat heart failure?

c. Blood pressure Although all identified assessments are appropriate for a patient receiving IV nesiritide, the priority assessment would be monitoring for hypotension, the main adverse effect of nesiritide.

An outpatient who has chronic heart failure returns to the clinic after 2 weeks of therapy with metoprolol (Toprol XL). Which assessment finding is most important for the nurse to report to the health care provider?

c. Blood pressure (BP) of 88/42 mm Hg The patient's BP indicates that the dose of metoprolol may need to be decreased because of hypotension.

A patient has recently started on digoxin (Lanoxin) in addition to furosemide (Lasix) and captopril (Capoten) for the management of heart failure. Which assessment finding by the home health nurse is a priority to communicate to the health care provider?

c. Serum potassium level 3.0 mEq/L after 1 week of therapy Hypokalemia can predispose the patient to life-threatening dysrhythmias (e.g., premature ventricular contractions), and potentiate the actions of digoxin and increase the risk for digoxin toxicity, which can also cause life-threatening dysrhythmias.

IV sodium nitroprusside (Nipride) is ordered for a patient with acute pulmonary edema. During the first hours of administration, the nurse will need to titrate the nitroprusside rate if the patient develops

c. a systolic BP <90 mm Hg. Sodium nitroprusside is a potent vasodilator, and the major adverse effect is severe hypotension.

A patient with chronic heart failure who is taking a diuretic and an angiotensin-converting enzyme (ACE) inhibitor and who is on a low-sodium diet tells the home health nurse about a 5-pound weight gain in the last 3 days. The nurse's priority action will be to.

c. assess the patient for clinical manifestations of acute heart failure. The 5-pound weight gain over 3 days indicates that the patient's chronic heart failure may be worsening.

While assessing a 68-year-old with ascites, the nurse also notes jugular venous distention (JVD) with the head of the patient's bed elevated 45 degrees. The nurse knows this finding indicates

c. increased right atrial pressure. jugular veins empty into the superior vena cava and then into the right atrium, so JVD with the patient sitting at a 45-degree angle reflects increased right atrial pressure.

A compensatory mechanism involved in HF that leads to inappropriate fluid retention and additional workload of the heart is:

c. neurohormonal response The following mechanisms in heart failure lead to inappropriate fluid retention and additional workload of the heart: activation of the renin-angiotensin-aldosterone system (RAAS) cascade and release of antidiuretic hormone from the posterior pituitary gland in response to low cerebral perfusion pressure that results from low cardiac output.

The nurse plans discharge teaching for a patient with chronic heart failure who has prescriptions for digoxin (Lanoxin) and hydrochlorothiazide (HydroDIURIL). Appropriate instructions for the patient include

c. notify the health care provider if nausea develops. Nausea is an indication of digoxin toxicity and should be reported so that the provider can assess the patient for toxicity and adjust the digoxin dose, if necessary.

A patient who has chronic heart failure tells the nurse, "I was fine when I went to bed, but I woke up in the middle of the night feeling like I was suffocating!" The nurse will document this assessment finding as

c. paroxysmal nocturnal dyspnea. Paroxysmal nocturnal dyspnea is caused by the reabsorption of fluid from dependent body areas when the patient is sleeping and is characterized by waking up suddenly with the feeling of suffocation.

It would be a priority for the nurse to respond to which symptoms if exhibited by a patient who is receiving insulin therapy for diabetes? a. fatigue and blurred vision b. perineal itching and copious urine c. profuse sweating and difficult to arouse d. thirst and constant hunger

c. profuse sweating and difficult to arouse

A nurse is administering a purified protein derivative (PPD) test to a client. Which statement concerning PPD testing is true? a) A negative reaction always excludes the diagnosis of TB. b) The PPD can be read within 12 hours after the injection. c) A positive reaction indicates that the client has active tuberculosis (TB). d) A positive reaction indicates that the client has been exposed to the disease.

d) A positive reaction indicates that the client has been exposed to the disease. Explanation: A positive reaction means the client has been exposed to TB; it isn't conclusive for the presence of active disease. A positive reaction consists of palpable swelling and induration of 5 to 15 mm. It can be read 48 to 72 hours after the injection. In clients with positive reactions, further studies are usually done to rule out active disease. In immunosuppressed clients, a negative reaction doesn't exclude the presence of active disease. pg.588

A client in the emergency department complains of squeezing substernal pain that radiates to the left shoulder and jaw. He also complains of nausea, diaphoresis, and shortness of breath. What should the nurse do? a) Complete the client's registration information, perform an electrocardiogram, gain I.V. access, and take vital signs. b) Alert the cardiac catheterization team, administer oxygen, attach a cardiac monitor, and notify the physician. c) Gain I.V. access, give sublingual nitroglycerin, and alert the cardiac catheterization team. d) Administer oxygen, attach a cardiac monitor, take vital signs, and administer sublingual nitroglycerin.

d) Administer oxygen, attach a cardiac monitor, take vital signs, and administer sublingual nitroglycerin. Cardiac chest pain is caused by myocardial ischemia. Therefore the nurse should administer supplemental oxygen to increase the myocardial oxygen supply, attach a cardiac monitor to help detect life-threatening arrhythmias, and take vital signs to ensure that the client isn't hypotensive before giving sublingual nitroglycerin for chest pain. Registration information may be delayed until the client is stabilized. Alerting the cardiac catheterization team or the physician before completing the initial assessment is premature.

The nurse recognizes which of the following lab tests is a key diagnostic indicator of heart failure? a) Blood urea nitrogen (BUN) b) Complete blood count (CBC) c) Creatinine d) Brain natriuretic peptide (BNP)

d) Brain natriuretic peptide (BNP)

Which of the following medications is given to patients diagnosed with angina and is allergic to aspirin? a) Diltiazem (Cardizem) b) Felodipine (Plendil) c) Amlodipine (Norvasc) d) Clopidogrel (Plavix)

d) Clopidogrel (Plavix) Plavix or Ticlid is given to patients who are allergic to aspirin or given in addition to aspirin to patients at high risk for MI. Norvasc, Cardizem, and Plendil are calcium channel blockers.

The nurse identifies which of the following symptoms as a manifestation of right-sided heart failure (HF)? a) Reduction in cardiac output b) Reduction in forward flow c) Accumulation of blood in the lungs d) Congestion in the peripheral tissues

d) Congestion in the peripheral tissues

A client has been symptomatic for several months and is seeing a cardiologist for diagnostics to determine the cause of his cardiac symptoms. You review the diagnostic procedures the cardiologist will perform. How will the client's ejection fraction be measured? a) Cardiac ultrasound b) Cardiac catheterization c) Electrocardiogram d) Echocardiogram

d) Echocardiogram

Which of the following interventions does a nurse implement for patients with empyema? a) Institute droplet precautions b) Do not allow visitors with respiratory infections c) Place suspected patients together d) Encourage breathing exercises

d) Encourage breathing exercises Explanation: The nurse instructs the patient in lung-expanding breathing exercises to restore normal respiratory function. pg.595

Which of the following symptoms should the nurse expect to find as an early symptom of chronic heart failure? a) Pedal edema b) Nocturia c) Irregular pulse d) Fatigue

d) Fatigue

A patient's elevated cholesterol levels are being managed with Lipitor, 40 mg daily. The nurse practitioner reviews the patient's blood work every 6 months before renewing the prescription. The nurse explains to the patient's daughter that this is necessary because of a major side effect of Lipitor that she is checking for. What is that side-effect? a) Hyperuricemia b) Hyperglycemia c) Gastrointestinal distress d) Increased liver enzymes

d) Increased liver enzymes Myopathy and increased liver enzymes are significant side effects of the statins, HMG-CoA reductase inhibitors that are used to affect lipoprotein metabolism.

After 2-hour onset of acute chest pain, the client is brought to the emergency department for evaluation. Elevation of which diagnostic findings would the nurse identify as suggestive of an acute myocardial infarction at this time? a) WBC (white blood cell) count b) Troponin I c) C-reactive protein d) Myoglobin

d) Myoglobin Myoglobin is a biomarker that rises in 2 to 3 hours after heart damage. Troponin is the gold standard for determining heart damage, but troponin I levels due not rise until 4 to 6 hours after MI. WBCs and C-reactive protein levels will rise but not until about day 3.

A nurse reading a chart notes that the patient had a Mantoux skin test result with no induration and a 1-mm area of ecchymosis. How does the nurse interpret this result? a) Uncertain b) Positive c) Borderline d) Negative

d) Negative Explanation: The size of the induration determines the significance of the reaction. A reaction of 0-4 mm is not considered to be significant. A reaction of 5 mm or greater may be significant in people who are considered to be at risk. An induration of 10 mm or greater is usually considered significant in people who have normal or mildly impaired immunity. pg.588

The nurse is preparing to administer furosemide (Lasix) to a client with severe heart failure. What lab study should be of most concern for this client while taking Lasix? a) Sodium level of 135 b) BNP of 100 c) Hemoglobin of 12 d) Potassium level of 3.1

d) Potassium level of 3.1

A client diagnosed with tuberculosis (TB) is taking medication for the treatment of TB. The nurse should instruct the client that he will be safe from infecting others approximately how long after initiation of the chemotherapy regimen? a) Within 48 hours after initiation of bacteriocidal drugs b) Results vary with each client, so it is difficult to predict c) After completion of 6 months of bacteriocidal drugs d) Two to 3 weeks after initiation of bacteriocidal drugs

d) Two to 3 weeks after initiation of bacteriocidal drugs Explanation: The client needs to take the prescribed medications for approximately 2 to 3 weeks before discontinuing precautions against infecting others. Effectiveness of the drug therapy is determined by negative sputum smears obtained on three consecutive days. Although results can vary among clients, the majority respond to therapy within 2 to 3 weeks. pg.590

The nurse is preparing to administer digoxin to a client with heart failure. The nurse obtains an apical pulse rate for 1 minute and determines a rate of 52 beats/minute. What is the first action by the nurse? a) Administer the medication and inform the charge nurse about the rate. b) Administer atropine to speed the heart rate and then administer the digoxin. c) Administer the medications and then notify the physician. d) Withhold the medication and notify the physician of the heart rate

d) Withhold the medication and notify the physician of the heart rate

A client asks a nurse a question about the Mantoux test for tuberculosis. The nurse should base her response on the fact that the: a) area of redness is measured in 3 days and determines whether tuberculosis is present. b) test stimulates a reddened response in some clients and requires a second test in 3 months. c) presence of a wheal at the injection site in 2 days indicates active tuberculosis. d) skin test doesn't differentiate between active and dormant tuberculosis infection.

d) skin test doesn't differentiate between active and dormant tuberculosis infection. Explanation: The Mantoux test doesn't differentiate between active and dormant infections. If a positive reaction occurs, a sputum smear and culture as well as a chest X-ray are necessary to provide more information. Although the area of redness is measured in 3 days, a second test may be needed; neither test indicates that tuberculosis is active. In the Mantoux test, an induration 5 to 9 mm in diameter indicates a borderline reaction; a larger induration indicates a positive reaction. The presence of a wheal within 2 days doesn't indicate active tuberculosis. pg.588

The nurse is administering a dose of digoxin (Lanoxin) to a patient with heart failure (HF). The nurse would become concerned with the possibility of digitalis toxicity if the patient reported which symptom(s)?

d. Anorexia and nausea Anorexia, nausea, vomiting, blurred or yellow vision, and cardiac dysrhythmias are all signs of digitalis toxicity.

A patient who has just been admitted with pulmonary edema is scheduled to receive the following medications. Which medication should the nurse question before giving?

d. Carvedilol (Coreg) 3.125 mg Although carvedilol is appropriate for the treatment of chronic heart failure, it is not used for patients with acute decompensated heart failure (ADHF) because of the risk of worsening the heart failure.

The nurse is caring for a patient who is receiving IV furosemide (Lasix) and morphine for the treatment of acute decompensated heart failure (ADHF) with severe orthopnea. Which clinical finding is the best indicator that the treatment has been effective?

d. Reduced dyspnea with the head of bed at 30 degrees Because the patient's major clinical manifestation of ADHF is orthopnea (caused by the presence of fluid in the alveoli), the best indicator that the medications are effective is a decrease in dyspnea with the head of the bed at 30 degrees

Which types of diabetes often exists for years before diagnosis, but fasting blood glucose is not elevated because of hyperinsulinemia? a. gestrational diabetes b. juvenile diabetes c. Type I diabetes d. Type II diabetes

d. Type II diabetes

A patient is admitted in a state of diabetic ketoacidosis. The resident orders insulin detemir 0.1 mg/ kg/ hr to be administered by intravenous (IV) drip. What should the nurse do? a. calculate the insulin dose and mix it with 100 mL of normal saline b. calculate the insulin dose and mix it with 100 mL of D5W c. calculate the insulin dose and infuse the solution prepared by the pharmacy d. consult the prescriber STAT

d. consult the prescriber STAT

When evaluating arterial blood gases (ABGs), which value is consistent with metabolic alkalosis? a) PaCO 36 b) pH 7.48 c) HCO 21 mEq/L d) O saturation 95%

pH 7.48

Which set of arterial blood gas (ABG) results requires further investigation? a) pH 7.44, PaCO2 43 mm Hg, PaO2 99 mm Hg, and HCO3- 26 mEq/L b) pH 7.49, PaCO2 30 mm Hg, PaO2 89 mm Hg, and HCO3- 18 mEq/L c) pH 7.35, PaCO2 40 mm Hg, PaO2 91 mm Hg, and HCO3- 22 mEq/L d) pH 7.38, partial pressure of arterial carbon dioxide (PaCO2) 36 mm Hg, partial pressure of arterial oxygen (PaO2) 95 mm Hg, bicarbonate (HCO3-) 24 mEq/L

pH 7.49, PaCO2 30 mm Hg, PaO2 89 mm Hg, and HCO3- 18 mEq/L


संबंधित स्टडी सेट्स

Outcomes Advanced U5 (Going Out, Staying In)

View Set

Brunner & Suddarth's Textbook of Medical-Surgical Nursing

View Set

MLT 112 Week 1 Media Lab OSHA Hazard Communication and Chemical Hygiene

View Set

CH.5 PUBLIC GOODS, CHOICE, AND GOV. FAILURE

View Set

Maths Test- Fractions, Decimals and Percentages

View Set

Econ Final Learning Curve Questions

View Set

Mastering Biology Chapter 13: How Populations Evolve

View Set

IGCSE Physics Formula's - Electricity and Magnetism

View Set